Download as pdf or txt
Download as pdf or txt
You are on page 1of 147

CATEGORIZED MCQ ANSWERS

2015-2020

SURGERY

FACULTY OF MEDICINE – UNIVERSITY OF PERADENIYA


Content
1. Vascular and lymphatic disorders
2. Transplant
3. Upper GIT disorders
4. Lower GIT disorders
5. Pancreas
6. Hepatobiliary disorders
7. Emergencies
8. ENT surgery
9. Ophthalmology
10. Neurosurgery
11. Paediatric surgery
12. Urosurgery
13. Trauma surgery
14. Anaesthesiology
15. Breast surgery
16. Hernia
17. Burns
18. Salivary gland disorders
19. Neck masses
20. Skin and subcutaneous tissue
Vascular and Lymphatic disorders
2020 June
17) 56 year old male with occlusive arterial disease in lower limbs presented with a
gangrene in a left big toe. On examination bilateral popliteal and pedal pulses absent.
He has not received any treatment up to now. Regarding management of this patient?
A. anticoagulants
B. antiplatelet
C. revascularization
D. Urgent big toe amputation
E. Vasodilators

Answers - F T T F T
According to Fontaine classification this patient is having (grade 4) critical limb ischemia, so he needs
treatments in preventing loosing of his big toe and treatments for further worsening of the
pathology, so he needs dual antiplatelet therapy and vasodilators and revascularization. Urgent big
toe amputation is not needed unless revascularization is failed within the relevant time window.

21) A 45 year old woman with adenocarcinoma of sigmoid colon was admitted for elective
sigmoidectomy. She has a history of antiphospolipid syndrome. Most effective option to prevent
venous thromboembolism is,
A. early mobilization
B. IVC filter
C. prophylactic dose of LMWH
D. starts warfarin until 5 days prior to the surgery and then start heparin infusion until 6
hours prior to the surgery
E. TED stockings

Answer - D
 Any pelvic surgery, any active malignancy, any prothombotic condition like APLS is
considered high risk.
 For a high risk group patient we give enoxaparin (thromboprophylaxis) and mechanical
(pneumatic pump/ stockings). (For moderate we might consider only mechanical methods)
 If there are previous episodes of thrombosis due to APLS we need to give warfarin also.
 If this SBA had that detail as well the best response is D
 If it was not mentioned the answer is C + E (give LMWH and TED stockings)
41) 35 years old lady presented with non-healing ulcer on left medial malleolus for 6 months duration.
There is pigmentation around the ulcer. Her legs are mildly edematous since her first pregnancy at
age of 20. From 2 years back she is found to have varicose veins. She had previous 2 cellulitis episodes
& currently on penicillin prophylaxis. What is the condition which describes best reason for her ulcer?
A. Chronic lymphedema
B. Malignancy
C. Previous DVT
D. Recurrent cellulitis
E. Superficial venous reflux

Answer – E Picture suggestive of a venous ulcer with underlying varicose veins.

2019 Nov
17) 64yr old male teacher presented with intermittent claudication after walking a 200m distance.
He is a chronic smoker for 20yrs. Popliteal pulse and dorsalis pedis pulse not palpable.
a) Imaging is essential for further management
b) Expected ABPI =0.3-0.5
c) Revascularization is needed
d) Risk is of developing a cardiovascular event is 3 to 4 times higher than a normal individual
e) Caused by occlusion of superficial femoral artery

Answers - F F F F T
A - this patient is Fontain grade 2 and can be managed conservatively.
B – for intermittent claudication it’s 0.5 to 0.9
C - Revascularization is needed once he develops critical limb ischemia.
D - Claudication is often a marker of silent coronary arterial disease, its extent correlates with the
ABPI; a 0.1 decrease in ABPI below 0.9 is associated with a 10% increase in the relative risk of a
major cardiovascular event (So there is no 3 to 4 times greater risk.)
2019 March
49) 68 years old male patient who is diagnosed to have diabetes mellitus and hypertension
presented with intermittent claudication of his right leg. Claudication distance is 250m. On
examination popliteal and dorsalis pedis pulses were absent. He is on aspirin and atorvastatin and
his diabetic and hypertension control is good. What would be the best management option for him?
A. Angioplasty
B. Bypass surgery
C. Add anticoagulant
D. Dietary advices
E. Exercise programme
Answer – E
So it’s Fontain grade 2 and can be Mx conservatively. It should be supervised exercise therapy to
improve claudication distance.

2018 Nov
20. A 55 year old smoker presented with intermittent claudication of both lower limbs including
buttocks on walking a distance of 70m. He is likely to have,
a) An improvement in walking distance with clopidogrel
b) Critical limb ischemia
c) Erectile dysfunction.
d) Well felt dorsalis pedis pulse.
e) Atherosclerotic plaque involving the aortic bifurcation.

Answers - F F T F T
A – Clopidogrel does not improve claudication distance.
His walking distance will improve with graded exercise therapy ,he is having only intermittent
claudication at the moment ,as he is having buttock pain as well which means he has occlusion in
aorta including the bifurcation which means he can have erectile dysfunction.

49. A 56 year old male patient who is a manual laborer presented with intermittent claudication of
the right leg with claudication distance of 75 m. The arterial duplex scan reveals significant
obstruction of the dorsalis pedis, anterior tibial and posterior tibial arteries. Absent distal circulation.
What is the most appropriate management to this patient?
a) Angiogram and angioplasty
b) Popliteal distal bypass
c) Femoral popliteal distal bypass
d) Vasodilator therapy
e) Supervised exercise program

Answer - A
Being a manual laborer and having a claudication distance of 75m is quiet of a disabling factor.as
there is no distal circulation bypass is not a good option for him. Vasodilator therapy and graded
exercise will not give a satisfactory outcome and for this kind of shorter occlusions in the arterial
tree angioplasty would be a better option

2017 May
31. A 70-year-old female present with a well granulated wound measuring 15cm×10 cm on the
Lateral aspect of the right leg. What is the most appropriate management?
a) Allow healing by secondary infection
b) Fully thickness skin graft
c) Local rotational flap
d) Secondary suturing
e) Split skin graft

Answer – E
In a well granulated wound best management option would be covering with a split skin graft

34. A 30-year-old female is diagnosed with iliofemoral DVT one week after a caesarean section. What
is the most appropriate management?
a) Bed rest, graduated compression stockings and lifelong anticoagulation
b) Bed rest, graduated compression stockings and anticoagulation for a minimum 3 months
c) Bed rest, thrombolysis, graduated compression stockings and anticoagulation for a
minimum of 3 months
d) Early mobilization, graduated compression stockings and anticoagulation for a minimum of
3 months
e) Early mobilization, graduated compression stockings and lifelong antiplatelet medication

Answer - D
Early mobilization gives relief in pain and swelling and reduction of incidence of post thrombotic
syndrome. Since this is a provoked DVT, anticoagulation is needed for only 3 months
 In an already diagnosed patient with DVT we need to mobililize (not bed rest)

2016 Nov
37. 60-year-old male with patellar fracture immobilized for 1 week presented with calf pain. Duplex
scan showed thrombus in common femoral artery. Most appropriate management is
a. Heparin bolus plus infusion
b. Heparin infusion plus stocking
c. Heparin infusion plus warfarin
d. LMWH plus stocking
e. LMWH plus warfarin

Answer - D
43. 38-year-old known patient with rheumatic heart disease presented with pain and swelling of
right calf. On examination pulses were absent beyond the femoral level. What is the most
appropriate definitive management?
a. Heparin bolus followed by infusion
b. Iliofemoral bypass surgery
c. Trans femoral embolectomy
d. Digital subtraction angiogram and SK.
e. Arterial duplex scan
Answer – C  Embolectomy is done for acute lower extremity ischemia, (Churchill)

2015 Nov
11. Regarding occlusive arterial disease of lower limb
A. ABPI of 1.0 is abnormal
B. Critically ischemic limb cannot be saved
C. Rest pain is best managed with anticoagulants
D. Short segment occlusion is best treated with angioplasty
E. Distal vascular disease is contraindicated for bypass surgery

Answer - F F F T F
ABPI NORMAL RANGE 1- 1.3, critical limb ischemia can be saved with surgical intervention within
time frame, rest pain best managed surgically, short segment best treated with angioplasty, not
distal vascular disease total vascular occlusion is contraindicated for surgery

33. 28-year-old female has Sapheno femoral junction and long saphenous vein reflux. She has
pigmentations in gaiter area and varicose veins. What is the best management option?
A. Sclerotherapy to Sapheno femoral junction
B. Sapheno femoral ligation & striping & avulsion of long saphenous vein
C. Slerotherapy to varicose veins
D. Stab avulsion
E. exercise and compression stocking life time
Answer - B

2015 May
15. Regarding pulmonary embolism
A. Needs lifelong anticoagulation
B. D-dimers are diagnostic
C. Treated with LMWH in acute stage
D. Risk is eliminated by using DVT prophylaxis during surgery
E. Thrombolytic is the mainstay of treatment

Answers - F F T T F
A - Anticoagulation with warfarin 3 to 6 months.
B - D’ dimers are not diagnostic
E - Most pulmonary emboli can be Tx by anticoagulation and observation, but severe right heart
strain and SOB indicates the need for thrombolysis / radiologically guided catheter embolectomy.

35. 65 year old teacher developed arterial type intermittent claudication involving both calf on 500m
walking. He can do all of his daily activities. He was on statin & aspirin. What is the most appropriate
additional management option?
A. Exercise
B. Angioplasty
C. By-pass surgery
Answer – A

2014 Nov
46) 56 years old Elderly male, smoker, presented with intermittent claudication without any
evidence of tissue loss. He does not have DM, HT and dyslipidemia What is the best management
option?
a) stop smoking, antiplatelet, Vasodilators, Statins
b) stop smoking, antiplatelet, Vasodilators, Stenting
c) stop smoking, walking exercise, antiplatelet therapy, statin therapy
d) stop smoking, antiplatelet, Vasodilators,
e) Walking Exercise, Antiplatelet, Vasodilators and statins
Answer - C - explained before. He doesn’t need vasodilators for now as he doesn’t have rest pain,
but must stop smoking

2014 May
19) Regarding varicose veins
a) Occurs secondary to DVT
b) Venogram is a first line investigation
c) Lipodermatosclerosis is irreversible
d) Superficial thrombophlebitis is a complication
e) Anticoagulation reduces symptom severity
Answer - T F T T F
Venogram an invasive investigation,1st line would be a simple USS investigation, sclerosed changes,
atrophic changes are usually irreversible

47) 60 year old patient presented with chronic lymphedema. He has pain, erythema, swelling &
fever. Most appropriate management of this patient
a) Elevation of the limb & benzyl penicillin
b) Elevation of the limb & benzathene penicillin
c) Elevation & cloxacillin
d) Stocking & benzyl penicillin
e) Stocking & benzathene penicillin

Answer - A Straight forward cellulitis management.


Transplant Surgery
Allograft – tissue / organ transplanted from one person to another
Alloantigen – the transplant antigen
HLA – main trigger of graft rejection

powerfull
Allograft immune Graft rejection
response

By immunosuppressive treatment

 ABO compatibility and HLA are the most important factors


 ABO compatibility is a must. But no need for Rh compatibility.
 HLA incompatibility is the most common cause of graft rejection.

Graft rejection can be:


 Hyperacute - occurs immediately
 Acute – within first 6 months
 Chronic – months to years

1. Hyperacute rejection
- Occurs within minutes to hours
- Occurs due to ABO incompatibility or preformed anti HLA antibodies (due to previous blood
transfusions, previous failed transplant, pregnancy)
- Once the graft is revascularized antibodies immediately bind to the vasculature and activate
complement system leading to extensive intravascular thrombosis, interstitial hemorrhage
and graft destruction within minutes to hours.
- Kidney transplant is mostly vulnerable
- Can be avoided by group compatibility and crossmatching.

2. Acute rejection
- T lymphocyte mediated
- Alloantigens also may contribute
- Occurs due to mononuclear cell infiltration of the graft (by T cells, B cells, NK cells and
activated macrophages)
- All types of allografts are susceptible for this but most can be reversed by adequate
immunosuppressive treatment. May need plasmapheresis or immunoadsorption.

3. Chronic rejection
- Occurs after 6 months.
- All types of grafts are susceptible.
- It’s a major cause of graft rejection.
In renal transplant risk factors for chronic rejection are,
- Past history of acute rejection (most important factor)
- Poor HLA matching
- Long cold ischemic time
- CMV infection
- High blood lipids
- Inadequate immunosuppression

2019 Nov
42) A patient underwent a kidney transplant. What is the commonest cause for graft rejection from
the following?
a) ABO mismatch of donor & recipient
b) Age of donor
c) HLA mismatch of donor & recipient
d) ….
e) The time at the 1st dose of immune suppression was given
Answer – C - LA incompatible transplants are the main cause for graft rejection in general

2019 March
34) 58-year-old female underwent renal transplant 7 weeks back presented with polyuria polydypsia
abdominal pain. Her serum corrected calcium level was 2.8 mmol/l (2.2-2.6 mmol/L) phosphate 0.69
mmol/l (1.12 to 1.45 mmol/L). Most probable diagnosis.
a) Multiple myeloma
b) Paget's disease
c) Primary hyperparathyroidism
d) Secondary hyperparathyroidism
e) Tertiary hyperparathyroidism

Answer - E
- Patient underwent a renal transplant and coming with high calcium & low phosphate levels.
- CKD patients has secondary hyperparathyroidism leading to low or normal calcium and high
phosphate.
- And mainstay of treatment for that is renal transplant.
- The response to surgical treatment is dramatic and levels become normal almost immediately
and appear to be sustained for upto 3 years post op.
- But here the patient got high calcium levels and low phosphate. This is due to autonomous
activity of the parathyroid gland. This is development of tertiary hyperparathyroidism.
- However according to bailey and love 27th edition page 835 this should occur at least 1 year
post renal transplant.
50) What is the most likely malignancy that can occur in a patient who underwent Kidney Transplant
due to End Stage Renal Failure
A. Hetaptocellure CA
B. Hodgekin Lymphoma
C. Kaposi's sarcoma
D. Nasopharingeal CA
E. Squamous cell CA
Answer – E
- The two commonest ones are skin malignancy such as squamous cell CA and post-transplant
lymphoproliferative disorders such as lymphoma.
- The commonest one is skin malignancy mainly squamous cell CA.
- Kaposi sarcoma risk is high but still very uncommon to see. Therefore, squamous cell CA is
more likely.

2018 Nov
50. A 40-year-old female who has undergone a successful renal transplant 6 months ago is being
followed up and was found to have rising Creatinine levels in the recent visit. What is the most
possible cause?
a) Chronic Glomerular nephritis of the new kidney
b) Graft rejection
c) Immunosuppressive drug induced renal impairment
d) Technical error in vascular anastomosis
e) Renal cell carcinoma in the transplanted kidney
Answer – B
this should be chronic graft rejection. (graft rejection manifest as functional failure)
Technical error in vascular anastomosis will present much earlies.
Immunosuppression is lifelong. Drug induced side effects may occur early (first couple of months
after transplant) or several years later.
2017 May
40. A transplanted kidney turns dusky and congested shortly after revascularization despite having a
technically sound vascular anastomosis. As a result, a graft nephrectomy is performed.
What is the most likely cause for the above situation?
a) Acute cellular rejection
b) Chronic rejection
c) Graft versus host disease
d) Hyperacute rejection
e) Renal artery thrombosis
Answer is obviously D. This is hyperacute graft rejection.
- Occurs due to ABO incompatibility or preformed anti HLA antibodies (due to previous blood
transfusions, previous failed transplant, pregnancy)
- Once the graft is revascularized antibodies immediately bind to the vasculature and activate
complement system leading to extensive intravascular thrombosis, interstitial hemorrhage
and graft destruction within minutes to hours.

2021 March
50. During renal transplantation the transplanted kidney became grey and congested just after the
vascular anastomosis even though technically sound vascular anastomosis is appropriate. Therefore,
graft nephrectomy was done. What is the most likely cause?
A. Acute antibody mediated reaction
B. Acute cell mediated reaction
C. Acute tubular necrosis
D. Graft vs host reaction
E. Renal vein thrombosis

Answer – A
This question is almost same to 2017 May. Here they are asking about the reason for hyperacute
graft rejection.
It occurs either due to ABO incompatibility or presence of preformed antibodies against HLA class I.
(in simple terms the recipients blood has already formed antibodies against HLA / ABO. So once the
graft is placed they will attack the graft antigents)

Graft vs host disease –


Some donor organs contain large number of lymphocytes (liver, small bowel) and they may react
with recipient tissues.
Frequently cause a characteristic skin rash on palms and soles. And may also involve liver and GI
tract.
CATEGORIZED MCQ - UPPER GI
2020 June
03) Which of the following is/are features of acute perforation of peptic ulcer.
A. Absence of liver dullness
B. Generalized abdominal tenderness
C. fluid in the peritoneal cavity
D. Hematemesis
E. Palpable abdominal mass
Answer – T T T F F

Common complications of the peptic ulcer disease are perforation, bleeding and stenosis.
A – Clinical features of the acute perforations: Sudden onset, severe, generalized abdominal pain
and tenderness, tachycardia, abdominal rigidity. Also the abdomen does not move with respiration
and there will be absent liver dullness.
B – There is generalized abdominal tenderness due to irritation of the peritoneum by the gastric
contents.
C – In perforation the gastric content can leak and inflammation of the peritoneum also can cause
leakage of fluid into peritoneal cavity.
D – Usually perforation occurs in chronic peptic ulcers. Due to chronic nature of these ulcers blood
vessels at the ulcer are fibrosed and severe bleeding do not occur.
E – Usually there is no palpable mass.

25) 26 year previously well school teacher presented with dyspeptic symptoms for 2 weeks
duration.she has been transferrd to a urban school recently. what is the most probable managment
of this patient?
A.
B.
C.
D.Treatment with PPI and arrange urgent endoscopy
E.Treatment with PPI, dietary modification and re assess in 4 - 6 weeks

Answer – E (This is the likely answer. But can’t tell exactly because some answers are missing)
Initial management of dyspeptic symptoms is with lifestyle modifications such as weight loss,
smoking cessation, avoidance of stress, large meals at night etc. But these lifestyle modifications are
not always effective and they are rarely followed.
PPI is the most effective drug treatment for for GORD and most of the patients experience relief of
symptoms within days. Therefore, for this patient treatment with PPI should be started. But if the
patient has red flag symptoms such as LOA, LOW, Maleena, and Dysphagia or if the medical
therapy fails patient should be investigated with UGIE.
This patient has been transferred to an urban school recently and this plays an important role in this
patient as it can cause mental stress due to change of the workplace, loss of meals due to busy
schedule etc. So in the management that also should be taken into consideration.
26) 65 yr old patient with copd admitted due to dyspepsia for 1yr. Endoscopy reveals Barrets
oesophagus with high grade dysplasia. What is the best management option
A. Lower spinchtorectomy
B. Endoscopic resection of the barrets
C. Give PPI and follow up in 6 weeks
D. Radio frequency ablation
E. Radiotherapy
Answer – D
High grade dysplasia in Barrets oesophagus is the critical step before invasive oesophageal
adenocarcinoma.
Oesophagectomy represents the only treatment able to reliably eradicate the neoplastic epithelium.
But for that fitness for surgery should be there and in this patient it is not a good option because this
patient has COPD. In this patient endoscopic methods can be used and mainly two endoscopic
therapeutic methods are there.
 Endoscopic ablation using thermal or radiofrequency energy
 Endoscopic mucosal resection
The use of radiofrequency for the ablation of dysplastic epithelium is more effective, posing less
riskfor damage beyond the desired limits. Also it carries less risk of oesophageal strictures. (from
Medscape)

2019 November
3) Peptic ulcer disease
a) Occurs more commonly in antrum than body of the stomach
b) Unlikely to recur
c) May lead to microcytic anaemia
d) Erode into prepyloric vein leading to massive upper GI bleeding
e) Occurs in Meckel’s diverticulum
Answer – T F T ? T
a - chronic gastric ulcers are much more common in the lesser curvature of the stomach (specially
in the incisura angularis) than in the greater curvature of the stomach.
b - Peptic ulcer disease is likely to recur especially with the persistence of the risk factors.
c - All peptic ulcers can bleed and the bleeding can become chronic and patient can present with
microcytic anemia.
d - Commonest site of the bleeding is duodenum and the most commonly involved artery in
bleeding is gastroduodenal artery.
e - Meckels diverticulum has ectopic gastric epithelium and it can cause peptic ulcer disease.

26.) A 65-year-old male presented with LOA, LOW and anaemia for 2 years. He has been treated for
TB 10 years back. UGIE revealed a peptic ulcer with everted edges, central necrosis, and contact
bleeding. What is the most likely diagnosis?
a) Benign peptic ulcer
b) Gastric cancer
c) Gastro-intestinal stromal tumours
d) Primary gastric lymphoma
e) Tuberculous gastritis
Answer – e
This patient has been having red flag symptoms such as LOA, LOW and anaemia for 2 years. This long
duration suggests that this is unlikely to be a malignancy because if the patient has been having a CA
for 2 years the condition must be more severe with mets. This can’t be a benign ulcer as well
because the ulcer has everted edges and central necrosis which are not features of benign ulcers.
Also they do not cause symptoms such as LOA, LOW.
So this is more likely to be TB gastritis because this patient has been treated for TB 10 years back.
Gastro Intestinal TB can affect GI tract from mouth to anus. This commonly occurs secondary to
pulmonary TB and most commonly affected areas are ileum and proximal colon. Most patients with
GI TB get ulcers. (From a research article)

2019 MARCH
22) A 70year old man complains of progressive dysphagia for 8 weeks. Upper GI Endoscopy revealed
malignant looking lesion at 30cm. What is the most appropriate investigation to assess local invasion
a) Barium swallow
b) CXR
c) Endoscopic USS
d) MRI
e) PET scan
Answer – C
In staging of the oesophageal CA one of the most important aspects is assessing the oesophageal
wall penetration. CECT does not give a very clear picture of the depth of the wall penetration and
LN involvement. Therefore to assess the wall penetration and local LN involvement, endoscopic US
is the preferred investigation.

2018 NOVEMBER
1. Which of the following are caused by Helicobacter pylori,
a) Antral gastritis
b) Gastric carcinoma
c) Duodenal Ulcer
d) Gastric lymphoma
e) Reflux oesophagitis
Answer – T T T T F
a, b, c - Common manifestations of H. pylori infection are chronic gastritis, Peptic ulcer
disease(gastric ulcers and duodenal ulcers) and gastric carcinoma.
d - H. pylori has been identified as a cause for gastric lymphoma. Early gastric lymphomas may
regress and disappear when the H. pylori infection is treated.
e - Reflux oesophagitis is caused by defect in lower oesophageal sphincter. Even if a patient is
having H. pylori infection if the lower oesophageal sphincter function is intact reflux oesophagitis will
not occur.

22. A 30 year old male presents with burning epigastric pain for 2 weeks duration. He does not
complain of loss of appetite or weight. What is the most appropriate management option?
a) Perform an upper Gastro Intestinal Endoscopy.
b) Empirical treatment with proton pump inhibitors.
c) Start Helicobactor Pylori Eradication regimen.
d) Reassure and send home.
e) Advise on anti reflux life style modifications.
Answer – B
History is more suggestive of GORD. In GORD initial management is with lifestyle modifications such
as weight loss, smoking cessation, avoidance of large meals at night. But these lifestyle
modifications are not always effective and they are rarely followed.
PPI is the most effective drug treatment for for GORD and most of the patients experience relief of
symptoms within days. Therefore for this patient treatment with PPI should be started. But if the
patient has red flag symptoms such as LOA, LOW, Maleena, and Dysphagia or if the medical
therapy fails patient should be investigated with UGIE.

23. 32 year old patient presented with persistent vomiting and epigastric abdominal pain for 1 day
duration. On examination, there was abdominal tenderness and guarding. Postero-anterior chest
Xray shows gas bubbles above the liver shadow. What is the most possible cause?
a) Chronic pancreatitis
b) Acute pancreatitis
c) Perforated peptic ulcer
d) Reflux oesophagitis
e) Peptic ulcer disease
Answer – C
In perforated peptic ulcer typically they get a generalized, acute, severe abdominal pain. But
sometimes they can get less severe symptoms with epigastric pain. Due to irritation of peritoneum
they get guarding and rigidity of the abdomen. The most important feature here is gas under the
diaphragm which implies the perforation.

2017 May
1. Barrett esophagus is
a) A columnar dysplasia
b) A precursor for squamous cell carcinoma
c) Associated with achalasia cardia
d) Confirmed by 24-hour PH monitoring
e) Often asymptomatic
Answer – F F F F T
a - Barrett’s oesophagus is intestinal metaplasia (not dysplasia) of the epithelium of the oesophagus
due to chronic GORD.
b - When there is an intestinal metaplasia there is an increased risk of adenocarcinoma. Not
squamous cell carcinoma.
c - Barrett’s oesophagus is due to chronic irritation of the oesophagus due to reflux of the gastric
contents. In achalasia there is absent relaxation of the lower oesophageal sphincter. So the lower
oesophageal sphincter is tight resulting in inability to reflux. This avoids reflux.
d - Barrets oesophagus is a histological diagnosis. Therefore should take a biopsy for the diagnosis.
GORD is diagnosed with 24hr PH monitoring.
e - Most people are asymptomatic. Barrets oesophagus in the community is 10 times the incidence
discovered by UGIE.

21. A 55 years old man presents with sudden onset severe upper abdominal pain and vomiting.
Examination reveals a tender rigid abdomen. He has cirrhosis and a history of peptic ulcer disease.
Which of the following is the most appropriate next step in his management?
a) Diagnostic peritoneal lavage
b) Erect abdominal X-ray
c) Erect chest X-ray
d) USS of the abdomen
e) Upper GI endoscope
Answer – C
With the given history most likely causes are spontaneous bacterial peritonitis and perforated peptic
ulcer. SBP is usually associated with fever and not sudden onset. So the most likely diagnosis is
perforated peptic ulcer.
Clinical features of perforated peptic ulcer; Sudden onset, severe, generalized abdominal pain and
tenderness, tachycardia, abdominal rigidity and abdomen does not move with respiration. Also there
will be absent liver dullness due to perforation.
Presence of air under the diaphragm in erect chest x ray can be used as the first line investigation
finding to arrive at a diagnosis.

47. A 45 year old female is investigated for dysphagia of 2 year duration. Upper GI endoscopy shows
no lesion what is the most appropriate next Ix?
a) 24 hour pH monitoring
b) Barium swallow
c) Chest X ray PA
d) CECT
e) Endoscopic ultrasonography
Answer – B
History is suggestive of achalasia cardia. Usually middle aged females present with achalasia cardia.
Also in a 45 years old lady it is unlikely to be a carcinoma. Also UGIE is normal which means there is
no structural lesion.
Achalasia can be diagnosed with Barium swallow which shows birds beak appearance. Gold standard
is oesophageal manometry in which the tone of the oesophageal sphincter is measured.
2016 NOVEMBER
1. Which of the following are used to stage esophageal cancer?
a. CECT abdomen and thorax
b. endoscopic ultrasound
c. MRI of abdomen and thorax
d. PET CT
e. upper GI endoscopy
Answer – T T T T F
For the staging of oesophageal CA TNM staging is used. So local invasion, LN involvement and distant
metastasis should be assessed.
a - Most commonly used and haematogenous spread to other organs can be assessed. Usually in our
set up once the diagnosis is made with UGIE and biopsy the staging is done with CECT neck,
abdomen and pelvis.
b - CT does not give a very clear picture of the depth of the wall penetration and LN involvement.
Therefore to assess the wall penetration and local LN involvement, endoscopic US is the preferred
investigation.
c - Can be used to assess distant metastasis. Advantage over CT is this has no exposure to radiation.
But accuracy wise there is no advantage over CT.
d - Basis of the PET scan is increased metabolic activity of tumor cells. A radiopharmaceutical agent
is injected and they get accumulated in the cells with high metabolic activity and those tissues can
be identified by imaging. Even though not routinely done PET also can be used to stage.
e - UGIE and biopsy is mainly used to diagnose oesophageal CA. Cannot get an accurate idea about
local invasion, therefore with UGIE ‘T’ staging also not done.

Other methods of staging


• Bronchoscopy - can visualize the involvement of the trachea and bronchi. Done only if the
history or UGIE findings suggestive of tracheal or bronchial invasion.
• Laparoscopy - can detect intra-abdominal and liver mets. Advantage is can get biopsies from
the metastatic deposits.

21. 69 year old male comes with progressive dyspnea and has undergone Barium swallow due to
difficulties at the upper GI endoscopy. Which of the following finding would likely to indicate
carcinoma of the oesophagus?
a. Narrowing of the oesophagus with bird beak appearance
b. Dilation of the upper oesophagus
c. Persistent filling defect in oesophagus
d. Air-fluid levels in upper oesophagus
e. Aspirated contrast in trachea-oesophageal tract
Answer – C
Presence of filling defect indicates a mechanical obstruction. Oesophageal carcinoma is a mechanical
obstruction and it can be suggested with the presence of a filling defect.
Bird beak appearance – Feature of achalasia
Dilatation of upper oesophagus – In functional obstructions such as achalasia also can give rise to
dilatation of the upper oesophagus.
Aspirated contrast in trachea oesophageal tract – We use water soluble contrast medium instead of
Barium if the history is suggestive of a trachea oesophageal fistula. ( Buddhika sir)

50. 54 year old chronic smoker & betel chewing person presented with dysphagia and LOA. Most
probable diagnosis?
a. CA in cervical oesophagus
b. CA in thoracic oesophagus
c. CA in antrum
d. CA in pylorus
e. CA in body of the stomach
Risk factors are more suggestive of squamous carcinoma of the oesophagus. But LOA is more
suggestive of a gastric carcinoma.
In a squamous carcinoma of the oesophagus if it has liver mets it can give rise to above picture. But
then carcinoma of the cervical oesophagus and thoracic oesophagus both becomes equally possible.
If LOA is due to primary tumor this is more likely to be due to gastric carcinoma, but for gastric CA to
become dysphagia it should be at gastro oesophageal junction or at fundus causing external
compression. ( Buddhika sir)

ALL THE ANSWERS ARE TAKEN FROM THE BAILY AND LOVE EXCEPT A FEW, IN WHICH I HAVE
MENTIONED.
Lower GI surgery
2020 June
04) regarding presentations of colorectal carcinoma
A. colonic ca causes faecal incontinence.
B. Colonic obstruction is common in left side
C.
D. sigmoid carcinoma present with PR bleeding
E. carcinoma in transverse colon present as emergency

Rectal CA Left Colon Right colon


Per Rectal bleeding (blood altered bowel habits RIF mass
mixed with stools) (Constipation for a few days melena
Tenesmus followed by a watery anaemia
urgency, frequency diarrhoea) No altered bowel habits
sense of incomplete emptying altered blood or blood mixed Caecal tumours can cause LOA
spurious diarrhoea with stools
pneumaturia, recurrent UTI Colonic obstruction- tumours
and haematuria due to bladder grow in to the lumen, Stool is
fistulation, solid and harder
faecal incontinence due to
sphincter involvement
loin pain due to ureteric
involvement
neuropathic pain in the
perineum and ED due to sacral
nerve involvement

Table: Clinical Presentations of colorectal CA

Faecal incontinence- due to sphincter involvement (Direct invasion or sacral nerve involvement) or
true fistula
Rectal CA can cause Faecal incontinence, but colonic CA are rare to cause incontinence
A-True?
B- True
C-
D- True
E- True (CA in transverse colon can present as Intestinal obstruction, massive rectal bleeding)

05) Regarding surgery on colon and rectum


A. Anastomotic leak common in RHC than AR
B. end colostomy is irreversible
C. Hartmann procedure is the safest surgery for perforated sigmoid colon
D. ileal pouch improves the quality of life after a total proctocolectomy
E. total mesorectal excision reduces the recurrence after anterior abdominal resection

A- F? (Small bowel to large bowel anastomoses heal quicker than large bowel to large bowel
anastomoses and anastomotic leaks are less common in prior one. So, in large bowel to large bowel
anastomoses, it is usual practice to de-function the site with a stoma to reduce risk of anastomotic
leakage)
B- false (not always irreversible; Hartmann’s procedure)
C- true (The risk of anastomotic failure is high, as there is contamination of the peritoneal cavity by
the ruptured sigmoid colon. In those cases, Hartmann procedure is the safest)
D- True (Restorative proctocolectomy with an ileoanal pouch avoids a permanent stoma. Pouch is
made out of ileum as a substitute for the rectum and sewn or stapled to the anal canal.)
E- true (most cases can be treated by rectal resection followed by total meso rectal resection; all LN
are in the mesorectum.)

28) 58 year old women developed painless abdominal distention found to have absent bowel
sounds with on post op day 2 following anterior resection for CA at rectosigmoid junction. She is
haemodynamically stable. Erect chest X-ray showed free gas under diaphragm. She was on
Diclofenac Na for pain management. What could be the most probable cause for the above clinical
presentation?
A. Anastomotic leakage
B. Bowel ischemia
C. Internal bowel herniation
D. Paralytic ileus
E. Perforated peptic ulcer

Answer- D
Painless abdominal distension and absent bowel sounds suggests paralytic ileus which can occur in
48-72 h post op. She is pain free and haemodynamically stable, so can’t be perforated peptic ulcer
although she is on diclofenac. Free gas can be there as her abdomen was open for the surgery.

29) Best diagnostic test to diagnose local invasion of rectal carcinoma?


A. CT
B. diagnostic laparotomy
C. EUS
D. MRI
E. PET

Answer - C
MRI is best for staging of disease, as it gives details of local invasion and nodal involvement. But the
best test to diagnose local invasion is endoscopic Ultra sound.

2019 Nov
1) A woman underwent anterior resection,
a) If develops fever on Day 1, it could be due anastomoses leakage
b) Hypokalaemia can lead to paralytic ileus
c) If develops fever on Day 1, it could be due to lung atelectasis
d) Kept nil orally until flatus is passed
e) Risk of DVT is similar to that of right hemicolectomy done for caecal tumour
A-False
B- True
C-True
D- False
E- True (All abdominal surgeries are of equal risk, malignancies increase risk DVT)

4) 42-year-old male undergo right hemicolectomy for proliferative growth in ascending colon.
Histopathology reveals moderately differential adenocarcinoma of the colon with infiltration to
mesentery. Seven out of nine lymph node show malignant deposit. Which of the following are true
regarding further management of this patient?
a) Adjuvant chemotherapy
b) Adjuvant radiotherapy
c) Colonoscopy follow up
d) Expectant management
e) Family screening

Adjuvant chemotherapy- for both colonic and rectal (node positive disease)
Radiotherapy- not given for colonic CA due to risk of damage to small bowel
Neoadjuvant chemoradiotherapy- given in rectal CA, no benefit in colonic CA

Follow up of colorectal CA – Colonic CA are known for metachronous lesions


History and physical examination in every 3 months for 2 years and then in every 6 months
for total of 5 years
CEA in every 3 months for 2 years and then in every 6 months for total of 5 years
Annual CECT of chest, abdomen and pelvis for 5 years
Colonoscopy in 1, 3 and 5 years

A- True
B- False
C- True (Follow up colonoscopy in 1,3 and 5 years)
D- False
E- True (Positive Family hx is a known risk factor, family screening is done)

5) 45 years old woman presented with 2 weeks of increased frequency of bowel movement,
abdominal pain, PR bleeding and she’s a diagnosed patient with ulcerative colitis. This was an
exacerbation True regarding this
a) Can be managed as outpatient
b) He needs urgent laparotomy
c) He needs immediate colonoscopy
d) Responds to steroid therapy
e) Sulfasalazine effective as the initial treatment

we classify ulcerative colitis into mild, moderate, severe and fulminant cases.
Mild - Stool frequency< 4/ day, Hb normal, ESR normal, No systemic signs
Can be Mx with topical Tx alone. Rectal steroid suppositories can be used.
No need to use systemic steroids. 5-ASA topical is also possible.
Usually if beyond the rectum involved might need to use enemas of steroids or 5- ASA.
Moderate - 4-6 stools/day, Hb may be low, ESR slightly elevated, Some systemic features present.

Severe - >6/day, other features may be markedly present

Fulminant - >10/day, persistent bleeding needing transfusion, Toxic megacolon (Rare complication of
UC. Transverse colon dilates massively and there is impending perforation. Should be Mx with
immediate colectomy.

There are no systemic signs mentioned, so this can be considered as a mild exacerbation. (Details are
not sufficient)
A- True
B- False
C- False (colonoscopy in severe acute exacerbation, increases risk of perforation and bleeding.
Colonoscopy plays an important role in diagnosis and management, but as he is already a
diagnosed patient, he does not need immediate colonoscopy)
D- True
E- True (It is said that sulfasalazine is effective in remission induction and maintenance.)

21) 50 yr old patient is awaiting Colonic Surgery. Which of the following is most important to prevent
surgical site infections?
a) Pre-Op carbohydrate loading
b) Mechanical bowel preparation
c) Preoperative nutrient supplementation
d) Prophylactic antibiotic use
e) Shaving the abdomen

Answer- D
some clean and all clean contaminated surgeries will need antibiotic prophylaxis. Contaminated and
dirty wounds will have high load of bacteria, so prophylaxis will not be effective, therapeutic
antibiotics will have to be given.

27) A 55-year-old women presented with sudden onset bilious vomiting and abdominal distension.
She has had a previous laparotomy for bowel perforation 5 years ago. She also has a history of
dyspepsia and gall stones. What is the most likely cause for her current presentation? a) Adhesions
b) Carcinoma of Colon
c) Gallstone ileus
d) Inguinal hernia
e) Paralytic ileus

Bilious vomiting suggests intestinal obstruction beyond opening of bile duct. This is likely small bowel
obstruction, in large bowel vomiting comes later. The most common cause for bowel obstruction is
adhesions. Though she has a hx of gall stones, the most likely cause is adhesions. And the most
common site of gall stone getting impacted in chole-enteric fistula is ileo colic junction, and it takes
time to develop vomiting.
Answer-A

28) 64-year-old women who had a history of altered bowel habits was admitted to surgical unit with
lower abdominal pain, fever, vomiting. She had a similar episode 1 year ago. On examination, she
was tachycardic and found to have a mass in the left iliac fossa. What is the most likely diagnosis of
this patient?
a) Acute pyelonephritis
b) Diverticular mass
c) Fulminant ulcerative colitis
d) Left tubo-ovarian mass
e) Locally advanced carcinoma of sigmoid colon

Hx of altered bowel habits states the condition is recurrent. Mass in the left iliac fossa should be toxic
mega colon.
Answer- C

29) A 25-year-old man complains of painless fresh bleeding per rectum. Digital rectal examination
is normal. What is the most likely diagnosis?
a) Anal carcinoma
b) Anal fissure
c) Anal polyp
d) Haemorrhoids
e) Rectal carcinoma

Answer- D
Painless fresh bleeding with normal DRE suggests haemorrhoids (unless thrombosed)

44) 80-year-old lady presented with abdominal distension and constipation for 4 days. DRE showed
empty rectum. Which of the following prompts urgent surgical intervention?
a) Absent rectal gas
b) Dilated caecum more than 12 cm
c) Dilated small bowel loops
d) Dilated sigmoid colon
e) Dilated stomach

Answer- B

Colicky abdominal pain and empty rectum suggests intestinal obstruction.


caecal diameter of 12cm is suggestive of imminent perforation and an indication for early intervention.

2018 Nov
2. Regarding familial adenomatous polyposis
a) 50% of affected individuals have a positive family history.
b) Diagnosed when there are more than 100 polyps in the colon during colonoscopy.
c) Total proctocolectomy should be done in diagnosed patients
d) Screening done from 25 years of age.
e) Common to have extraintestinal manifestations.

Familial adenomatous polyposis-


▪ Autosomal dominant inherited condition. >80% have a family Hx.
▪ APC gene mutation is responsible.
▪ >100 polyps need to be present. To diagnose
▪ Polyps visible by 15y of age and Invariably (100%) becomes malignant by 30y to 40 y of age.
▪ Associated with - desmoid tumours, osteomas, retinal pigment hypertrophy, epidermoid cysts
(Gardner’s syndrome), pilomatrixomas, medulloblastomas (Turcot syndrome), Dental abnormalities.
▪ Can develop gastric, duodenal, thyroid CAs also.
▪ Need prophylactic pan proctocolectomy with end ileostomy.
▪ Commonest cause for death after this is ampullary CA.

a) False
b) True
c) True
d) False
e) True

3. Small intestinal obstruction,


a) Associated with elevated serum amylase.
b) Continuous abdominal pain indicates strangulation.
c) Commonly caused by post op adhesions.
d) Passage of stools exclude complete obstruction
e) Abdominal distension is an early feature

a) False
b) True
c) True
d) False
e) True

Intestinal obstruction Causes


✓ In a patient who has previously undergone abdominal surgery adhesions should always be
suspected as the most likely cause.
✓ Even in the general population the commonest cause for small bowel obstruction is adhesions.
✓ Hernial orifices should always be checked as it could be an obstructed hernia.
✓ In elderly patients with a preceding history of altered bowel habits colorectal CA should always be
excluded.
✓ Other important causes include volvulus, Strictures due to Crohn disease or diverticular disease.
✓ In Paediatrics malrotation or intussusception are always possibilities.
✓ Foreign bodies and gall stone ileus are also possibilities.

Presentation
✓ Small bowel obstruction presents with colicky periumbilical pain, vomiting and abdominal
distension. Vomiting will result in dehydration and acid base imbalances. Constipation is a late sign.
✓ In colonic obstruction colicky suprapubic pain, abdominal distension and constipation occurs. In
colonic obstruction abdominal distension is a late sign compared to distal small bowel obstruction.
(Bailey And Love) Vomiting is a late feature. On Ex bowel sounds will be increased. If strangulation
and gangrene bowel sounds will reduce.

If there are features to suggest strangulation, gangrene or perforation such as guarding, rigidity and
rebound tenderness or localized tenderness and persistent tachycardia suggestive of strangulation
emergency laparotomy is needed.

The development of severe pain is suggestive of the presence of strangulation, especially if that severe
pain is continuous. Beware the patient whose pain is not controlled with intravenous opiates.

Serum amylase may rise when no free abdominal fluid is present. Furthermore, when abdominal fluid
does form because intestinal obstruction, its amylase activity is never greater than that of the serum
drawn simultaneously so long as intestinal continuity is maintained.
Constipation
This may be classified as absolute (i.e., neither faeces nor flatus is passed) or relative (where only flatus
is passed). Absolute constipation is a cardinal feature of complete intestinal obstruction. Some
patients may pass flatus or faeces after the onset of obstruction as a result of the evacuation of the
distal bowel contents. The administration of enemas should be avoided in cases of suspected
obstruction. This merely stimulates evacuation of bowel contents distal to the obstruction and
confuses the clinical picture.

4. Regarding haemorrhoids,
a) Haemorrhoidectomy is the treatment for Grade 1 haemorrhoids
b) Straining while defecation is predisposing factor.
c) Thrombosed haemorrhoids can be managed conservatively.
d) Is a premalignant lesion.
e) Those are prolapsed anal cushions.

a) F
b) T
c) F
d) F
e) T

Haemorrhoids-

✓ Painless, Fresh blood not mixed with stools, falling as drops towards the end of defecation or as a
fresh splash in the pan.
✓ lump at anus (not a must. Only grade 3 or 4 haemorrhoids will be obvious)
✓ A preceding history of constipation may be present. Can occur at any age.
✓ No mass felt during DRE. Remember haemorrhoids are best visualized and not felt. ✓ Proctoscopy
is needed in most cases.
✓ Prolapse, thrombosis, infection and bleeding are complications.
✓ Commonest risk factor is constipation. Any condition that increases straining in the toilet such as a
rectal CA may also give rise to this.
✓ Portal hypertension giving rise to dilated rectal varices are not true haemorrhoids.
✓ True haemorrhoids are dilated anal cushions which appear at 3, 7 and 11 o clock positions. These
are internal haemorrhoids.
✓ Obesity, pregnancy, Uterine and ovarian neoplasms, neurological disorders with low anal sphincter
tone such as MS are 2ry causes for haemorrhoids.

Management of haemorrhoids
Grade 1
Bleed only. No prolapse. Can be managed conservatively. Adequate fluid, high fibre diet, lactulose
can be used. If poor response sclerotherapy with 5% phenol in arachis or almond oil.

Grade 2
Prolapsed but spontaneously reduce. Can initially try conservative measures like sclerotherapy. If
fails rubber band ligation and cryotherapy can also be used.

Grade 3
Prolapsed and need manual reduction. Need haemorrhoidectomy

Grade 4
Prolapsed and cannot reduce manually. Needs Haemorrhoidectomy

Grade 2 haemorrhoids which have not responded to other measures, fibrosed haemorrhoids,
hemorrhoidal bleeding severe enough to cause anaemia also need haemorrhoidectomy.

Mx of complicated haemorrhoids
Strangulation, thrombosis and gangrene- Currently immediate surgery under antibiotic coverage is
considered possible. Alternatively, bed rest, analgesia and cold compressors result in reduction of
haemorrhoids in many.

25. A 65 year old female patient presented with Per rectal bleeding. A lower rectal carcinoma which
is 6cm away from the anal verge, was detected during flexible sigmoidoscopy. Histology reveals
poorly differentiated rectal Carcinoma MRI shows locally advanced cancer. What is the most
appropriate next step in the management?
a) Neoadjuvant chemotherapy
b) Anterior resection with total mesorectal excision.
c) Abdomino-Perineal resection.
d) Neoadjuvant chemoradiotherapy.
e) Hartman's Procedure and palliative care.

Answer-d
In locally advanced tumours, neoadjuvant chemoradiotherapy should be done to shrink the tumour.

Rectal tumour Mx–


✓ T1, T2NOMO Can be mx by surgery.
✓ During surgery the rectum, the mesorectum and lymph nodes removed.
✓ There are 2 main surgeries available.
1. Anterior resection - For rectal tumours 2cm proximal and 2cm distal resection margin is
enough. To do this the lower border of the tumour should be >2cm away from the anal verge.
2. Abdomino perineal resection- When the sphincter cannot be preserved. Needs a
permanent end colostomy. For tumours >T2 and N1 and beyond tumours Neoadjuvant
chemoradiotherapy given before surgery

✓ Other options- Trans anal total mesorectal excision- Less invasive than other surgeries. Same
outcome in the long run
✓ Local trans anal excision - For Tis and T1a tumours.
✓ Even if liver metastasis are present liver resection plus APR or AR is done. It improves prognosis.
✓ 5 yr survival is about 40%. Even removal of multiple liver mets improves long term survival. Even
liver transplantation is an option.
✓ Even removal of single pulmonary metastasis has a benefit.
✓ Multiple pulmonary mets, bone or brain mets cannot be resected.

2017 May
2. Causes of massive fresh bleeding per-rectum include,
a) Angiodysplasia of the colon
b) Diverticular disease of the colon
c) Mallory Weiss syndrome
d) Meckel diverticulum
e) Radiation proctitis

a) T
b) T
c) F
d) T
e) T?

Massive fresh PR bleeding - Also known as haematochezia.


✓ More commonly caused by UGI pathology. Peptic ulcer, ruptured varices, gastric CA, aortoenteric
fistula are Ex.
✓ Diverticular disease, UC, Colo-rectal CA, Angiodysplasia, rarely haemorrhoids, ischaemic colitis are
some causes. Meckel’s diverticulum can also cause.

Angiodysplasia-
✓ Occurs in elderly. Uncommon.
✓ May be associated with Aortic stenosis.
✓ Vascular malformation that can cause recurrent bleeding.
✓ Difficult to detect on colonoscopy. May need RBC scan.
✓ Mostly occurs on the right side.

Diverticular disease-
-diverticular disease commonly affects the elderly and commonly involves the sigmoid colon.
- With diverticulitis the patient can develop fever and abdominal pain.
- Constipation is a risk factor for diverticular disease.
- That might be the reason there is a past history of altered bowel habits. - With diverticulitis there
may be localized abscess formation.
- On the long run diverticular disease can result in strictures, adhesions, perforations and fistula
formation.
- It can present with PR bleeding and can sometimes give rise to massive bleeding. - It never affects
the rectum.
- In the Asian population it commonly affects the right side and the caecum. - Gold standard for
diagnosis is the colonoscopy.
- In the acute setting if a colonoscopy is done it may result in perforation. Usually done 6 weeks later.
- So, in the acute setting with inflammation CECT is the best.
- Acute diverticulitis is managed with I.V. fluids, analgesics, antibiotics etc
- Chronic disease is managed with high fibre diet, fluids and laxatives or stool softeners.
- Diverticular disease will not increase malignancy risk.

Severe bleeding can occur with chronic radiation proctitis. Untreated radiation proctitis can cause
severe bleeding, anaemia, ulcers, fistulas.

Mallory Weiss syndrome refers to a tear or laceration of the mucous membrane, most commonly at
the point is gastroesophageal junction. This may result in severe bleeding from the GI tract.

3. Regarding small intestinal obstruction


a) Bowel opening excludes complete obstruction
b) Elevated serum amylase excludes obstruction
c) Persistent severe pain indicates strangulation
d) The commonest cause is adhesions
e) The level of obstruction is assessed by a supine abdominal X-ray

a) F
b) F
c) T
d) T
e) F

Erect abdominal films are no longer routinely obtained and the radiological diagnosis is based on a
supine abdominal film. An erect film may subsequently be requested when further doubt exists.
intestinal obstruction, fluid levels appear later than gas shadows as it takes time for gas and fluid to
separate. These are most prominent on an erect film.

4. Extra intestinal manifestations of inflammatory bowel disease include,


a) Ankylosing spondylitis
b) Erythema multiforme
c) Lage joint arthritis
d) Sclerosing cholangitis
e) Uveitis

a) T
b) F
c) T
d) T
e) T

Uveitis, scleritis, episcleritis, skin rashes such as erythema nodosum and pyoderma gangrenosum,
Joint disease such as spondylitis, sacroiliitis, large joint arthritis can occur

Oral ulcers, 1ry sclerosing cholangitis can occur in ulcerative colitis.

22. An 82 year old male with a history of atrial fibrillation presents with sudden onset abdominal
pain and rectal bleeding. On examination his blood pressure is 100/60mmHg, pulse rate is 100bpm
and respiratory rate is 24bpm.his white blood cell count is 23000/m3.
What is the most likely diagnosis?
a) Angiodysplasia of colon
b) Carcinoma of the colon
c) Diverticulitis
d) Ischemic colitis
e) Ulcerative colitis

Answer- d

Ischemic colitis (has also been questioned with regard to PR bleeding in the past.)
✓ It usually occurs in elderly with valvular heart disease or atrial fibrillation due to embolism.
✓ They present with sudden onset abdominal pain, bloody diarrhoea and vomiting.
✓ Inflammatory markers may be elevated and neutrophil leucocytosis can occur.

45.A 28 year old previously healthy female presents with a painful lump at the anus of 8 hours
duration that had appeared acutely on straining. On Examination, a tender subcutaneous lump is
seen at the anal verge.
What is the most likely diagnosis?
a) Acute anal fissure
b) Fistula-in-ano
c) Perianal abscess
d) Perianal hematoma
e) Second degree haemorrhoid

Answer-d

Thrombosed external haemorrhoid Mx - Also called a perianal hematoma.

● Presents as a sudden onset, olive shaped, painful blue subcutaneous swelling and occurs upon
straining at stool, coughing.

●If patient presents within 48hrs clot may be evacuated under local anaesthesia. Most untreated
cases resolve within 5 days.

2016 Nov
2. 30 year old man presents with altered bowel habits associated with blood and mucus. Which of
the following features favour a diagnosis of Crohn’s disease over Ulcerative colitis?
a. Episodes of intestinal obstruction
b. Polyarthralgia
c. Recurrent abdominal pain
d. Anal fissures
e. Recurrent UTI

Ulcerative Colitis Crohn Disease


UC- M: F= 1:1 More in Females
Usually occurs in the age group of 20-40 Bimodal distribution. Peak around 60-70y as
well. Also 15-30
Smoking is protective Smoking precipitates
Rectum is always involved. Ascends to a Usually, rectum is spared. Can involve any
variable length from there. place from mouth to anus. Ileocecal disease is
commonest. Perianal disease is 2nd commonest.
Results in continuous lesions Results in skip lesions.

Non granulomatous inflammation Granulomatous


Inflammation occurs in the mucosa and Transmural inflammation
submucosa
Since no transmural inflammation does not cause strictures, fistula, adhesions and
cause strictures, fistula, adhesions and obstruction
obstruction
Pseudo polyps can occur. Crypt abscesses No pseudo polyps. Cobblestone appearance.
occur. Aphthous ulcers seen.
Usually presents with blood and mucous More commonly presents with recurrent
diarrhoea, lower abdominal pain, tenesmus, chronic watery diarrhoea, malnutrition, fever
urgency, frequency and fever LOW etc. If colon involved can have a bloody
diarrhoea
Malabsorption is uncommon Malabsorption common. Ileal disease results in
Vit B 12 malabsorption, bile acid malabsorption
which in turn leads to fat soluble vitamin
malabsorption.
Does not cause perianal ulcers, fistulas and Can cause all these.
fissures
Does not cause a RIF mass Can cause RIF mass
Oral ulcers, 1ry sclerosing cholangitis can occur Usually not seen
Does not cause recurrent renal calculi. Can cause.
Uveitis, scleritis, episcleritis, skin rashes such as The same can occur
erythema nodosum and pyoderma
gangrenosum, Joint disease such as spondylitis,
sacroiliitis, large joint arthritis can occur
Risk of malignancy is more. Long standing Risk of malignancy less but still increased
disease (Usually risk increases about 8yrs after
the onset), extensive colonic involvement,
presence of 1ry sclerosing cholangitis increase
the malignancy risk.
Proctoscopy alone may be diagnostic. Usually, Ix will depend on the site of the disease. If
we do a colonoscopy to look for the extent of small bowel needs small bowel enteroscopy. If
the disease and detect dysplastic changes ileocecal colonoscopy can be used.
P- ANCA may be positive. ASCAS may be positive as a biochemical marker
Usually, no mesenteric lymphadenopathy Mesenteric lymphadenopathy may be present
as the inflammation is transmural.

A. True (transmural inflammation can cause strictures)


B. False (can occur in both)
C. False
D. True (peri anal dx is commoner in CD)
E. True (bladder fistula can cause recurrent UTI)

3. pre-malignant lesion of colon are,


a. Crohn’s disease.
b. Diverticular disease
c. Ulcerative colitis.
d. Peutz Jegher syndrome.
e. Serrated adenoma

Benign polyps of the colon


Juvenile polyps, peutz jeghers polyps and pseudo polyps are benign ones.
Juvenile polyps never become malignant.
Peutz jeghers can rarely become malignant.
Peutz jeghers Syndrome is autosomal dominant and presents with painless PR bleeding and rarely
intussusception.
Colorectal CA occurs through adenoma carcinoma sequence. Villous adenomas, Sessile serrated
adenoma, multiple polyps and polyps >2cm in size have a high risk of malignant transformation.

A. True
B. False
C. True
D. False
E. True

4. Acute anal fissure


a. best diagnosed by proctoscopy
b. lateral internal sphincterotomy is performed to treat intractable fissures
c. local application nitro glycerine gel is used in treatment
d. majority recurs following conservative management
e. mostly located at 6 0’ clock position of the anal canal

Anal fissure
 Blood-streaked stools, intense pain during defecation which may persist for a variable time after
defecation, may have pruritus which lasts for a while after defecation (mainly in chronic fissure
with a sentinel skin tag).
 Usually occurs at 6 o clock position in the anus. In females due to child birth fissures may occur
anteriorly.
 DRE or proctoscopy is not done.
 Causes for anal fissures at atypical sites - Crohn disease, TB, sexually transmitted infections such as
HIV, Chlamydia, Gonorrhea, Anal carcinoma

Management of anal fissures


 Treat underlying constipation with adequate fluid intake, high fiber diet, laxatives like lactulose.
(1st line treatment) - All acute fissures and most chronic ones will resolve.
 If response is poor 2nd line - Topical GTN
 Alternatives- Topical diltiazem, Botulinum toxin injections to relax the sphincter
 Last resort- Lateral anal sphincterotomy
Can end up with hemorrhage, hematoma, perianal abscess and fistula.
Most dreaded complication is anal incontinence due to sphincter damage.
In women and those with normal resting anal pressures - anal advancement flap can be used
instead.

A. False
B. True
C. True
D. False
E. True

23. 50-year-old man with no co morbidities presented with right sided obstructive lesion confirmed
to be recto-sigmoid adenocarcinoma. CT scan showed a focal lesion in the left lobe of the liver. CT
chest is normal. Most appropriate management?
a. Defunctioning colostomy and palliative chemotherapy
b. Hartmann’s procedure and palliative chemotherapy
c. Hartmann’s procedure and liver resection
d. Stenting and neoadjuvant chemotherapy
e. Neoadjuvant chemotherapy and anterior resection

Answer- E
Rectal Ca has good prognosis even with a single liver met. AR plus liver resection can be done following
neoadjuvant chemoradiotherapy.
25. 47-year-old woman was presented with anal pain and spotting of blood on defecation. On DRE
an ulcer was detected. Other physical examination was normal. Biopsy showed squamous cell CA.
What is the most appropriate curative management?
a. Abdomino-Perineal Resection
b. Anterior Resection
c. Chemo-radiotherapy
d. Primary radiation therapy
e. Trans-anal excision

Anal carcinoma
 Causes painful fresh PR bleeding.
 If it is invading the sphincter, they may have fecal incontinence.
 In women can develop ano-vaginal fistula.
 Risk factors may be present such as HPV infection (16 and 18) and male to male sex etc.
 If DRE done painful ulcer which is irregular and indurated may be felt.
 Usually, they are squamous cell CA.
 Lymphatic spread is to the inguinal nodes.
 Local staging is best done by MRI.

Management of Anal CA
Chemoradiotherapy is the 1st line management. Surgery is done only if poor response to
chemoradiotherapy.

Answer- D

26. A 42 years old man is coming with alternative bowel habits. Patient has undergone a lower GI
endoscopy and revealed malignant growth of 4 cm × 5 cm size. Before the surgery decided to give
neoadjuvant chemotherapy. What is the main objective of that?
a. Increase survival.
b. Reduce the local recurrence.
c. Reduce liver metastasis.
d. Reduce metachronous tumour

Question does not clearly say whether it’s a colonic or rectal ca.
Neoadjuvant chemo radio therapy for rectal CAs reduce the chance of local recurrence, make
inoperable tumours operable by downstaging but does not improve the survival.
In colonic tumours there is no benefit in neoadjuvant chemoradiotherapy unlike rectal tumors.
Adjuvant post op chemotherapy can be used in both colonic and rectal CAs. Usually used for node
positive disease. They improve survival.

Answer - B
Corrections in categorized surgery MCQs
Lower GIT
2020 June
29) Best diagnostic test to diagnose local invasion of rectal carcinoma?
A. CT
B. diagnostic laparotomy
C. EUS
D. MRI
E. PET
Answer – C change to answer D
Buddika sir explanation -
 EUS is the most sensitive to see exactly which layer of the bowel wall the tumour has gone
up to (i.e. The T stage)
 MRI is more useful in telling us what is happening beyond the rectal wall such as nodal
spread in the mesorectum.
 checked with Dr Janaka Ekanayake who says "local infiltration" means extending OUT of the
rectum.
 Therefore, MRI is the correct answer. (Answer D)

2017 May
3. Regarding small intestinal obstruction
a) Bowel opening excludes complete obstruction
b) Elevated serum amylase excludes obstruction
c) Persistent severe pain indicates strangulation
d) The commonest cause is adhesions
e) The level of obstruction is assessed by a supine abdominal X-ray

Consider E as True (not false) – we usually do supine AP xray to diagnose.


So the answers are F F T T T

2016 November
25. 47-year-old woman was presented with anal pain and spotting of blood on defecation. On DRE
an ulcer was detected. Other physical examination was normal. Biopsy showed squamous cell CA.
What is the most appropriate curative management?
a. Abdomino-Perineal Resection
b. Anterior Resection
c. Chemo-radiotherapy
d. Primary radiation therapy
e. Trans-anal excision
Consider C as the answer – Chemoradiotherapy
THE PANCREAS
Pancreatic carcinoma
Risk Factors
 Demographic factors
- Age (peak 35-75 years)
- Male gender
- Black ethnicity
 Environment/ lifestyle
 Cigarette smoking
 Genetic factors and medical conditions
- Family history - Peutz-jeghers syndrome
- Hereditary pancreatitis - Familial breast ovarian cancer syndrome
- Chronic pancreatitis - Familial atypical multiple mole melanoma
- Lynch syndrome - Familial adenomatous polyposis
- Ataxia telangiectasia - Diabetes mellitus

>85% - ductal adenocarcinoma


 most commonly found in head of the gland
 infiltrate locally, typically along the nerve sheaths, along lymphatics and into the blood vessels
 liver and peritoneal metastasis is common

clinical features
 obstruction of distal bile duct jaundice
 painless jaundice associated with nausea and epigastric discomfort
 pruritus, dark urine and pale stool with steatorrhea
 if no jaundice – symptoms are non specific (anorexia, weight loss)
 occasionally – unexplained attack of pancreatitis
 tumours of body and tail of the gland often grow silently, and present at an advanced
unresectable stage
 palpable liver and palpable gall bladder
 other signs of intra-abdominal malignancy
(ascites, supraclavicular lymph nodes)
 thrombophlebitis migrans

Investigations
 Hb  LFTs
 FBC  Blood sugar
 ESR  CA 19-9 (elevated in 75%)
 USS – (small mass obscured by bowel gas)
 CT scan – degree of invasion, metastases, guided biopsy, assess the resectability
 EUS
 PET scan- metastases
 MRCP
 ERCP – frequently reserved for biliary decompression

Management
Carcinoma of the head of pancreas – pancreaticoduodenectomy (Kausch-Whipple procedure)
Involves partial gastrectomy, partial pancreatectomy, distal choledochectomy and
cholecystectomy
Continuity is re-established via Roux-en-Y choledochojejunostomy, pancreaticojejunostomy
and gastroenterostomy
For extensive tumour – total pancreatectomy
Tumours in tail – distal pancreatectomy
Palliative decompression and relief of jaundice
If inoperable – biliary stenting by ERCP
Prognosis
5 year survival rate following curative surgery – 15-20 %
Overall 5 year survival – poor 5%

2020 june
08. T/ F regarding pancreatic carcinoma
A. gastric outflow is obstructed by pancreatic head carcinomas - T
Pancreatic cancer is the most common malignancy causing GOO. Outlet obstruction may
occur in 10-20% of patients with pancreatic cancer (Medscape)
B. coeliac lymph nodes are involved – T
final lymph drainage of pancreas occurs into coeliac, superior mesenteric, para-aortic
nodes
C. Most common are ductal adenocarcinoma - T
D. Smoking is a risk factor - T
E. Pancreatic body tumours can identify earlier than head tumour – F

2016 Nov
6. Regarding carcinoma of the pancreas
A. Carcinoma in the body pancreas presents later than the head of pancreas cancer - T
B. Endoscopic US used in staging - T
The pancreas and its surrounding vasculature and lymph nodes can be assessed
Useful to identify small tumours and in demonstrating the relationship of pancreatic
tumour to major vessels.
EUS accuracy for local staging of pancreatic cancer ranges from 70 -90 %, superior or
equivalent to other imaging modalities.
C. Ductal adenocarcinoma is more common in the head of pancreas than the body of
pancreas - T
D. Histological diagnosis is mandatory prior to surgical resection. - F
E. Pancreatic body tumours treated with Whipple’s procedure. -F
For tumors of body and tail distal pancreatectomy with splenectomy is standard. In
Whipple procedure pancreatic head is removed.
2015 Nov
03. Pancreatic carcinoma
A. Chronic pancreatitis is a risk factor - T
B. Adeno carcinoma is the commonest type - T
C. Head of the pancreas is the commonest site - T
D. Survival rate is 50 % - F

Chronic Pancreatitis
Progressive inflammatory disease with irreversible destruction of pancreatic tissue.
Male:Female – 4:1
Mean age of onset – 40 years
 Aetiology
High alcohol consumption
Pancreatic cancer
Strictures after trauma pancreatic duct obstruction
After acute pancreatitis
Congenital abnormalities
Pancreas divisum
Annular pancreas associated with papillary stenosis
Hereditary pancreatitis Hyperlipidemia
Cystic fibrosis Idiopathic pancreatitis
Infantile malnutrition Autoimmune pancreatitis
 Clinical features
Pain
Disease mainly in the head of pancreas – epigastric and right subcostal
Disease in the left side the pancreas – left back pain and back pain
Nausea
Vomiting
All the complications of acute pancreatitis can occur with chronic pancreatitis
The patient’s lifestyle is affected due to pain.
Loss of exocrine function – steatorrhea
Loss of endocrine function – diabetes mellitus

 Investigations
serum amylase – elevated only in the early stages of the disease
later atrophy of the pancreas lead to normal amylase levels
abdominal xray – pancreatic calcifications
CT – atrophic pancreas, calcification, dilated pancreatic duct, pseudocysts
USS – cyst change and duct dilatation
MRCP – identification of biliary obstruction and the state of pancreatic duct
ERCP - most accurate way elucidating the anatomy of the duct.
EUS
 Management
Management of pain
Changing the diet -high CHO and low fat and protein diet
Simple analgesics and anti inflammatory drugs
PCM, Diclofenac sodium, tramadol
Surgery
inflammation NO inflammation
Duct dilataion Frey’s procedure pancreaticojejunostomy

No duct dilatation Whipple’s procedure Coeliac plexus block


Or Beger’s procedure Thoracoscopic splanchicectomy

Inflammation of the tail of pancreas – distal pancreatectomy

2019 March
6) A 45 years old man presented within epigastric pain after meals for 6 months duration. Imaging
reveals pancreatic calcification and there is no duct dilatation. Which of the following management
which relive his pain.
A. Celiac plexus block. - T
B. Distal pancreatectomy – F
Rare patient with disease limited to the tail can be benefitted by this procedure
C. Pancreatic enzyme supplement. – T
Pancreatic enzyme supplementation may be helpful in reducing the pain. The hypothesis
is that stimulation of pancreas by food causes pain. Cholecystokinin is one of the possible
mediators of this response. When exogenous pancreatic enzymes are taken with a meal,
CCK releasing factors are degraded and CCK release in response to meal is reduced. This
decreases pancreatic stimulation and pain.
D. Oral tramadol. – T
E. Endoscopic pancreatocography and main duct dilation. – F
Decompressing an obstructed pancreatic duct can provide pain relief in some patients
assuming the duct hypertension causes the pain. In this patient there is no duct dilatation.

2019 Nov
8) Regarding chronic pancreatitis
A. Gallstones is the commonnest causative factor – F
High alcohol consumption is the most common cause
B. Hypoglycemia is a clinical feature – F
Blood glucose may be raised
C. Intraductal calcification is noted in plain abdominal x-ray - T
Pancreatic calcification can be seen in x ray ; but intraductal ?
Medscape – pancreatic calcifications are often considered pathognomonic and observed
in 30% cases. Calcifications form within the ductal system.
D. splanchnic artery pseudoaneurysm is a complication -T
Pseudoaneurysms are rare but deadly. Affected vessels – splenic, hepatic, gastroduodenal
and pancreaticoduodenal
E. Serum amylase can be normal - T

Pancreatic pseudocyst
 Collection of amylase rich fluid enclosed in a well-defined wall of fibrous or granulation tissue.
 Typically arise after an attack of mild acute pancreatitis lie outside the pancreas
 Formation requires 4 weeks or more from the onset of acute pancreatitis
 Often single, occasionally can develop multiple
 Has a non-epithelialized wall.
 CF- tender epigastric mass, fever, weight loss, nausea, vomiting
 Identified on USS or CT
 Will resolve spontaneously in most instances
 Therapeutic interventions are advised only if the pseudocyst causing symptoms, if
complications develop, or if a distinction has to be made between tumor and a pseudocyst
 Three possible approaches to drain the cyst : percutaneous, endoscopic and surgical
 Percutaneous transgastric cystgastrostomy – for infected pseudocysts
 Endoscopic drainage transmural and transpapillary
 Surgery - cystgastrostomy

2014 May
34) What is the best management of pancreatic pseudocyst.
A. Conservative management
B. Surgical excision
C. Endoscopic cystogastrostomy
D. Open cystogastrostomy
E. External drainage
Most will resolve spontaneously. Therefore, best management will be conservative
Answer – A

2013 Nov - Colombo


5) Pancreatic pseudo cyst
A. Lined by pseudo stratified columnar epithelium – F
Its non-epithelialized
B. in lesser sac - T
one third of cysts appear in the head and two third appear in the tail
Usually found extending to the lesser sac
C. Occurs due to blunt trauma – T
Acute or, rarely, chronic pancreatitis or abdominal trauma causes pseudocysts.
D. Occurs within 1 week of acute pancreatitis – F
Requires 4 or more weeks
E. Resolve majority without surgery - T
Hepatobiliary Disorders
2020 June
07)Regarding functions of the liver
A. alkaline phosphate secrete by liver parenchymal cells
B. ammonia metabolize in to urea
C. gamma carboxylation of factor 2 ,7,9,10 can impaired due to vitamin D deficiency
D. lactate metabolism is deranged in acute liver failure
E. AST and ALT levels are high in liver parenchymal ischemia

Answers - F T F T T
 ALP is found in hepatic canaliculi and sinusoidal membranes. Addition to that in Bone,
Intestines, Placenta.
 Amino acids converted into ammonia which is is coverted to Urea and exctreted via kidneys.
2,7,9, 10 are Vit K dependent: A fat soluble vitamin absorbed with the aid of bile acids
produced by liver.
 Liver removes 70% of lactate. Patient can go into Metabolic acidosis
 AST – primarily a mitochondrial enzyme.
 ALT- cytosolic enzyme
 Both increase in hepatocellular damage.

22) 51 yr old female undergone an uncomplicated laparoscopic cholecystectomy. After 14 hours she
developed fever (38.2°C), mild pain and nausea. O/E normal except mild tender abdomen. What is
the likely cause for fever?
A. Bile leakage
B. Metabolic response to trauma
C. Pulmonary atelectasis
D. urinary tract infection.
E. wound infection

Answer - C
Post op fever –
D1 to 3 – Wind – Atelectasis and pneumonia
D3 to 5 – Water – UTI
D4 to 8 – Walking – DVT and PE
D5 to 7 – Wound – SSI
Drug fever – anytime

 Since it’s 14 hrs after cholecystectomy patient can have all these features due to Metabolic
response to trauma.
 But in this case patient has undergone a lap cholecystectomy. This pulmonary atelectasis is
also likely.
27) 40-year-old known patient with hypertension presented with severe progressive right
hypochondrial pain for two days’ duration with progressive yellowish discolouration of eyes and
high grade fever with chills and rigors. He had similar two episodes during past few years. What is
the diagnosis?
A. Acute cholecystitis
B. Gallstones
C. Acute cholangitis
D. Biliary colic
E. Obstructive jaundice

Answer – C
Painful Jaundice
- CBD stones
- Cholangitis
- Ca head of pancreas

Charcot’s triad
- Upper abdominal pain
- Jaundice
- Fever with chills and rigors

Reynolds Pentad
- Charcot’s triad + Hypotension + Altered Mental Status

32)60 years old man presented with fever & abdominal pain. On examination there was right
hypochondrial tenderness. He is haemodynamically stable. Ultrasound scan shows empyema of gall
bladder. What is the most appropriate management to improve his condition?
A. Broad spectrum antibiotics
B. Emergency laparoscopic cholecystectomy
C. Emergency open cholecystectomy
D. Right hemi thyroidectomy
E. Total thyroidectomy

Answer- A
Mx of Empyema of GB
1. Antibiotics – Cefotaxime, Quinolones, Ceftriaxone
2. Cholecystectomy- drainage
3. Cholecystectomy - later

43)A man following a laparoscopic cholecystectomy complains upper abdominal pain on day 2.
Generalized tenderness present on palpation. What is the 1st investigation you do?
A. CECT abdomen
B. ERCP
C. Erect chest X ray
D. MRCP
E. USS of Abdomen

Complications Lap cholecystectomy


1. Access complications
2. Bile duct injuries. ( Can present with bile peritonitis if a drainage has not been provided)
Bile duct injuries present with fever, pain, Fx of bile stasis like cholangitis, jaundice
Considering this Answer – E

2019 Nov
7) Regarding neoplasia of liver,
a) Adenoma occurs due to long term OCP use
b) Cirrhosis is a risk factor for liver cell carcinoma
c) Metastasis of colorectal cancer can be cured completely by resection
d) Haemangioma is the commonest
e) Liver transplantation can be curative for HCC

Haemangiomas
 Most common liver lesions
 Consists of abnormal plexus of vessels
 Ix – USS, Ct scan with delayed contrast (only if diagnostic uncertainty exists)
 Percutaneous biopsies are avoided as they may bleed

Hepatic adenoma
 Rare
 Ix- CT, MRI
 Bx maybe necessary for confirmation and characterization of the nature of the lesion
 These tumors may bleed and has a malignant potential
 Tx- resection
 Associated with sex hormones (OCP) – regression of symptomatic adenomas on withdrawal of
hormone stimulation

Focal Nodular Hyperplasia


 Unknown aetiology
 Pts are middle aged females usually
 Ix- USS (does not help in discrimination), Contrast CTor/and MRI
 No malignant potential. Thus treatment is not required.

Hepatocellular carcinoma
 Associate with CLCD (Hep B,C)
 CLCD pts are screened for HCC by doing USS, AFP
 Can present in middle age with CLCD symptoms (malaise, weakness, jaundice, ascites, variceal
bleeding, encephalopathy) or anorexia,
LOW. (advanced cancer)
 Ix – USS, CECT abdomen (more reliable and ideal), Tumour markers, Ct angiography (essential
when planning hepatic resection, LFT
 Tx – resection of liver tumour, Liver transplantation

Liver metastases
 Solitary, multiple unilobar and multiple bilobar liver mets are all considered for resection.
 Percutaneous bx of resectable lesions should be avoided as there is good evidence that malignant
cells can seed along the bx track.
 Prognostic factors in pts undergoing resection of colorectal liver mets

Stage of primary
 Time of primary resection
 CEA level
 Size of largest lesion
 Number of lesions
Answer – T T ? T T
“C” says “completely”, so not sure

32) A 50-year-old women presents with obstructive jaundice, colicky right upper quadrant pain and
fever for 3 days. She had a laparoscopic cholecystectomy for symptomatic gall stones 2 weeks ago.
What is the most likely diagnosis?
a) Benign biliary stricture
b) Bile duct injury
c) Gall stone pancreatitis
d) Local bile collection
e) Retained common bile duct stone

Answer – E
Jaundice in post op period  indicates a bile duct obstruction by a gall stone or a surgical failure.
- Cholangitis
- Obstructing stone
- Bile duct strictures (develop after several years)

2019 March
5) Regarding gallstones disease
a) pigmented stones are due to parasites
b) higher in nulliparous than in multiparous
c) common among patient with cirrhosis
d) ileal resection is a predisposing factor
e) majority are asymptomatic

Answers - T F T T T
3 main types
- Cholesterol
- Pigment
- Mixed

Cholesterol or mixed  80% Europe and USA


- 51-99% pure cholesterol plus admixture of calcium salts, bile acids, bile pigments, phospholipids

Risk factors
- Obesity
- High caloric diets
- Medications (OCP)
- Terminal ileal resection

Pigment stones
- Has <30% cholesterol
- Two types  black and brown
- 20-305 are black  associated with haemolysis, hereditary spherocytosis or sickle cell disease.
- Pts with cirrhosis have a higher chance of pigmented stones.
- Brown pigment stones  calcium bicarbonate, Ca palmitate, Ca stearate + cholesterol. A/w
presence of foreign bodies with in bile ducts(endoprosthesis, parasites – chlonorchis sinensis, Ascaris
lumbricoides)
Most gallstones can be asymptomatic.
Gallstones are infrequent in nulliparous.

2018 Nov
6. 45 year old man presented with an obstructive jaundice and a right hypochondral mass. What
are the possibilities?
a) Pancreatic carcinoma
b) Gallbladder stones
c) Chronic pancreatitis
d) Common bile duct Stones
e) Periampullary CA

OJ can be caused by Pancreatic head Ca, CBD Stones, Stones impacted in the cystic duct or
infundibulum of GB. (Mirizzi syndrome), Chronic pancreatitis giving rise to an inflammatory mass,
and periampullary CA.
RHQ mass can be seen in,
All the above except CBD stones.( gall bladder stones in Mirzzis syndrome)

Courvoisiers’ Law – A palpable GB in the presence of jaundice is unlikely to be due to gallstones.


Answer – T (T?) T F T
26. A 38 year old female who is a previously incidentally diagnosed patient with Gallbladder stones,
presented with a persistent Right Hypochondrial pain which radiates to the back along the
abdominal wall, vomiting and fever; after 4 days from the onset of symptoms. On examination,
Murphy's sign was positive. FBC shows neutrophil leukocytosis. What is the most appropriate
management for this patient?
a) Endoscopy retrograde cholangio pancreatography
b) Symptomatic management
c) Emergency laparotomy

Mx
1. Antibiotics
2. Analgesics
3. Antipyretics
4. IV fluids
5. Interval cholecystectomy after 8 weeks Or Hot cholecystectomy with in 24-72 hrs.

Most appropriate Mx is cholecystectomy – Pt is having acute cholecystitis. ( Murphys signs positive +


Fever + Hx suggestive of biliary colic)
Since patient is Haemodynamically stable can go for Lap cholecystectomy.
Answer – D

2017 May
5. Regarding a patient with obstructive jaundice
a) ERCP is the first line investigation
b) Palpable gallbladder indicates malignant disease
c) The degree of pruritus is proportionate to the level of serum bilirubin
d) Trans-abdominal ultrasonography reliably demonstrates bile duct stones
e) Urine urobilinogen is elevated

OJ Mx
1s t line is USS Abdomen – Can look for duct dilatation (intrahepatic) and identify the level of
obstruction.
USS  suspect CBD stones  ERCP (diagnostic and therapeutic)

MRCP  To see biliary and pancreatic ducts better but not therapeutic

USS  suspects lesions close to the stomach wall, duodenum and pancreatic head  EUS (can do
needle bx)

USS  If suspecting mass lesion outside ducts  CECT

Cholangiocarcinoma – Best ix  MRCP, MRI, MRA


 If suspecting head of pancreas T – CECT
 More information on local invasion – MRI and MRA
 Small ampullary tumour – ERCP
 CT fails to show Tumour – EUS

Urine urobillinogen is absent in OJ


Degree of pruritus is not proportional to the Serum bilirubin level.

Answer - F T (not necessarily but can indicate) F F F

6. Regarding hepatocellular carcinoma


a) Liver biopsy is mandatory before resection
b) Liver transplantation is contraindicated
c) Majority occur in the background cirrhosis
d) Serum alpha-fetoprotein level is elevated
e) Tri-phasic CT abdomen is diagnostic
Answer - F F T T T

24. A patient with acute severe cholangitis is planned for an urgent ERCP an binary biliary stenting.
The INRis 1.8 and you are requested to correct the coagulopathy prior to the procedure. Which of
the following is the most appropriate agent to correct the INR before ERCP?
a) Activated factor VII
b) Cryoprecipitate
c) Fresh frozen plasma
d) Tranexamic acid
e) Vitamin K

Vit K – takes more time than FFP to act. Given IV as can’t be absorbed when given orally due to
biliary obstruction
Not given IM as haematomas can be formed.
FFP – Acts quickly

Answer – C

46.A 45 year old female presents with fever and severe right upper quadrant. She is febrile and has
a Pulse rate of 104/min.USS shows gallstones and pericholecystic fluid with non-dilated bile duct .
WBC is 14,600/m𝑚3 with a neutrophil leukocytosis, there is mild derangement of the liver function
. Tests and the serum amylase is 160u/L(24-80) What is the most likely diagnosis?
a) Acute cholangitis
b) Acute cholecystitis
c) Acute pancreatitis
d) Biliary colic
e) Pyogenic liver abscess

Pericholecystic fluid and clinical picture suggests acute cholecystitis


2016 Nov
5. Regarding acute cholecystitis,
a. Cholecystectomy not done in the first 48 hours
b. Jaundice is a characteristic feature.
c. May occur even in absence of gallstones
d. Pericolic fluid in ultrasound scan confirm the diagnosis.
e. ERCP done

Answer – F F T (acalculous cholecystitis) T F (Done in CBD stones)

28. 34-year-old woman presented with right upper quadrant pain and diagnosed to have gall
stones. On US scan there were multiple gall stones in the gall bladder. She was given a date for
interval cholecystectomy and discharged. 4 days later she presented with fever jaundice and
abdominal pain. What is the best next step of investigation?
a. ERCP
b. CECT abdomen
c. USS abdomen
d. Urgent laparoscopic cholecystectomy and on table cholangiogram
e. Urgent open cholecystectomy and on table cholangiogram

1st Ix when presenting with jaundice is USS Abdomen

Answer – C

29. 65-year-old male patient was found to be having a solitary mass lesion in the right lobe of the
liver. What is the most appropriate investigation to diagnose?
a. FNAC
b. CECT Liver
c. USS abdomen
d. Liver biopsy
e.Laparoscopy
Thinking of the possibility of a tumour best would be CECT Liver.

Answer - B
Surgical Emergencies
2019 Nov
25) 60 years old patient cirrhotic patient presented to the ETU with hematemesis.
He was conscious. PR 90bpm. BP 100/80. What is the best way of management?
a) Gain IV access, and start massive transfusion
b) Gain IV access, IV octreotide infusion, and emergency endoscopy
c) Gain IV access, and give normal saline bolus
d) Gain IV access, send blood for grouping and DT, and emergency endoscopy
e) Sengstaken tube

Answer – D The best management would be to do emergency endoscope.

UGI bleeding with hematemesis and melena


1. Start with A, B, C approach.
Any patient with a GCS <8 the next most important step in Mx is intubation and ventilation.
2. Connect to a multipara monitor. Give O2 via face mask if low saturation.
3. Assess PR, BP. Gain I.V. access by inserting two large bore cannula.
4. Collect blood for grouping and cross matching, INR, APTT, FBC, S.E., BU, S.Cr and LFTs.
5. Give I.V. fluids according to hemodynamic parameters.
6. Give I.V. PPIs. If past Hx of CLCD add I.V. Terlipressin/ Octreotide + I.V. Ceftriaxone.
7. Keep the patient NBM. Transfuse blood if refractory hypotension, grade 3 or 4 hemorrhage or if
Hb <8.
8. Do UGIE after stabilization. Injection of diluted adrenaline, glue injection can be tried if ulcer seen.
For ulcers endoscopic laser, argon plasma coagulation are also options.
If varices do banding, after banding oral beta blockers can be given to prevent re-bleeding.
Best option to prevent re-bleeding is recurrent band ligation.
9. Indicators of large bleed at UGIE-
Clot adherent to the base of the ulcer or a visible spurting blood vessel or a visible artery at the
base. Clinically if not responding to resuscitation and requiring massive transfusion they are taken
as indicators of massive bleeding.
10. If not responding put a Sengstaken Blackmore tube to tamponade bleeding varices.
11. Consider open surgery or endovascular TIPS as a last resort. TIPS is better if available

2019 March
18) Regarding antibiotic prophylaxis
A. Best given as a loading dose 2 hours prior to surgery
B. Broad-spectrum antibiotics is contraindicated
C. Given up to 3 doses
D. Has high risk of causing antibiotic resistance
E. No indication in clean surgery

Answers – F F T T F
Prophylactic antibiotics should be ideally given 30 minutes prior to incision except for few conditions.
Generally limited to single dose but can be given maximum 3 doses.
It should not be continued for more than 24 hours of the end of surgery.

Antibiotic prophylaxis is indicated in some clean and all clean-contaminated surgeries.


In contaminated surgeries either antibiotic prophylaxis or therapy is recommended depending on
patient’s clinical condition.
In dirty surgeries antibiotic therapy (not prophylaxis) is indicated.
Antibiotic prophylaxis is indicated in the following clean surgical procedures
 Surgery involving introduction of prosthetic material
 Surgery where consequences of infection would be catastrophic - E.g. neurosurgery, open
heart surgery, orthopedic surgery or ophthalmic surgery
 Surgery with impaired host defenses

23) A 60 yr old man with esophageal varices presents with haematemesis. He is unconscious. His BP
is 80/50mmHg & PR is 124 bpm. What is the most appropriate next step in the management?
A. IV Octreotide
B. Insertion of wide calibre NG
C. Insertion of Sengstaken Blakemore tube
D. Intubation with cuffed ETT
E. UGIE & banding of varices

Answer – D
Start with A, B, C approach.
As unconscious and has hematemesis the next most important step in Mx is intubation and ventilation.

43) A 40 year old known diabetic patient underwent a laparoscopic Cholecystectomy. She was
readimitted with fever. On examination PR 100, RR 16, BP 120/70, tenderness in right hypochondrium
and there was no jaundice. What is the Best management option?
a) MRCP
b) Urgent ERCP
c) Diagnostic laparotomy
d) USS abdomen
e) CECT abdomen

Answer – D
- Patient developed fever and RCH pain following a cholecystectomy. This presentation could
be due to acute cholecystitis or acute pancreatitis.
- The patient does not have high fever with chills and rigors. Thus, acute pancreatitis more
likely.
- First do USS abdomen to see gallstones / enlarged pancreas with peripancreatic fluid and
inflammatory changes.
- Optimum modality is CT and optimum timing is 48-72 hours.
(This question was different in different papers and the details given are contradictory. Therefore,
learn post cholecystectomy complications and ERCP complications)
2018 Nov
5. Regarding upper gastrointestinal bleeding
a) Can present as an emergency.
b) Peptic ulcer disease is a cause.
c) Oral medication to control bleeding is effective.
d) Endoscopic stenting is a treatment option.
e) Long term prophylaxis is indicated in all to prevent a recurrent more severe attack

Answers – T T F F F
B - UGI bleeding causes- oesophageal/ gastric/ duodenal ulcers or erosions, Mallory Weiss tear,
oesophageal varices, tumours, vascular lesions like Dieulafoy’s disease, aorto-enteric fistula
D - endoscopic treatment options- injection therapy (adrenaline), sclerotherapy, banding,
haemostatic clipping, argon plasma coagulation, laser therapy (stenting is done in oesophageal CA)
E – not required in all patients

17. Which of the following cause post-operative fever in Day 5 after elective bowel surgery,
a) Anastomotic leak
b) Atelectasis
c) Deep Vein Thrombosis
d) Reactionary/ SIRS
e) Surgical wound infection

Answers - T F ? F T
Post op fever
- Upto 48 hrs – The commonest cause is inflammatory response to tissue trauma. Usually the
fever is a mild fever < 100 Fahrenheit. It usually doesn’t persist beyond 48 hrs.
- Atelectasis can also cause fever from the day 2-5.
- Days 4 to 5- Superficial and deep wound infections
- Day 5- UTI – usually if a catheter is in place.
- Hypostatic pneumonia or infectious pneumonias - After laparotomies
- Cannula site infection - Look for swelling and tenderness
- > Day 5 - Wound infection, anastomotic leak, abscesses and anastomotic leakage and
peritonitis can occur. may have peritonitis features.

21. 4 year old boy is brought to the accident and emergency unit by the mother complaining of an
accidental ingestion of one rupee coin 5 hours ago. On examination, He is clinically stable and no
abnormality is detected. What is the most appropriate next step in the management?
a) Perform a barium swallow.
b) Perform a Bronchoscopy.
c) Reassure and discharge.
d) Perform an UGI endoscopy.
e) Perform X ray Neck, Chest and abdomen.

Answer – E
Swallowed or inhaled foreign bodies Coins are the most frequently swallowed foreign bodies in
children. Once they are beyond the cardia, they are usually passed in a few days. A plain radiograph
of the abdomen, chest and neck should establish the site of radio-opaque objects.

24. A 45 patient undergone an elective bowel surgery. After 24 hours, he developed abdominal pain
and vomiting. On examination, Bowel sounds were absent. Serum Na - 145mmol/l, Serum K -
2.8mmol/l. What is the most likely cause for this acute presentation?
a) Anastomotic leakage
b) Bowel Perforation
c) Thermal Bowel Injury
d) Paralytic ileus
e) Intraperitoneal bleed ing

Answer – D
Paralytic ileus- Functional obstruction
 Commonly occurs after bowel surgery due to excessive handling.
 Usually recovers within 72 hours.
 Presents with abdominal distension, absolute constipation and effortless vomiting. Usually
painless. May be caused by conditions like pancreatitis, peritonitis due to any cause,
hypokalemia or hypoalbuminemia.
 Bowel sounds will be absent. In obstruction due to other cause bowel sounds will be
increased. If strangulation or perforation occurs the bowel sounds will decrease and later be
absent.
 Paralytic ileus is managed by keeping the patient nil by mouth with NG decompression and
correcting any precipitating cause until complete recovery.

43. 16-year-old girl underwent open appendectomy. Inflamed non perforated appendix was removed.
During the surgery, a local non purulent exudate seen. How do you best classify this surgery?
a) Clean Surgery
b) Clean contaminated Surgery
c) Contaminated Surgery
d) Dirty Surgery
e) Infected

Answer – C ,Contaminated surgery


2017 May
39. A 70-year-old male develop fever of 38C, 8 hours following an anterior resection for rectal
adenocarcinoma. He is pain free with an epidural. His BP is 130/80 mm/Hg, pulse rate is 80/min and
respiratory rate is 16/min.
What is the most likely explanation for this fever?
a) Anastomotic leak
b) Deep vein thrombosis
c) Epidural abscess
d) Pneumonia
e) Systemic response to surgical trauma.

Answer – E, systemic response to trauma (already explained)

2016 Nov
13. Investigations in acute abdominal pain,
a. A supine abdominal x-ray to diagnose acute intestinal obstruction
b. Erect chest x-ray to see intra-peritoneal gas leakage
c. Contrast CT abdomen should be done in a patient suspected to have a ureteric colic.
d. Ultrasonography is the 1st line imaging modality in a patient with suspected rupture of aortic
aneurism.
e. X-ray KUB should be done immediately in a patient with a ureteric colic.

Answers – T T F F F
A – plain supine abdominal x-ray a useful tool in diagnosing bowel obstruction. However, remember
that a normal x-ray does not exclude an obstruction and if there is persistent concern, further
imaging is indicated.
B - Erect CXR is the ideal 1st test for hollow organ perforation and as little as 10-20ml of free air can
be detected under the diaphragm.
C – the best diagnostic investigation for ureteric stone is Non Contrast CT KUB
24. A 60 years old female presented with abdominal pain for 4 days. She is afebrile and having left
iliac fossa tenderness. CRP – 150, WBC - 20 000/mm3 with neutrophilia. What is the most
appropriate next investigation?
a. USS abdomen.
b. Barium enema.
c. Colonoscopy
d. Sigmoidoscopy

this presentation is suggestive of diverticulitis.


While some authors have promoted the use of focused ultrasoundfor this indication, in general it is
best diagnosed with a CT scan. The typical CT appearance is of pericolic inflammatory change around
a diverticulum, most commonly in the sigmoid colon. (Bailey and Love 27th , page 212)

2015 May
26. A man presented with massive haematemesis. Upper GI endoscopy revealed bleeding duodenal
ulcer. What is the most probable artery that gives rise to bleeding?
A. Gastroduodenal artery
B. Left gastric artery Right gastric artery
C. Right gastroepiploic artery
D. Superior pancreaticodoudenal artery

Answer - A
Most gastric ulcers occur in the lesser curvature of the stomach.
Anterior ulcers have a higher risk of perforation whereas posterior ones have a risk of
bleeding.
Sometimes large posterior ones may erode into the splenic artery.
Duodenal ulcers mostly erode into the gastroduodenal artery.

38. 45 year old man presented with episodic haematemesis. Endoscopy was done & identify
oesophageal varices. What is the best next step of management?
A. Ballon temponade
B. Band ligation
C. IV terlipressin
D. Oral propranolol
E. Sclerotherapy

Answer – B
2014 Nov
27) 60yr old man who underwent hernia repair, with a mesh is readmitted with fever. He is a diabetic
patient & on oral hypoglycemic drugs. On examination there was erythema around the surgical scar.
What is the most appropriate next step in management?
a) change to insulin
b) send blood culture
c) start IV antibiotics
d) remove sutures
e) remove mesh

Answer – B
 If it is just a localized skin Infection only with and erythema and tenderness without a
discharge or a mass, do wound swab and culture followed by Antibiotics.
 If there is a discharge there may be a pus collection and in these cases sutures should be
removed, pus should be drained out and sent for culture, wound should be thoroughly
cleaned and debrided and antibiotics should be started.
2019 March
9) Which of the following are true or false regarding recurrent laryngeal nerve palsy
following thyroidectomy
a) exertional dyspnoea
b) stridor
c) snoring
d) hoarseness of voice
e) trousseaus sign
Answers – F T F T F
Except cricothyroid muscle which make vocal cords to Para median position all the others are
supplied by RLN (cricothyroid is by superior laryngeal nerve)
A – Causes of exertional dyspnoea are pulmonary, cardiovascular, or neuromuscular disorders.
B – Turbulent flow of air at respiration through vocal cords causes stridor
C – Occur due to obstructed upper pharynx
E – Occur due to hypocalcaemia following parathyroid gland injury. It does not have to do anything
with RLN palsy.

38) 28y male, ear pain, vertigo and unilateral facial palsy. There are vesicles around the ear. what is
the diagnosis?
a) acoustic neuroma
b) bell’s palsy
c) Lyme disease
d) mucoepidermoid carcinoma of parotid gland
e) Ramsay hunt syndrome – secondary
Answer – E
This is a very straight forward question and the answer should be at the tip of your tongue.
Ramsay hunt syndrome – caused by Herpes zoster virus. Has LMN facial nerve palsy, pain and
vesicles on tympanic membrane, ear canal, pinna or inside the cheek. Also may have vertigo and
sensoneural hearing loss. (Bailey 27th edition page 715)

2018 Nov
38. 26 year old female patient presented with severe earache and purulent discharge from the ear
for 2 days duration. On examination Tenderness when pressing the tragus What is the most likely
diagnosis?
a) Acute Otitis Externa
b) Acute Otitis Media
c) Chronic Otitis Externa
d) Chronic Otitis Media
e) Acute labrynthitis
Answer – A
Only the outer ear involvement gives rise to tragal tenderness. Initial serous discharge in
acute otitis externa converts to purulent discharge in few days. (Dhingra 6th edition, page 51)
2017 May
36. A 44 year old male presents with a 2 month history of painless, purulent, foul smelling discharge
from the right ear. His hearing is getting worse. On examination the tympanic membrane is
retracted. What is the most likely diagnosis?
a) Acute otitis media
b) Cerumen (ear wax)
c) Cholesteatoma
d) Glue ear
e) Swimmer’s ear
Answer – C
Chronic painless ear discharge is characteristic in chronic otitis media. Retracted tympanic
membrane is due to cholesteatoma.
Cholesteatoma – it is presence of (skin in the wrong place) keratinized squamous epithelium in the
middle ear or mastoid. (Dhingra 6th edition, page 67)
Glue ear = otitis media with effusion = accumulation of non-purulent effusion in the middle ear
cavity. Often the fluid is thick and viscid but may be thin and serous. (Dhingra 6th edition, page 64)
Swimmers ear = Acute Otitis Externa

2016 Nov
35. A heavy smoker presents with hoarseness of voice for 4 weeks’ duration. On clinical examination,
no thyroid enlargement and no palpable cervical lymph nodes. What is the best investigation to
diagnose the condition?
a. Chest x-ray
b. Laryngoscopy
c. Bronchoscopy
d. CT of the chest
e. US of neck
Answer – B
Hoarseness of the voice is mainly due to vocal cord problems. Best investigated by direct
Laryngoscopy.

2015 Nov
17. Middle ear infections,
A. Vertigo is s feature of infection
B. Common in children
C. Majority are haematogenous
D. Pain is a prominent feature in chronic otitis media
E. Is a cause for meningitis
Answer – F T F F T
A – Vertigo occurs due to inner ear problems
B – Due to short Eustachian tube
C – Majority are through the spread from Eustachian tube. Also can be through TM perforation, if
there is ventilation tube or with haematogenous infections.
D – Pain is less prominent in chronic OM
E – A complication of OM

2014 Nov
20. Regarding AOM
A. Oral antibiotics are superior to local Ab
B.
C. Otitis media causes hearing loss
D. Grommet is used to treat otitis media
E. Suspected complications can assessed by CT head
Answer – T _ T T T
A – In AOM, local antibiotics doesn’t reach the infected area. Therefore systemic antibiotics are
superior.
C – it causes conductive hearing loss
D – In AOM with effusion
E – Mastoiditis, Meningitis, Abscess Etc.

48) A 25yr old man received a direct blow to his nose during a fight. What is the best Indication for
ENT referral?
a) swelling of nose
b) laceration over nose
c) septal deviation
d) septal hematoma
e) epistaxis
Answer – D
Septal hematoma if not drained, may organize into fibrous tissue leading to permanent damage.
Therefore need immediate draining.

2013 Nov – Colombo


12) Regarding acute otitis media,
a) Commonly present with chills and fever
b) Deafness is a late feature
c) The tympanic membrane loses its shine
d) Antibiotics are effective in the early stage
e) Surgical treatment by myringotomy
Answer – F T T T T
B – Initially reduced hearing later develops into complete deafness
C – Dull light reflex in tympanic membrane is characteristic
D – Early treatment with antibiotics is superior to other treatments.
E – Sometimes done to evacuate pus
Ophthalmology
2020 June
50) A 55 years old diabetic patient presented with a two-hour history of sudden
onset severe pain in right eye. He also complains of some haziness in vision & vomiting.
O/E right eye appears diffusely injected & hypersensitive to light. The pupil is mildly
dilated & oval shaped & cornea appears hazy. What is the likely cause for his presentation?
A. Acute closed angle glaucoma
B. Anterior uveitis
C. Infective conjunctivitis
D. Keratitis
E. Sub conjunctival haemorrhage
Answer – A
This is suggestive of acute angle closure glaucoma. (AACG)
AACG patient has an acute painful loss of vision, headache, halos around lights nausea and vomiting.
On examination they have reduced vision, red eye, corneal oedema, hazy cornea, mild dilated oval
pupils. In gonioscopy they have closed aqueous drainage angle.

2019 Nov
50) A 50-year-old male presented with an acutely painful red eye. Which of the following finding
necessitates immediate ophthalmology referral?
a) Circumcorneal redness
b) Conjunctival haemorrhage
c) Corneal abrasion
d) Episcleritis
e) Photophobia

Answer – A? (this response should be circumcorneal ingestion)


 Circumcorneal injection – Often associated with acute angle closure glaucoma, keratitis, iritis
This is a sight threatening emergency.
 Sub conjunctival haemorrhage – presents as bright red eye with only minimal discomfort
and normal vision. More common in people receiving antiplatelet or anticoagulant therapy.
Reassurance and treatment of underlying cause is needed (bailey page 680)
 Corneal abrasion - Complain of excruciating eye pain and an inability to open the eye due to
photophobia and/or foreign body sensation. Corneal abrasion per say is not sight threatening
 Episcleritis - is a localized ocular redness from inflammation of episcleral vessels.
 Photophobia – Non-specific symptom can be associated with corneal abrasions, uveitis,
glaucoma, and even other systemic causes like meningitis or migraine
 Circumcorneal ingestion suggests that patient might be having acute angle closure glaucoma
and this needs urgent opthamological referral
2019 March
41) 18-year-old man presented with redness and itching of the eyes. He also has stickiness of the
eyes. He has dysuria and pain in the left knee joint. On examination eyes are diffusely red and
injected. Possible organism would be
A. Adenovirus
B. Chlamydia
C. Gonorrhoea
D. Staphylococcus
E. Streptococcus
Answer – C
This young patient has joint involvement, eye involvement and urinary system involvement. So
Reiter’s syndrome should come into mind. Both Chlamydia and Neisseria gonorrhoea causes it.
This patient has sticky eyes. Therefore, Chlamydia is unlikely because they cause watery discharge
whereas other bacterial diseases causes sticky eyes. (refer medicine lecture notes for further details)

2018 Nov
41. A 55 year old woman with history of Type 2 Diabetes mellitus presented with left eye pain and
reduced visual acuity for 3 days duration. What is the most likely diagnosis?
a) Corneal abrasions
b) Diabetic retinopathy
c) Dislocation of the lens
d) Retinal Detachment
e) Retinal Hemorrhage
Answer – A
There is painful loss of vision. Causes include: keratitis, uveitis, acute angle closure glaucoma, optic
neuritis and trauma.
Eye complications of Diabetes patients
1. Diabetic retinopathy
2. Diabetic maculopathy
3. Cataract
4. Glaucoma
5. Diabetic keratopathy
Acute eye manifestaions are
1. Retinal detachment
2. Retinal heamorrage
3. Anterior ischemic optic neurpathy
4. Central retinal artery occlusion
5. Central vein occlusion
6. Corneal abrasions
7. Acute glaucoma
Last two causes painful reduced vision.
So out of the answers here corneal abrasions will be most suitable. But usually there should be some
trauma for the onset of corneal abrasions. Due to the lack of epithelial integrity in diabetes patients,
risk of corneal abrasions is high (keratopathy)

2017 May
32.A 60-year-old male p resents with an acutely painful red eye. He also complains of poor Vision
and vomiting. What is the most likely diagnosis?
a) Acute granuloma
b) Conjunctivit
c) Episcleritis
d) Keratitis
e) Uveltis
Answer – A
Again this should be acute angle closure glaucoma because there si acute painful loss of vision with
vomiting. (there is no medical/surgical condition called acute granuloma of eye)

2015 Nov
19. Viral conjunctivitis
A. Causes pain
B. Causes haloes around light
C. Does not impair the visual acuity
D. Has gritty sensation
E. Causes small irregular pupils
Answers – ? F T T F

Viral conjunctivitis
patients have red, watery eyes, Gritty uncomfortable feeling
O/E – generalized conjunctival injection and watery discharge, vision unaffected unless cornea
involved, has petechial conjunctival haemorrhages, follicles in tarsal conjunctiva, enlarged pre
auricular LN

C – Vision is not commonly affected in conjunctivitis but, with some viral infections a keratitis may
be present and result in visual impairment and pain.(B & L 27th edition. P 681)
2014 Nov
49) Elderly patient C/O Flashes and floaters "in the eye/E there is a visual field defect. What is the
condition?
a) Retinal detachment
b) Vitreous hemorrhage
c) Glaucoma
d) Retinal artery occlusion
e) Posterior uveitis
Answer – A

Common symptoms in ophthalmology


 The red eye” (pain, redness, photophobia, discharge) = front of eye
 Painless loss of vision = back of eye
 Misty vision/glare = cataract
 Distortion of vision/central scotoma = macula
 Flashes and floaters = vitreous/retina\

So there is a problem in vitreous / retina


Retinal detachment and vitreous haemorrhage both causes painless loss of vision.
Retinal detachment causes visual field defects also. They initially get the field defects, then suddenly
present with painless loss of vision. This is not seen in vitreous haemorrhage. also if the macula is
spared, surgical treatment carries a better prognosis than a macula involved retinal detachment.
(Ophthalmology lecture notes by Saman sir)
Neurosurgery
2020 June
46) 26y old school teacher who is previously healthy complains sudden onset severe
headache & then suddenly collapsed. O/E she was unresponsive & neck stiffness noted.
Rest of systemic examination normal. What would be the most probable diagnosis?
A. Rupture into cerebellar tumour
B. Meningitis
C. Meningioma
D. Middle meningeal artery rupture
E. Rupture of aneurysm in circle of Willis

Answer – E
This is a sub arachnoid hemorrhage.
Patient presents with a sudden onset thunderclap headache (occipital region). And may have low
GCS, focal neurological signs, photophobia and neck stiffness. Most of SAH results from rupture of
aneurysm of circle of willis (B & L 27th edition, page 661)

2019 Nov
47) 30-year-old man fallen from height coming with neck pain, all 4 limb affected upper limb
more than lower limbs affected. Motor more than sensory. Best fit diagnosis?
a) Anterior cord syndrome
b) Central cord syndrome
c) Complete transverse myelopathy
d) Cauda equina syndrome
e) Spinal shock syndrome

Answer – B - In central cord syndrome the centrally travelling upper limb fibres will be affected
more than lower limb fibres.

2017 May
33. A previously healthy 35-year-old male present with a sudden severe headache in the Occipital
region and photophobia. On examination he has neck stiffness. What is the most likely diagnosis?
a) Acute subdural haemorrhage
b) Bacterial meningitis
c) Cavernous sinus thrombosis
d) Chronic subdural haemorrhage
e) Subarachnoid haemorrhage.

Answer – E - this is also a SAH and this is an easy question.

2015 May
14. Sudural haematoma
A. Presents as confusion in elderly
B. Is managed by evacuation through burrhole
C. Lucid interval is characteristic
D. Is seen with biconvex shape in CT scan

Answers – T T F F
SDH can present as confusion. IT DOES NOT HAVE A LUCID INTERVAL, lucid interval is for EDH.
Its crescent shape in CT and Most Acute SDH with midline shift and compressive symptoms need
surgical evacuation. Small ones can be closely monitored.

2015 November Colombo Paper


46. A 20-year-old male was knocked down by a car. He could speak immediately but he was
confused on admission. 1 hour after the accident, he was found to be deeply unconscious. GCS was
5/15 & had decorticate flexion to pain. Pupils were asymmetrical but both pupils reacted to light
slightly. What is the most likely diagnosis?
A. EDH
B. SDH
C. Concussion
D. Intra cerebral hemorrhage

Answer – A this is an EDH


Trauma could speak immediately confused deeply unconscious
Also has asymmetrical pupils and deteriorating neurological features.
Paediatric surgery
2019 Nov
30) A 11-month-old child presented with a tender, tense lump at the right inguinal
region, for oneday duration. On examination the child is irritable and had
abdominal distension. What would be the most appropriate management?
a) Intravenous hydration and pain relief
b) Manual reduction and in ward observation
c) Manual reduction and routine surgery
d) Resuscitation and emergency surgery
e) USS and IV antibiotics

ANSWER – (d) – Resuscitation and emergency surgery.


 This inguinal hernia seems to be strangulated rather than incarcerated.

Clinical features of a strangulated hernia


 Sudden severe pain- Initially over the hernia and then generalized abdominal pain.
 Persistent vomiting, constipation and distension of the abdomen.
 Tense
 Tender
 Irreducible
 No cough impulse
 Rebound tenderness.

If the hernia was incarcerated i.e., hernia that is irreducible and developing towards strangulation,
management is as follows.
Most incarcerated hernias in children can be successfully reduced by sustained gentle
compression (‘taxis’) aided by cautious analgesia.
• Repair can be delayed for 24 hours to let the oedema settle.
• If truly irreducible, emergency surgery is required because of the risk of vascular compromise to
the bowel, ovary, or testis.
(Ref: Bailey and Love 27th edition Page 124, Page 1024 / Short notes in Surgery by Dr. S. Sri Shankar)
Treatment
• NBM
• IV Fluids
• Urinary catheter
• IP/OP Chart
• NG tube
• IV Antibiotics
• Emergency surgery- Hernia mesh is not used in the repair.
• No attempts are taken to reduce the hernia.
40) A father brought his 5-year-old child, with empty right side hemi scrotum. On examination there
is normal size right side testis in inguinal area. It is easily manipulated to the scrotum and remain
without traction. What is the most likely diagnosis?
a) Ascending testis
b) Ectopic testis
c) Retractile testis
d) Transverse testicular ectopia
e) Undescended testis

ANSWER - (c) – Retractile testis


• In impalpable testes, important differential diagnosis for undescended testes is the so called
“retractile testes”.
• During childhood, the testes are mobile and the cremasteric reflex is active so that in some boys,
any stimulation of the skin of the scrotum or thigh causes the testis to ascend and to temporarily
disappear into the inguinal canal. When the cremaster relaxes, the testis reappears only to vanish
when the scrotal skin is touched again. A retractile testis can be gently milked from its position in
the inguinal region to the bottom of the scrotum. A diagnosis of true incomplete descent should
be made only if this is not possible. A retractile testis can be manipulated into the base of the
scrotum without tension but is pulled up by the cremaster muscle.
• With time, retractile testes reside permanently in the scrotum; however, follow-up is advisable
because, rarely, the testis ascends back into the inguinal canal. The ascending testis needs an
orchidopexy.
• An ectopic testis lies outside its normal line of descent, often in the perineum.
• An undescended testis is in the line of normal descent

When the testis is impalpable, ultrasound may be helpful in identifying the intracanalicular testis,
while laparoscopy may be needed to differentiate between the abdominal testis and a truly
absent testis.
49) 8 months old boy who sleeps in a separate room has a sudden onset of cough that resolves after
a few minutes. The baby appeared to be normal except for the drooling and refusal of feeding. Most
probable diagnosis is
a) Epiglottitis
b) foreign body in the piriform fossa
c) foreign body in the airways
d) foreign body in oesophagus
e) GORD
ANSWER – (b) - Foreign body in the piriform fossa
• Examination may be prevented by trismus, pain, and anxiety, but the presence of a foreign body
may be suspected by a salivary pool within the piriform fossa or adjacent oedema and erythema
of the pharyngo-laryngeal mucosa.
• Drooling of saliva is seen when the foreign body is lodged somewhere close to the oral cavity,
mostly in the piriform fossa.

2019 March
3) Regarding appendicitis
a) Most commonly occur in child<3 years
b) Lymphoid proliferation is a cause
c) Most tender area is Mc Burney's point
d) Non operative intervention done for appendicular mass
e) Underlying caecal ca should excluded in elderly 'presented with appendicitis'
Answers
a) True. Common in child and early adult life
b) True. Causes are,
• Fecaliths
• Lymphoid hyperplasia
• Parasites
• Foreign bodies
• Neoplasms
c) True. Location of appendix
d) true. Conservative management, monitor PR, BP, RR, QHT and mass size.
e) true. Ca of caecum can mimic or cause obstructive appendicitis in adults.

14) What are the causes for continuous dribbling in a 3-year-old girl?
a) ectopic ureter
b) meningomyelocele
c) posterior urethral valve
d) vesico-ureteric reflux
e) recto vesical fistula
Answers
a) True
b) True
c) false. Causes urinary retension
d) false
e) True

48) 9-month-old baby boy presented with right inguinal hernia. What Is the appropriate
management?
a) Herniotomy
b) Herniotomy with Bassini repair
c) Laparoscopic mesh repair
d) Lichtenstein mesh repair
e) Observe for six months
Answer B. much feasible, reduce damage. Usually due to patent processes vaginalis. In infants, it is
repaired promptly to prevent risk of strangulation.
2018 Nov
40) A new-born baby is having continuous frothing from the mouth and a respiratory distress. What
is the most likely diagnosis?
a) Duodenal atresia
b) Volvulus
c) Intestinal malrotation
d) Oesophageal atresia
e) Diaphragmatic Hernia
Answer - D

Clinical Features of Oesophageal atresia


• continuous frothing from mouth
• cough after feeds
• recurrent lung infection
• raspatory distress following meals.
• Most of the oesophageal atresia have oesophageal fistula also. (90%)
• sometimes oesophageal atresia also associated with VACTREL anomalies.
Next investigation will be chest x-ray following insertion of NG tube. There will be coiling of NG tube
and gaseous shadows in stomach indicate fistula.

2017 May
42. A 9-month-old baby is admitted to hospital with episodic crying and the passage of blood with
Stools over the last 6 hours. A vague mass is palpable in the right hypochondrium.
What is the most appropriate Ix?
a) Abdominal USS
b) Barium enema
c) Colonoscopy
d) CECT abdomen
e) Supine X-ray of the abdomen
Answer A
This is an intussusception (common in 2 months to 2 years) causing intestinal obstruction in ileocolic
area which is the most common area for this condition. This can be manifested as,
• Vomiting, dehydration
• Abdominal Pain, distension
• PR bleeding/ Red currant jelly stool
• Sausage shaped abdominal mass in right upper quadrant
• Irritability
• Constipation
Diagnosis is done with plain X-ray abdomen, but to confirm it requires an abdominal USS.
Management includes immediate fluid and electrolyte replacement and non-operative reduction
with air enema if no perforation is suspected.
2016 Nov
42. A new-born on day 5 of birth had bile-stained vomiting. His vitals, hydration status and abdomen
were normal. Which of the following is the most likely cause?
a. Pyloric stenosis
b. Duodenal atresia
c. Intestinal malrotation
d. Hirschsprung disease
e. Gastro oesophageal reflux disease
Answer C
Pyloric stenosis doesn’t cause bilious vomiting, it causes non-bilious projectile vomiting. Duodenal
atresia if present it appears since the first feed.
In neonates, bilious vomiting is usually caused by life threatening conditions such as malrotation and
volvulus.

2015 May
18. Regarding intussusception in children
A. Palpable abdominal mass is a feature.
B. USS abdomen is a reliable diagnostic test.
C. HSP is a predisposing condition.
D. Hydrostatic reduction is an accepted method of treatment.
E. “Red-currant jelly” stool is the most common feature.
Answer
A- True. Sausage shaped abdominal mass in right upper quadrant
B- True. To confirm, it requires an abdominal USS.
C- False. Causes are hyperplasia of lymphoid tissue (payers’ patches) which may be secondary to a
viral infection majorly and pathologies such as Mickell’s Diverticulum, enteric duplication cyst or
small bowel lymphoma.
D- True. Air reduction and hydrostatic reduction done if the baby hasn’t perforated bowl.
E- True. An infant with bloody stools intussusception should be considered.

36. 3 weeks old baby brought with bilious vomiting. Midgut malrotation is suspected clinically. The
best diagnostic test
A. USS abdomen
B. Laparoscopy
C. Lower GI contrast study
D. Upper GI contrast study
E. UGI endoscopy
Answer D

2013 Nov – Colombo SBA


17) 40-year-old mother with an antenatal history of polyhydramnios delivered at 36 week of
gestation 10 hours after delivery baby has been vomiting since his 2d breast feed and vomitus is
green. Baby has passed a stool that was greyish in colour. There is epigastric fullness and vomiting
soon after feeding. What is the most likely diagnosis?
a) Duodenal atresia
b) Ileal atresia
c) Infant pyloric atresia
d) Meconium plug syndrome
e) Necrotising enterocolitis
Answer A.
In duodenal atresia distal to ampulla atresia can give a presentation of polyhydramnios, bilious
vomiting and grey stools as no bile goes further down. This is confirmed with a plain abdominal X-ray
and USS abdomen which would show a “Double Bubble” appearance.

2013 – Eastern
21) 9 months old baby was admitted to hospital with crying lasted for 10 minutes duration. He had
similar attacks for last 5 hours. Also, he passed red colour stools. On examination he is quite well. He
is not pale. No abdominal abnormality except vague lump in RHC area. What is the probable
diagnosis?
a) Meckel’s diverticulum
b) Intussusception
c) Pyloric stenosis
d) Paralytic ileus
e) Duodenal atresia
Answer B

2012 – Nov
47) True or false regarding Hirschsprung’s disease
a) Present as delay in passage of meconium
b) Associated with gestational diabetes mellitus
c) Can manage conservatively if a short segment is affected
d) Can lead to toxic mega colon
Answers
Hirschsprung disease is a condition that is characterised by absence of intramural ganglion cells and
presence of hypertrophic nerves in the distal large bowel. This causes spasm in affected gut leading
to functional intestinal obstruction.
A- True. Can present with delayed passage of meconium, abdominal distension and bilious vomiting,
but may not be diagnosed since childhood or even adult life.
B- False. can be familial or associated with Down syndrome or other genetic disorders (Chromosome
10 and 13)
C- False. Surgical intervention is performed to remove the aganglionic part.
D- False. Complications are enterocolitis, residual constipation and/or faecal incontinence.
Urology MCQs
2020 June
(15) Regarding management of BPH
A. large asymptomatic benign prostatic hyperplasia needs surgical intervention
B. Bladder residual volume of 100 ml need urgent TURP
C. Finesteride reduces the size the prostate gland
D. Tamsulozin increase the success rate of trial with a catheter
E. Transurtheral resection can leads retrograde ejaculation

Benign prostatic hyperplasia


 Occurs in the transitional zone.
 Usually Presents with voiding LUTS such as hesitancy, poor stream, intermittent stream and
post micturition dribbling.
 In long standing cases storage LUTS such as urgency and frequency may occur.
 It Does not increase risk of malignancy.
Complications- 1. Recurrent UTI
2. Calculi
3. Chronic retention
4. Acute retention on a background of LUTS and renal failure
5. Bladder diverticuli
6. Hemorrhage
Investigations: 1. USS KUBP with post micturition residual volume is the Ix of choice.
2. S.Cr, BU and SE- To asses renal dysfunction
3. UFR- UTI
4. Repeated assessment of IPSS to check symptom progression.
5. Urine flow rate measurements.
6. If haematuria occurs Cystoscopy should be done.
Treatment can be medical or surgical.
Medical Mx- 1. Tamsulosin – Alpha blocker- Symptomatic improval may occur in 48 hrs.
2. 5 alpha reductase inhibitor- Finasteride, dutasteride- They act by reducing the
prostate size. It takes about 6M.

Surgical Mx- Indications- 1. Renal dysfunction


2. Recurrent complications such as UTI, stones
3. Persistent symptoms with high IPSS scores despite medical Mx
Surgery of choice- TURP

Post TURP complications (can be asked in MCQ and during long case viva)
1. Immediate complications: Haemarrhage and Post TURP syndrome
2. Infections and urosepsis can occur during the hospital stay.
3. Urinary incontinence due to external sphincter damage
4. Bladder neck hypertrophy
5. Retrograde ejaculation due to internal sphincter damage
6. Erectile dysfuction
7. Recurrence
 TURP syndrome has been asked. It is due to use of hypotonic fluids for bladder irrigation. These
fluids will be absorbed to the prostatic veins and will result in a dilutional hyponatraemia with the
patient becoming drowsy, confused, vomiting and may develop fluid overload as well. Immediate
step is to do a SE to confirm. If seizures or very low Na needs hypertonic saline.

Answers:
A. False. Read the answer carefully because it says large asymptotic BPH. See indications for
surgical management
B. False. Severe symptoms and large residual volume of urine will usually require treatment.
Men with mild symptoms, good flow rates ( >15ml/s) and good bladder emptying ( Residual
urine <100ml) may be safely managed by reassurance and review. Such patients rarely
develop severe complications such as retention in long term.
C. True
D. True. Tamsulosin doesn’t shrink prostate but it works by relaxing the muscles in the prostate
and bladder.
E. True. See complications of TURP.

( 16) A 50 year old man is diagnosed to have recurrent non muscle invasive transitional cell carcinoma
of bladder. Which of the following modalities are used to reduce the recurrence?
A. Diathermy ablation of complete bladder mucosa
B. Intravesical BCG
C. Intravesical Mitomycin C therapy
D. Low grade radiotherapy
E. Systemic chemotherapy

Bladder Carcinoma
 Most are transitional cell CAs.
 Squamous cell CA occurs mostly with schistosomiasis infection, bladder calculi etc.
 Adeno CAs may occur but are very rare.

Clinical features – Dysuria, urgency, frequency- Mostly in Carcinoma in situ.


– Painless macroscopic haematuria is the most common presentation.
– Suprapubic and back pain with extravesicular spread
– Renal failure due to B/L ureteric obstruction

Risk factors: 1. smoking. It is the No.one of risk factors.


2. Occupations such as hair saloons, textile industries are also at risk.
3. dye industry, painting etc.

Investigations:
 First line Ix for haematuria
1. UFR- > 3 RBC/hpf is significant
2. Urine culture and ABST- UTI
3. USS KUB
4. X ray KUB- Ureteric calculi
5. Cystoscopy- Bladder tumors, stones, prostate size all can be assessed.
 Other Investigations;
1. Urine cytology- May be +ve in Carcinoma in situ
2. Early morning 3 urine samples for AFB and culture- TB
3. CT KUB- Renal masses.
4. 24hr urinary Ca excretion
5. IVU/CTU- Calculi detection If all –ve nephrology referral for renal biopsy.
Management of bladder CA;
 In all cases TURBT should be done during cystoscopy.
 Tx after that will depend on the stage.
 All muscle invasive bladder CAs need radical cystectomy or radiotherapy.
 CIS is usually Mx with intrevesicular BCG.
 T1a can be Mx with intravesicular mitomycin. Then BCG can be used if it fails. If that also
fails radical cystectomy is needed.
 After cystectomy either an ileal conduit or an orthotropic bladder is used.

Answers:
A. False.
B. True
C. True
D. False
E. False

(39) 26 years old pregnant woman in 18 weeks of POA presented with ureteric colic. Pain settled with
analgesia. Ultrasound scan revealed a 6mm stone in the lower ureter. No signs of sepsis. What is the
next management step? (SBA)
A. ESWL
B. Pyelolithotomy
C. Ureteric stent
D. Medical espulsion therapy
E. Ureteroscopic extraction

Answer ( D)
Management of ureteric stones: (can be questioned in MCQ, SEQ, and during viva) detailed and
comprehensive and only needed information for undergraduate level is given in sirs note.
As a summary;
Urinary stones
● The commonest urinary tract stones are calcium oxalate.
● Stones are best diagnosed on a non-contrast computed tomography kidney–ureter–bladder (KUB)
● Most stones <5 mm will pass spontaneously.
● Medical expulsive therapy remains controversial.
● Indications for surgical intervention are persistent pain, obstruction and infection
(49) A 52 years old man is investigated for recurrent urinary tract infections. Ultrasound scan shows
single globular shape bladder stone in 8cm in size. What would be the best treatment in Sri lanka?
(SBA)
A. Conservative management
B. Cystolithoplexy
C. Extracorporeal shock wave lithotripsy
D. Laser lithotripsy
E. vesicolithotomy

Answer: (E )
Management of bladder stones can be asked in MCQ, viva
Bladder stones;
Characteristic feature is pain towards the end of micturition (This has been questioned previously).
Also dysuria, urgency, frequency, Intermittent stream due to obstruction relieved by change in
posture, pain during micturition radiating to the tip of the penis and haematuria may occur.

 Haematuria occurs towards the end of micturition but this is not specific as terminal haematuria
can occur even with small bladder tumors.
 USS will miss a bladder calculus most of the time.
 Best Diagnostic method is cystoscopy.
 Transurethral cystolithalopexy, Percutaneous suprapubic cystostomy, open supra pubic
cystostomy are other Treatment options.
Also be through with management of a patient with recurrent UTI
 Evaluate with a metabolic screen. Serum and urine calcium, phosphate, oxalate and urate levels
using 24 hr urine collection (This is the metabolic screen)
need metabolic screening for;
1. Formation of new stones while on Treatment for stones
2. large stones
3. young patients
(Detailed information given in pethiyagoda sirs UTI note. Do read that) because can be asked as a part
of SEQ, mcq

2019 November
(15) 55-year-old male presented with urinary retention. After catheterization urine output was
2000ml. Regarding his management in next the 48 hours
A) Measure serum creatinine
B) Prostatectomy
C) PSA levels
D) Tamsulosin
E) Trial without catheter
Management of acute urine retention

 Urgent catheterization ———> fails——> suprapubic puncture

If <1000ml If >1000ml,
 Go for trial without catheter ( TWOC)  Continue fenesteride and refer
 Discharge patient with catheter After to urology clinic.
starting on
 Tamsulosin 0.4mg nocte for one week
 Fenestride 5mg once daily for one
week
 Tadalafil 25mg
 Dulcolax 2tabs nocte for 48 hours

After on week of TWOC ,


At ward
Ask patient to drink plenty of water and make
the bladder full
Measure residual volume

If residual volume >200ml If residual volume <200ml or unable to pass urine


 Send patient home with  Catheterize patient again and send home
tamsulosin, finesteride and with finesteride and bisacodyl
bisacodyl ( Dulcolax)  Refer to urology clinic with USS- PMR, flow
 Refer to urology clinic for USS rate
bladder PMR and flow rate  Asses IPSS
 Asses IPSS  If >30, Identify a etiology and offer
 If going down can manage TURP/stenting.
medically.
 If increasing-surgical
management.
In this question Since urine output is >1000ml, points towards chronic retention.
Management of chronic urine retention

Two types.
1. High pressure chronic retention
2. Low pressure chronic retention
High pressure chronic retention Low pressure chronic retention
 Feel the bladder- it is hard  Management- Do USS and S.Cr
 Dangerous condition  If normal- monitoring
 Treatment depends on renal function  But if patient C/O frequency and
 If patient presents with high pressure chronic nocturia or recurrent UTI, offer
retention- Do S.Cr, S.E and USS-KUB intermittent self catheterization.
 If S.Cr is high, relieve obstruction immediately via
catheterization.
Once catheterization ask patient to measure UOP
hourly, Daily weight chart, SCr twice daily.
If UOP >250ml/hr- replace 60% of previous UOP as
IV saline
If Body weight and S.cr reduces- improving.
If Body weight and S.Cr increases- over hydration.
If body weight reduces and S.Cr increase-
dehydration.
When become normal, offer definitive treatment
(TURP) and should be started on finesteride and
alpha blockers.

 If S.Cr is normal, No catheterization. Offer corrective


surgery immediately. (TURP)
 If catheterization fails, insert a suprapubic catheter
and do a descending/ ascending urethrogram.

Answers:
A) True. The increased intramural tension results in functional obstruction of the upper urinary tract
with the development of bilateral hydronephrosis. Thus leading to CKD. So need to measure S.Cr to
asses renal function.
B) False. TURP is done. Nowadays prostatectomy usually not done (practically).
C) True. PSA level can be taken to get and idea of underlying pathology.
D) True. This causes relaxation of bladder neck.
E) False. Should allow a period without catheterization, because if catheter is kept in dwelling
continuously, the bladder loses the sensation of dissension and so the micturition reflex may get
altered and this leads the patient catheter bound for rest of the life. But at least two weeks of
catheterization is done in chronic retention. So as it is given within 48 hours, it is considered as false.
(Bailey and love 27E page 1463).
(16) Which of the following are predisposing factors for renal cell carcinoma?
A) Recurrent dialysis
B) Renal papillary necrosis
C) Recurrent Pyelonephritis
D) Vesicoureteric reflux
E) Von Hippel Lindau disease

Renal cell carcinoma (RCC) encompasses a heterogeneous group of cancers derived from renal tubular
epithelial cells. The male: female ratio is approximately 2:1.
Risk factors for RCC include:
1. Age
2. Sex
3. Obesity
4. Hypertension
5. cigarette smoking
6. Other medical conditions have been associated with RCC including chronic kidney disease,
hemodialysis, kidney transplantation and acquired cystic disease of the kidney (ACDK).
7. Genetic factors also contribute to RCC risk; example VHL, the mutated gene underlying von
Hippel–Lindau disease, which is characterized by a high risk of developing ccRCC.
(References Bailey and Love 27E page 1417)
Answers:
A) True
B) false
C) false
D) false
E) True

(19) Regarding acute kidney injury


A) Hemodialysis’s mandatory
B) Hypokalemia is a feature
C) Intra-operative hypotension is a risk factor
D) is caused by IV contrast
E) Is irreversible

Answers
A) False. Hemodialysis is not needed always. Know indication might come in handy in other
subjects also.
Indications for Dialysis
1. Symptomatic uremia
2. Hyperkalemia not controlled by medical management
3. Pulmonary oedema unresponsive to diuresis
4. Severe acidosis
5. To remove medications causing AKI ( eg; Gentamicin, Li, severe aspirin overdose)
B) False. It results in Hyperkalemia
C) True. Hypervolemia resulting in hypotension due to blood loss in surgery is an pre renal
cause for AKI
D) True. Is an Intra parenchymal cause for AKI
E) False. It is usually reversible over days or weeks (but not always).

(36) 40-year-old women presented with polyuria, polydipsia, and abdominal cramps. She has a history
of recurrent renal calculi. What is the most appropriate step in the management? (SBA)
A) 24-hour urine collection for metabolic screening
B) Serum calcium level
C) Serum parathyroid hormone level
D) Stone analysis
E) X ray Kidney, ureter, bladder

management of a patient with recurrent UTI


Evaluate with a metabolic screen. Serum and urine calcium, phosphate, oxalate and urate levels using
24 hr urine collection (This is the metabolic screen)
need metabolic screening for;
1. Formation of new stones while on Treatment for stones
2. large stones
3. young patients
(Detailed information given in pethiyagoda sirs UTI note. Do read that) because can be asked as a
part of SEQ, mcq)

Answer: (A)

(39) Otherwise healthy man is diagnosed with high intensity(1350HU) 2cm stone in the renal pelvis.
He has episodic pain and haematuria. Renal functions are normal. No evidence of infection. He is
best treated with
A) Extracorporeal shock wave lithotripsy
B) Medical desolation therapy
C) Medical expulsion therapy
D) Percutaneous nephrolithotomy
E) ……..

Answer ( D )
 Small stones (<5 mm), especially in the distal ureter, are treated expectantly as they are likely to
pass spontaneously.
Medical expulsive therapy may be considered if it fails. Interventions may be needed if,
 The stone is not moving
 If patient has only 1 kidney
 Stones larger than 5mm needs, some intervention depending on the site of stone
 Renal stones
 <2cm – Extracorporeal shock wave lithotripsy (ESWL)
 >2cm – Percutaneous Nephrolithotomy (PCNL)
 For Stones of higher density (> 1000 HU), ESWL can’t be used.
 Cystine stones are therefore relatively resistant to ESWL due to their hardness. Therefore,
although less than 2cm, here we should use PCNL.
 PCNL is used in obese, pregnant, and patients on anticoagulants because ESWL is
contraindicated
 Renal pelvis and upper 1/3rd
 PCNL
 ESWL
 Uretero reno scopy (URS)
 Open surgery
 Mid ureter
 URS + IPL (intracoporeal pneumatic lithotripsy)
 Lower ureter
 URS + IPL
 Dormia basket extraction

In this question, the stone is in the kidney and the size is exactly 2cm, so both ESWL and PCNL seems
possible, but the density of the stone is > 1000 HU, so ESWL is not effective. PCNL is the best option

(40) A farther brought his 5-year-old child, with empty right side hemi scrotum. On examination
there is normal size right side testis in inguinal area. It is easily manipulated to the scrotum and
remain without traction. What is the most likely diagnosis? (SBA)
A) Ascending testis
B) Ectopic testis
C) Retractile testis
D) Transverse testicular ecotopia
E) Undescended testis

Answer: ( C )
 In impalpable testes, important differential diagnosis for undescended testes is the so called
‘retractile testis’.
 During childhood the testes are mobile and the cremasteric reflex is active so that in some
boys, any stimulation of the skin of the scrotum or thigh causes the testis to ascend
and to temporarily disappear into the inguinal canal.
 When the cremaster relaxes, the testis reappears only to vanish when the scrotal skin
is touched again.
 A retractile testis can be gently milked from its position in the inguinal region to the
bottom of the scrotum.
 A diagnosis of true incomplete descent should be made only if this is not possible.
 A retractile testis can be manipulated into the base of the scrotum without tension
but is pulled up by the cremaster muscle.
 With time, retractile testes reside permanently in the scrotum; however, follow-up is
advisable because, rarely, the testis ascends back into the inguinal canal.
 The ascending testis needs an orchidopexy.
 An ectopic testis lies outside its normal line of descent, often in the perineum.
 An undescended testis is in the line of normal descent.
 When the testis is impalpable, ultrasound may be helpful in identifying the
intracanalicular testis, while laparoscopy may be needed to differentiate between the
abdominal testis and a truly absent testis.

2019 March
(13) A 45 year women presents with microscopic haematuria. Investigations include
A) CRP
B) Cystourethroscopy
C) Repeat if the women had menstruation
D) USS KUB
E) Uroflowmetry

 Haematuria occurs when there is blood in the urine. This is now classified as visible haematuria
(VH) or non-visible hae-maturia (NVH). Older terminology referred to macroscopic (or frank
or gross) haematuria and microscopic (or dipstick) haematuria.
 Causes of haematuria include Trauma, Infection and Neoplasm (think of TIN) at all levels of
the urinary tract.
 Apart from young females with a proven UTI, most patients with haematuria, whether visible
or non-visible, will be investigated urgently at a haematuria clinic where they will have a renal
ultrasound scan (USS) and a flexible cystoscopy as a minimum. ( references: Bailey and love
27E pages 1375-1376)
First line investigations:
1. UFR
2. Urine culture and ABST
3. Cytology
4. AFB
5. USS- KUB
6. X ray- KUB
7. Cystoscopy
Answers:
A) True
B) True
C) True
D) True
E) False

(14) Causes for continuous dribbling of urine in a female child


A) Ectopic ureteric opening
B) Myelomeningocele
C) Posterior urethral valve
D) Recto-vaginal fistula
E) Vesico ureteric reflux

Constant dribbling occurs when there is a ureteric fistula or an ectopic ureter associated with a duplex
system opening into the urethra beyond the urethral sphincter in females or into the vagina. The
history is diagnostic, and IVU or ultrasonography may reveal the upper pole segment, which is
often poorly functioning. Treatment is by excision of the aberrant ureter and portion of kidney. A
ureteric fistula can be difficult to diagnose and its demonstration may require retrograde
ureterography and a high degree of suspicion. ( Bailey and love 27E page 1432)

Answers:
A) True
B) True
C) False. Occurs in a male child. Question says a female child. So to miss getting these type of
answers wrong, read the question carefully and underline the important clues.
D) False.
E) False. Usually occurs in a male child due to posterior urethral valve.

2018 November
(13). 70 years old patient presented with acute urinary retention due to benign prostatic enlargement.
Regarding his management
A) Initial Urine volume should be measured while catherizing
B) Intermitent clamping needed following initial catheterisation.
C)
D) Treated with opiod analgesics
E) Oral tamsolusln helps In rendering the patient catheter free later on

Answers:
A) True.
B) False.
C)
D) False. Treatment is to catheterize.
E) True. Will cause relaxation of smooth muscle and relieve obstruction.

(14) Regarding simple renal cyst


A) ……….
B) Causes hematuria
C) Infection is a complication
D) Most of the simple renal cysts are found incidently on imaging
E) Mostly needs renal biopsy

Classification of renal cysts based on CT criteria known as the Bosniak classification was devised.
Nowadays, this classification can also be applied to MR imaging.
The Bosniak classification of renal cystic masses is as outlined:

I Simple benign cysts with hairline thin walls. The cysts do not contain septations, calcifications
or solid components. There are no enhancing soft tissue components.

II Benign cystic lesions - hairline thin septations. fine calcifications in the walls. Minimal
enhancement of the hairline thin smooth wall. No enhancing soft tissue components. Non-
enhancing high-attentuation cysts <3 cm in size are included.

III Indeterminate masses - thickened walls and septations that are irregular. The cyst walls or
septations show enhancement.

IV Malignant cystic masses - Thick and irregular walls and septations may be present, but enhancing
soft tissue components are seen, independent of wall or septal enhancement.
Complications:
1. flank or abdominal mass
2. Pain is commonly a dull loin ache. Haemorrhage into a cyst causes a more acute severe
pain, as does passing a calculus from the diseased kidney.
3. Heamturia and cyst heamorrhage
4. Urinary tract infection ( bailey and love 27E pages 1389-1390)
5. In rare cases, simple kidney cysts can become large enough to
 block blood or urine flow through the kidneys or the ureters
 cause high blood pressure

Answers;
A)
B) True
C) True
D) True. Often discovered incidentally on imaging. Rarely give symptoms and are often multiple.
E) False

(32) 60 year old male presented with increased frequency of urinary passage. What is the most
supportive feature for bladder urinary stones? ( SBA)
A) Urgency
B) Hematuria at the end of the stream
C) Pain at the end of micturition
D) Colicky loin to groin pain
E) Blood at the urethral meatus

Answer: (C )
Definitions –
 A primary bladder stone is one that develops in sterile urine; it often originates in the
kidney.
 A secondary stone occurs in the presence of infection, outflow obstruction, impaired
bladder emptying or a foreign body.

Clinical features –
Men are affected eight times more frequently than women. Stones may be asymptomatic and
found incidentally.
1. Frequency is the earliest symptom.
2. sensation of incomplete bladder emptying.
3. Pain (strangury) is most often found in patients with a spiculated oxalate calculus. It occurs
at the end of micturition and is referred to the tip of the penis or the labia majora; more
rarely it is referred to the perineum or suprapubic region. The pain is worsened by
movement.
 In young boys, screaming and pulling at the penis with the hand at the end of
micturition are indicative of a bladder stone.
4. Haematuria is characterized by the passage of a few drops of bright-red blood at the end of
micturition, and is due to the stone abrading the vascular trigone.
5. Interruption of the urinary stream is due to the stone blocking the internal meatus.
6. Urinary infection is a common presenting symptom.

(48). A 7 year old boy was brought to the surgical casualty complaining of an acute testicular pain for
4 hours duration. On examination, Right testis is in the transverse position and very tender. What is
the most appropriate next step in the management?
A) Ultrasound scan of the testis and scrotum.
B) Emergency surgical exploration.
C) Give antibiotics and testicular support.
D) Analgesics and reassess in 2 hours.
E) Ice pack and scrotal support.

Answer: (B)
Know causes of acute testicular pain.
There are 5 conditions for acute painful scrotum.
1. Testicular torsion
2. Epididymoorchitis
3. Bleeding into a tumor
4. Torsion of testicular appendix
5. Strangulated inguinoscrotal hernia

Torsion:
 Testicular torsion is most common between 10 and 25 years of age, although a few cases occur
outside this age range.
 Typically there is sudden agonizing pain in the groin and the lower abdomen and the patient
feels nauseated and may vomit.
 Torsion of a fully descended testis is usually easily recognized.
 Can occur spontaneously without any trauma.
 Can be triggered by sexual intercourse, trauma, nocturnal erections and heavy straining
during defecation.
 The scrotum is swollen and tender, while the skin is usually not erythematous initially
(although it may become so with a prolonged history) and the patient is apyrexial.
 The testis itself is swollen and tender and seems high within the scrotum, while the tender
twisted cord can often be palpated above it. The cremasteric reflex is lost.
 Pain will become worse upon elevation of the testis.
 Even though the classical picture is not there if someone presents with sudden on set
testicular pain without a demonstrable cause it is managed as testicular torsion until
proven otherwise. So no investigations are done.
 Urgent surgical exploration of the testis is done as gangrene may occur within 4hours.
 If torsion is seen that testis as well as the other testis should be fixed immediately.
 If gangrenous testis orchidectomy needs to be done.
 Torsion of one testis alone may be enough to cause subfertility as the antigens released
may result in formation of antisperm antibodies.
 USS with Doppler m ay show absent blood flow to the testis but it may give false
negatives . So never delay surgery until USS Doppler in torsion.

Differential diagnosis
 Redness of the skin and a mild pyrexia may result in the condition being confused with epididymo-
orchitis in the older patient;
however, in epididymo-orchitis there will usually be dysuria associated with the accompanying
urinary infection.
Elevation of the testis reduces the pain in epididymo-orchitis while it makes it worse in torsion.
 Torsion of a testicular appendage cannot always be distinguished with certainty from testicular
torsion. The most common structure to twist is the appendix of the testis (the pedunculated
hydatid of Morgagni). The twisted testicular appendages can sometimes be visible through the
scrotal wall as a small dark spot. If the diagnosis is made clinically, then conservative management
is possible, but if in doubt then surgical exploration should be undertaken with ligation and
amputation of the twisted appendage.
 In mumps orchitis, the cord is not particularly thickened and the condition is often bilateral.
Idiopathic scrotal oedema is an oddity that occurs between the age of 4 and 12 years and must be
differentiated from torsion. The scrotum is very swollen but there is little pain or tenderness.
The swelling is usually bilateral and may extend into the perineum, groin and penis. It is thought
to be an allergic phenomenon and occasionally there is eosinophilia. The swelling subsides after a
day or so but may recur.
 Very occasionally, torsion can be convincingly mimicked by a small tense strangulated inguinal
hernia compressing the cord and causing compression of the pampiniform plexus.

( References: Bailey and love 27E pages 1500-1501)

(Clues given in this question are young boy, acute pain, testis position change, tender)

2017 May
(13). High pressure chronic retention
A) Cause nocturnal enuresis
B) Is complicated by bilateral hydronephrosis
C) Is pain full
D) Is treated effectively with alpha blockers
E) Results in diuresis after catheterization

In chronic retention there is no pain. These patients are at risk of upper tract dilatation because of
high intravesical tension – they require urgent urological referral. Men with impaired renal
function may develop postobstructive diuresis after catheterisation. Such men need careful
monitoring, with replacement of inappropriate urinary losses by intravenous saline; they are
also at risk of haematuria as the distended urinary tract empties. Often it is several days before
full renal recovery occurs.

Answers:
A) True. Nocturnal enuresis is involuntary loss of urine during sleep. A common cause in an
elderly male is chronic retention of urine with overflow incontinence.
B) True. Due to chronic back pressure.
C) False. Painless
D) False. In high pressure chronic retention, catheter/ TURP is treatment of choice. No
pharmacological management.
E) True. Post obstructive diuresis. Is due to three mechanisms.
 High fluid volume in body
 Increased BU (osmotically active)
 Acute tubular necrosis
Post obstructive diuresis is detected if hourly UOP >200ml.

(14) Regarding scrotal conditions


A) Acute epidydimo-orchitis in young men associated with non-gonococcal urethritis
B) In testicular torsion the contralateral testis requires fixation
C) Infantile hydroceles communicate with the peritoneal cavity
D) Testicular tumours commonly metastasize to inguinal lymph nodes
E) Undescended testis are best treated at puberty

Answers:
A) True.
Inflammation confined to the epididymis is epididymitis; infection spreading to the testis is
epididymo-orchitis.
In young sexually active men, the most common cause of epididymitis is now Chlamydia
trachomatis, but gonococcal epididymitis is still occasionally seen.
In older men with bladder outflow obstruction, epididymitis may result from a urinary infection.

B) True
Management of Testicular torsion- Exploration for torsion should be performed through a
transverse scrotal incision. If the testis is viable when the cord is untwisted, it should be
prevented from twisting again by fixation with three non-absorbable sutures between the
tunica albuginea of the testis and the scrotal raphe. The use of absorbable sutures risks the
possibility of recurrent torsion at some time in the future. The other testis should also be
fixed because the anatomical predisposition is likely to be bilateral. If there is clinical doubt
as to testicular viability after detorsion of the testis, then it should be wrapped in a warm
swab and observed over a few minutes. If a small incision in the tunica albuginea
demonstrates bright red arterial bleeding then the testis may survive. An infarcted testis
should be removed – the patient can be counselled later about a prosthetic replacement.

C) True.
A hydrocoele is an abnormal collection of serous fluid in a part of the processus vaginalis,
usually the tunica vaginalis.
Acquired hydrocoeles are primary or idiopathic, or secondary to epididymal or testicular
disease.
Aetiology - A hydrocoele can be produced in four different ways
1. By connection with the peritoneal cavity via a patent processus vaginalis (congenital).
2. By excessive production of fluid within the sac, e.g. a secondary hydrocoele.
3. By defective absorption of fluid; this appears to be the explanation for most primary
hydrocoeles, although the reason why the fluid is not absorbed is obscure. They are
sometimes called vaginal hydrocoeles.
4. By interference with the lymphatic drainage of scrotal structures.

In congenital hydrocoele, the processus vaginalis is patent and connects with the peritoneal
cavity. The communication is usually too small to allow herniation of intra-abdominal contents.
Pressure on the hydrocoele does not always empty it but the hydrocoele fluid may drain into the
peritoneal cavity when the child is lying down; thus, the hydrocoele may be intermittent. Ascites
should be checked for if the swellings are bilateral.

D) False.
Testicular tumour is a solid testicular lump that cannot be felt separately from the testis may
be a malignant tumour. Lymphatic spread is to the para-aortic lymph nodes. Ultrasound is a
mandatory investigation in all cases of suspected testicular tumour. Tumour markers (AFP
and hCG) should be measured prior to orchidectomy.

E) False.
 Incomplete descent of the testis occurs when the testis is arrested in some part of its
normal path to the scrotum.
 An ectopic testis is a testis that is abnormally placed outside this path.

Undescended testis
 About 4% of boys are born with one or both testes incompletely descended.
 About two-thirds of these reach the scrotum during the first 3 months of life, but full
descent after that is uncommon.
 Histological changes in the testis can be seen from 1 year of age.
 An incompletely descended testis tends to atrophy as puberty approaches.
 Boys with undescended testes are at greater risk of infertility, testicular malignancy, hernia
and testicular torsion.
 Therefore Orchidopexy is usually performed before the boy reaches 12 months of age in
an attempt to prevent the consequences described earlier.
(20) Regarding Fournier gangrene
A) Broad spectrum antibiotic therapy is the main modality of treatment
B) Cause gangrene of the testis
C) Immunocompromised patients are at a higher risk
D) Is caused by multiple organisms
E) Occurs in ischemic issues

Fourniers gangrene is a Polymicrobial infection of skin and fascia of scrotum, penis and perineum.
Testis is healthy. Needs urgent Surgical debridement with IV broad spectrum antibiotics. DM, HIV, IV
drug users like immunocompromised states and post Surgical ( specially perineal Surgery , injection Tx
for haemarrhoids ) patients are at a higher risk. Gangrene occurs due to an obliterative arteritis caused
by the infection.

Answers:
A) False. Surgery is main modality. Treatment of a case of Fournier’s gangrene is a surgical
emergency. Initial management involves intravenous fluid resuscitation and early use of broad
spectrum intravenous antibiotics. Urgent wide surgical excision of the dead and infected
tissue is essential and the extent of the internal necrosis is typically much greater than the
external appearances suggest, such that extensive debridement is often necessary.
B) False. Testis is healthy.
C) True.
D) True. It is characterized by a polymicrobial infection of the soft tissues of the perineum,
external genitalia and perianal region. It is a form of necrotising fasciitis. There is a mixed
infection of aerobic and anaerobic bacteria in a fulminating inflammation of the subcutaneous
tissues, which results in an obliterative arteritis of the arterioles to the scrotal skin that in turn
results in gangrene.
E) True.

(29). A 50-year-old male present with fever, chills and rigors. Ultrasonography shows pyonephrosis
Caused by an obstructing 2 cm calculus at the pelvi-ureteric junction. His serum creatinine is 2.5 mg/dl
(Normal value <1.2 mg/dl). He was started on intravenous antibiotics. What is the most appropriate
management? (SBA)
A) Extracorporeal shock wave lithrotripsy
B) Percutaneous nephrolithotomy
C) Percutaneous nephrostomy
D) Pyelolithotomy
E) Ureterorenoscopy and lithotripsy

Answer ( C )
(43) A 40 year old male involved in a high speed automobile collision presents to the emergency
department. primary survey is done and life threatening injuries are excluded. He has a stable pelvic
fracture. There is blood at the urethral meatus and a scrotal hemotoma. Rectal Ex shows a high riding
prostate.. what is the most appropriate next Investigation? (SBA)
A) CT KUB
B) Endorectal ultrasonography
C) IVU
D) Retrograde urethrogram
E) USS

Pelvic fractures should be easily identified if ATLS guidelines are followed (i.e. clinical palpation and
compression of the pelvic brim from sacroiliac joint to pubic symphysis, and a routine chest and pelvic
radiograph for any blunt injury in a patient unable to walk). Clinical examination may reveal
instability. Any instability felt indicates the presence of major pelvic fracture, associated with
lifethreatening blood loss, and requires appropriate measures. The absence of clinical instability does
not, however, preclude an unstable pelvic fracture. Onethird of such trauma victims with pelvic
ring fractures present with circulatory instability on arrival. Inspection of the skin may reveal
lacerations in the groin, perineum or sacral area, indicating an open pelvic fracture, the result of gross
deformation. Evidence of perineal injury or haematuria mandates radiological evaluation of the
urinary tract from below upwards (retrograde urethrogram followed by cystogram or CT cystogram
and an excretory urogram, as appropriate) when the physiology allows. Inspection of the urethral
meatus may reveal a drop of blood, indicating urethral damage.
Inspection of the anus may reveal lacerations to the sphinc ter mechanism. Rectal examination may
reveal blood in the rectum and/or discontinuity of the rectal wall, indicating a rectal laceration. In
male patients, the prostate is palpated; a highriding prostate indicates a complete urethral avulsion.
A full neurological examination is performed of the perineal area, sphincter mechanism and femoral
and sciatic nerves. ( Bailey and love 27E)

So the answer is (D)

2016 November
(11) Regarding BPH
A) Commonly convert to the malignancy
B) 5 α reductase inhibitors relived symptoms within weeks
C) Can leads to bilateral hydronephrosis
D) Laser therapy is safer than TURP for larger prostates
E) Occurs in the central zone of the prostate
Answers:
A) false.
B) false.
C) True
D)
E) false.
(12) 36 years old male presents with recurrent bilateral uric acid urinary calculi. Which of the
following can be used to reduce recurrent attacks?
A) Allopurinol
B) High dose of Vitamin C
C) Hydrochlorothiazide
D) potassium citrate
E) sodium bicarbonate

Recurrent infection
 Recurrent infection may be associated with an underlying predisposing cause or may be a
result of bacterial resistance.
 In healthy women, infection after intercourse can occur without any demonstrable
abnormality of the urinary tract.
 Repeated attacks of UTIs in women, or a single attack in a man or a child of either sex, should
always be followed by investigation to discover and treat the cause; sometimes, however,
no cause can be found.
 Asymptomatic bacteriuria is common & investigation may fail to demonstrate any underlying
cause.
Answers:
A) True
B) False
C) False
D) True
E) True

(38) 26 year old scrotal swelling for 3 weeks. USS confirmed solid mass arising from testis. Next
appropriate management option is, (SBA)
A) Testicular biopsy
B) AFP and beta hCG level
C) Inguinal orchidectomy
D) MRI scan
E) ……..

Answer (B)
When managing a testicular tumor initial Investigation of choice is USS. On Examination they are felt
as a non tender mass attached to the testis, hard in consistency and can get above. A USS alone is
enough to diagnose a testicular CA. It is never biopsied trans scrotally as seedings may occur to the
skin. AFP and hCG should be done as tumor markers before Surgery. These are used to detect
recurrences and incompletely treated tumors or mets. Surgery of choice is inguinal orchidectomy.
This will prevent seedings. Tumors can metastasize to para aortic lymph nodes and the lungs. Lymph
spread is commoner than haematogenous. All patients need CT abdomen and chest to detect mets
but this is usually done after the Sx as even metastatic ones need orchidectomy. Testicular tumors
without scrotal invasion will not cause inguinal lymphadenopathy as the testis does not drain to those
nodes. Most tumors respond to chemotherapy very well where as seminomas respond to
radiotherapy as well . Most common type of testicular tumor is the germ cell tumor. Ex are seminomas
and NSGCT such as yolk sac tumors ( AFP) , Chorio carcinoma ( hCG) and teratomas. Interstitial tumors
are sertoli cell and leydig cell tumors. Leydig cell tumors have masculinizing effects where as sertoli
cell tumors feminize.

(39) Female loin pain and haematuria diagnosed to have 1.6 cm calculus in renal pelvis. Which of the
following investigations give most information?
A) IVU
B) NCCT
C) Contrast spiral CT of KUB
D) Xray KUB
E) ……..

Answer (B)
Renal calculi management
 Best investigation to diagnose renal or ureteric calculi is CT KUB.
 It can detect both radiolucent and radiopaque calculi.
 Renal calculi may be completely asymptomatic.
 Can present with loin pain and macroscopic haematuria.
 Can present with recurrent infections.
 May be incidentally detectred.
 USS KUB can visualize renal calculi.
 Asymptomatic calculi can be monitored carefully.
 Symptomatic ones need interventions.
 If the calculus is large and asymptomatic it needs active monitoring.
 Check wether the patient develops symptoms or not.
 Do annual X ray KUB.

1. ECSWL-
 If;
i. Calculi >5mm and less than 2cm- In sirs note ( Bailey says less than 1.5cm )
ii. Absence of active infection
iii. No coagulopathy
iv. Cant do during pregnancy
v. May not be effective in obese individuals as sound waves will not travel
vi. Cant do in mid and lower ureteric calculi and bladder calculi.

 1st line treatment if no contraindications but needs intervention.


 Can use for stones in pelvicayceal system and upper ureters.
 Can cause complications such as ureteric obstruction by calculi, infection,
haematoma and renal parenchymal injury.

2. Percutaneous nephrolithotomy
 For large stones more than 2 cm and also for staghorn calculi this can be used.
 Can cause bleeding, infections , renal parenchymal damage and damage to other
organs.

3. Pyelolithotomy, nephrolithotomy - If stone is large and cannot be removed by PCNL this is


used.

4. Flexible ureteroscopy and laser


 Can be used to Tx even staghorn calculi, calculi resistant to ECSWL and calculi not
visualized for ECSWL.
 Basically this is a method that can be used for any type of stone.

 MCQ point- Whenever an infection occurs in an obstructed kidney the obstruction should be
relieved immediately. This is commonly done by percutaneous nephrostomy. The other
option is ureteric stenting. But nephrostomy is usually better in the acute setting.

(40) A 65-year-old man with voiding LUTS presented to the surgical casualty. On examination, there
is a hard prostate on DRE. His PSA was 65ng/l. On bone scan, two lytic lesions were found in the iliac
crest and lumbar vertebra. What would be the best management for this patient?
A) androgen ablation therapy
B) oral bicalutamide
C) radical prostatectomy
D) primary radiotherapy
E) TURP

Answer (E)
Treatment of prostate CA depends on Age, life expectancy, Co morbidities, stage of the disease,
patient’s choice and Gleasons score.
Stage Ia- if < 70 years- curative treatment
If > 70 years – Active monitoring
Stage Ib- curative treatment
Stage II – curative treatment
Stage III / metastasis- if LUTS + —-> channel TURP and hormonal treatment.

2015 November
(06) 70 year old male presented with progressive difficulty in passing urine of 6 months duration.
Which of the following features are suggestive of carcinoma of prostate?
A) Haematuria
B) Bachache
C) Irregular hard prostate
D) Loss of weight of 5 kg
E) Palpable bladder after micturition
Prostate Carcinoma
 Very common among elderly. Family Hx is a major risk factor.
 Usually occurs in the peripheral zone of the prostate.
 They are adeno carcinomas.

Clinical features include -


 History - Incidental finding on removed prostate, LUTS, Bone mets with persistent back pain,
cord compression and LL neurological weakness, LOA,LOW, Anaemia or pancytopaenia,
haematospermia, Renal failure due to B/L ureteric compression, Hypercalcaemia.
 Examination - Obliterated median groove, Mucosa not freely mobile over the prostate, Hard
nodule, Asymmetrical lateral lobe enlargement

Investigations
 PSA- 1st Ix to be done. Elevated PSA may also occur in BPH, DRE, Prostate biopsy, Urethral
catheterization, prostatitis etc. PSA > 10 suggests possibility of mets.
 If the PSA is high the next Ix is USS guided prostate biopsy (TRUS biopsy). 6 per lobe is taken.
 If biopsy is positive next ideally a pelvic MRI should be done to assess local extent of the
disease.
 Bone scan should be done to detect bone mets.
Treatment of prostate CA
 Most tumors are very slow growing.
 watchful Waiting is done for patients with a Gleason score of 6 or less, Multiple comorbities
and expected life span < 10years with early prostate CA (T1 and T2).
 If not meeting these criteria Aggressive Tx is needed.
 For T3 tumors radical prostatectomy is an option. Radical radiotherapy can also be used as a
curative option if unwilling or unfit for Sx.
 For T4 and metastatic tumors Sx is of no use. Then need Sx and Medical Mx by hormonal
ablation. As Sx B/L orchidectomy is done.
As drugs androgen antagonists such as flutamide, bicalutamide, cyproterone are used. Also
GnRH analogues, Oestrogen Tx can also be tried. Life expectancy after hormonal ablation is <
6yrs.
Answers:
A) false. No hematuria. Hematospermia occurs.
B) True. Due to bone metastasis
C) True
D) True. Cancer cachexia
E) True

(22) 10-year-old boy presents with right sided testicular pain for 4 hours duration. There is no history
of fever. His right testis is extremely tender. Left testis is normal. What is the most appropriate next
step in management? (SBA)
A) Do an ultrasound scan
B) Start antibiotics
C) Scrotal support
D) Wait for 6 hours
E) Do immediate surgery
Answer : (E )

(42) A 35-year male presented with left side colicky abdominal pain and in distress. USS revealed a
10 mm stone in mid ureter & mild dilated ureter with moderate hydronephrosis. What is the best
management option?
A) Domiabasket
B) ESWL
C) ISWL
D) Pyelonephrostomy
E) Ureterolithotomy

Answers ( B )

Ureteric calculi
 Present with loin to groin pain, haeamaturia, vomiting, colicky pain, sweating etc.
 Management depends on following factors.
1. If serum creatinine is high that indicates the patient is going into renal failure. This needs
urgent surgical intervention.
2. Obstructed infected kidney needs urgent intervention.
3. Stones more than 5mm in diameter and are in the upper ureter are unlike to pass without
intervention.
4. Obstruction of a single kidney or bi lateral ureteric obstruction.

Interventions
1. Upper ureteric- Ureteroscopic laser therapy, ECSWL
2. Mid ureteric- Ureteroscopic laser therapy, Ureteroscope and ISWL
3. Lower ureteric- Ureteroscopic laser, ISWL or domia basket extraction (This has been mostly
mentioned for lower ureteric stones in bailey. Not for mid ureter. Ureterolithotomy may be used
if all other methods fail.

Medical expulsion therapy


1. Give diclofenac sodium as the analgaesic. May need suppositories in the acute setting as
patient may be vomiting. Diclofenac K may be better due to its quick action. Don’t take opiates
as 1st line as it can cause addiction.

2. Tamsulosin- Relaxes ureteric smooth muscle. Reduces the pain and helps the stone to pass
out.Other drugs such as calcium channel blockers and steroids can be used as alternatives for
this purpose.

3. Advice to take an adequate water intake. Not increased water as it can overload the kidneys.

4. Do USS KUB ( To look for absent urinary jet which means the kidney as shutdown itself) , X ray
KUB ( Cant do immediately. Needs bowel preparation ideally for 3 days. But here we do
overnight bowel preparation with dulcolax and fasting and then give dulcolax sup in the morning
and do Xray) , UFR ( pus cells and red cells) and serum creatinine to assess the patient.
5. If uncomplicated once symptoms resolve discharge. Advice to do excercises such as skipping
to aid in stone expulsion. Advice to avoid oxalate containing food such as spinach and tomato.
Optimize Ca intake.

6. Advice to seek immediate medical attention if develops fever without any obvious cause as it
can be due to an infection.

7. Repeat xray KUB in one mnth. Intervene if stone not expulsed.

8. Urate stones need allopurinol and Na HCO3 to alkalinize urine. This will reduce stone
formation Urate stones and xanthine stones are radiolucent. Others are radio opaque

(43) 16-year-old boy presented with inguinal scrotal lump which is irreducible during past 6 months.
He has a mild discomfort, otherwise he is healthy. What is the appropriate next step in the
management?
A) Early elective surgery
B) Bed rest, analgesia, ice pack to scrotum
C) Gentle manual reduction
D) X-ray abdomen to exclude obstruction
E) USS for exclude strangulation

Answer ( A)

The description is suggestive of Inguino scrotal hernia. Since this has been chronic and no acute
complications can go for early elective surgery. No need do expose to un unnecessary radiation by
doing x ray.

2015 May
(04) The complications of bladder outflow obstruction include
A) Bladder calculi
B) Bladder diverticuli
C) CKD
D) Hypercalciuria
E) Recurrent UTI
Answers:
A) True
B) True
C) True. Hydronephrosis —->obstructive urophathy —-> Chronic renal failure
D) False
E) True. due to acute urinary retention

(23) A 45 year old man presented with urinary frequency & urgency for 1/52. He is a smoker –
10/day. On examination he is afebrile & no flank tenderness. UFR – 60-70/hpf, pus cells 10/hpf &
U.culture – sterile. Most appropriate management
A) Cystoscopy
B) Early morning urine for AFB
C) IVU
D) NCCT KUB
E) Xray KUB

Answer: ( A)

(33) 75 year old man was found to have opacity in renal area during routine checkup. He has no
chronic lower bacvkach. USS shows 8mm size calculus in lower calyx of R/kidney. What is the best
management option?
A) Diuretic therapy
B) ECSWL
C) Percutaneous nephrolithotomy
D) Ureteroscopic stone extraction
E) No intervention is needed

Answer: ( E )
Nephrolithiasis is occasionally an incidental finding in patients who are asymptomatic. There is an
appreciable risk that a previously asymptomatic stone will become symptomatic with expectant
therapy (49 percent at five years in one study). Expectant therapy may be a reasonable approach in
asymptomatic patients with small, non-infected calculi, without evidence of obstruction. However,
certain asymptomatic patients, depending upon their occupation (airline pilots, frequent business
travelers) or complexity (neurologic disease, anatomic abnormalities of the urinary tract, such as
urinary diversion or solitary kidney), should undergo evaluation and treatment to reduce the risk of
recurrent stone formation or growth of existing stones. Prophylactic stone removal may also be
considered in high-risk patients.

2014 November

(13) Staging of prostatic CA?


A) Bone scan detects mets 3 - 4 months before X - ray
B) ….
C) …..
D) PSA > 20 suggestive of distal metastasis
E) PSA is specific for prostatic carcinoma

Answers:
A). True
B).
C).
D). True- PSA > 10 suggests possibility of mets.
E). False- Elevated PSA may also occur in BPH, DRE, Prostate biopsy, Urethral catheterization,
prostatitis etc.
(40) An otherwise healthy adult is admitted with ureteric colic is found to have 9mm stone In the
ureter at the level of sacral vertebrae.patient is pain free after analgesia. Renal functions are
normal. There is no evidence of sepsis but there is evidence of moderate hydronephrosis on USS.
WOF is the best management option?
A) medical expulsive therapy
B) extracorporeal shock wave lithotripsy
C) Ureteroscopy and ICSE
D) Nephrostomy
E) open ureterectomy
Answer ( C )

(41) A 76yr old male with dysuria,urine analysis,pus cells,red cells.PSA 23ng/dl.examination
normal.most appropriate management?
A) prostate biopsy
B) bilateral orchidectomy
C) Androgen antagonist treatment
D) commencement of antibiotics after sending urine culture & ABST
E) TURP

When investigating for prostate cancer, PSA- 1st Ix to be done. Elevated PSA may also occur in BPH,
DRE, Prostate biopsy, Urethral catheterization, prostatitis etc. PSA > 10 suggests possibility of mets.
If the PSA is high the next Ix is USS guided prostate biopsy ( TRUS biopsy). 6 per lobe is taken.

Answer ( A )

2014 May
(13) 24 years old male presented with acute onset scrotal pain
A) Surgery is not beneficial when patient presents after 24 hours in torsion
B)
C) Doppler use reliably excludes the necessity for the surgery
D) Fixation of the contra lateral testis is done
E) If it is due to a trauma always need a surgery
Answers:
A) True. If the torsion occurs within 4 hours the salvage rate is 96%. If untwisted after 24 hours
only 20% is salvageable.
B) ….
C) False
D) True
E) False

(14) Acute scrotum


A)
B)
C) Due to torsion surgery is done after Doppler USS
D) Due to trauma should always be operated
E) As a result a epididymitis common in elderly

Answers:
A)
B)
C) false. Even though the classical picture is not there if someone presents with sudden on
set testicular pain without a demonstrable cause it is managed as testicular torsion until
proven otherwise. So no investigations are done.
Urgent surgical exploration of the testis is done as gangrene may occur within
4hours .
D) False
E) True- In young sexually active men, the most common cause of epididymitis is now Chlamydia
trachomatis, but gonococcal epididymitis is still occasionally seen.
In older men with bladder outflow obstruction, epididymitis may result from a urinary infection.

(40) A 65 years old male presented with lower urinary tract voiding symptoms. Residual volume =
150ml, Prostate volume = 48ml, RTF -normal, DRE - enlarged prostate but benign, Normal PSA
levels, urine flow = 10ml/s. What is the best medical option?
A) Life style modification
B) Alpha blockers + fenesteride
C) Prostate biopsy
D) TURP
E) Immediate catheterization to reduce residual volume

Answer ( B )

(41) A 25 years old male with left loin pain and fever of 100F. PR 130/bpmin,BP 100/65 mmHg and
marked left loin tenderness. USS -hydronephrosis and proximal hydroureter. WOF is the most
appropriate next step in Management
A) Dormia basket extraction
B) Percutaneous nephrostomy
C) Start broad spectrum antibiotic after U.culture
D) Uretero lithotomy
E) Prepare to insert an IC tube

The description given is suggestive of a infected obstructed system.Have to relieve the obstruction
via a percutaneous nephrostomy.
Answer ( B )
2013 November Colombo
(02) Regarding urethral injuries
A) …….
B) Terminal haematuria is a feature
C) Posterior urethral injuries are associated with pelvic fractures
D) Extravasation of urine up to mid-thigh is a feature
E) Investigation of choice is ascending urethrogram

Injuries to the male urethra


The commonest cause of urethral trauma is that which occurs at the time of instrumentation,
either with a catheter or with a cystoscope.
Prevention is therefore the most important issue here, with the need for careful urethral
instrumentation being paramount.
Typically, when such an injury does occur it tends to happen either where the urethra bends or
where it narrows.
Such injuries initially result in bleeding and typically heal by scarring, which then results in a
urethral stricture.
The commonest sites of such strictures are the submeatal area, the bulbar urethra or the
region of the membranous urethra.
Injuries following external trauma are much less common, but require appropriate and prompt
management.
The most common sites for such injury are the bulbar urethra and the membranous urethra.

Bulbar urethral injury


● Suspect urethral injury after blunt perineal trauma when the man cannot void, when there is
perineal bruising and when there is blood at the urethral meatus
● Diagnosis is made by urethrography using water-soluble contrast
● The safest initial management is to insert a suprapubic catheter
● Beware of urinary extravasation and sepsis in the perineal haematoma
● Delayed urethroplasty is the preferred definitive management of complete disruptions

Pelvic fracture urethral disruption injury


● Suspect the diagnosis in cases of pelvic fracture, when the patient has not voided and when there
is blood at the urethral meatus.
● If the diagnosis is suspected, a water-soluble urethrogram is needed to confirm the diagnosis.
● Initial management is insertion of a suprapubic catheter.
● Immediate surgical exploration is needed if there is coexisting rupture of the bladder.
● Delayed anastomotic urethroplasty is the preferred definitive management.

Answers:
A)
B) False
C) True
D) True. Extravasation of urine can occur with extraperitoneal rupture of the bladder, pelvic
fracture urethral disruption or in the rare cases where the level of the disruption is the prostatic
urethra. Urine extravasates in the layers of the pelvic fascia and the retroperitoneal tissues.
Treatment is by suprapubic cystostomy. In rare cases where the extravasation persists despite the
suprapubic tube, drainage of the retropubic space and definitive repair of the urethral, prostatic or
bladder rupture is required. ( Bailey and love 27E page 1481)
E) True

SBA (12) 10 year old boy with 2 hour history of lower abdominal pain, abdomen soft, right testes
elevated with scrotal skin discoloration, Immediate management?
A) IV fluids
B) Reassurance
C) Antibiotics IV
D) Surgical exploration
E) USS of scrotum

Answer ( D)

(13) 16 year old boy, 6 hour left groin pain. On Examination tender lump in left inguinal region an
empty hemiscrotum?
A) Acute epididymoorchitis
B) Strangulated hernia
C) Torsion of testes
D) Torsion of cord
E) …….

Answer( C ). The description given is Suggestive of Testicular torsion ( so you need to be through
with common causes of testicular swelling, how to differentiate each and management. What you
need to know is given in sirs note and a short note from Bailey and love is given in the previous
mcqs discussed on testicular torsion).

2013 – Eastern
(11) High pressure chronic retention
A) Painful
B) Presented with nocturnal enuresis
C) B/L hydronephrosis can occur
D) Causing diuresis following catheterisation
E) Treat effectively with alpha blocker

In chronic retention there is no pain. These patients are at risk of upper tract dilatation because of
high intravesical tension – they require urgent urological referral. Men with impaired renal
function may develop postobstructive diuresis after catheterisation. Such men need careful
monitoring, with replacement of inappropriate urinary losses by intravenous saline; they are
also at risk of haematuria as the distended urinary tract empties. Often it is several days before
full renal recovery occurs.

Answers:
A) False- painless
B) True
C) True
D) True- Post obstructive diuresis is due to three mechanisms.
 High fluid volume in body
 Increased BU ( osmotically active)
 Acute tubular necrosis
Post obstructive diuresis is detected if hourly UOP >200ml.

E) False. In high pressure chronic retention, catheter/ TURP is treatment of choice. No


pharmacological management.

(12) Urethral tumour


A) Occur in female
B) Increased by cigarette smoking
C) Often multifactorial
D) Squamous carcinoma is common
E) Intravenous BCG induces the recurrence

Carcinoma of the urethra


This occurs twice as often in women as in men. Whether a caruncle can become malignant is
disputed, but caruncles and tumours often occur close together. Malignant swellings of the
urethra feel harder than benign ones. Treatment by radiotherapy or radical surgery is often
ineffective. The overall prognosis is poor.

Neoplasms of urethra in males:


Polyps are a relatively common finding in the prostatic urethra, where they may result from chronic
infection. Genital warts acquired by sexually transmitted infection are sometimes found in the
anterior urethra as an extension of warts on the skin of the glans penis. Angioma of the urethra is a
very rare cause of urethral bleeding. Bloody urethral discharge without infection should raise the
suspicion that the patient has a urethral tumour, although such tumours are rare. Multifocal
transitional cell cancers of the bladder are sometimes associated with tumours in the prostatic
urethra and occasionally more distally. Although superficial and susceptible to local ablation by
diathermy or laser, they are associated with a tendency to distant spread. Squamous carcinoma can
develop in an area of squamous metaplasia sometimes seen with a urethral stricture. It carries a
poor prognosis even if the patient is treated by radical surgery.

Answers:
A) True
B) True
C) True
D) True
E) False- Intravesical BCG is given for bladder CA

(31) A man fallen from pit wall and had pubic ramus injury and found to have blood in the urethral
meatus. Most probably injured urethra (SBA)
A) Bulbar urethra
B) Membranous urethra
C) Penile urethra
D) Anterior urethra
E) Posterior urethra

A common mcq and viva question. Injuries following external trauma are much less common,
but require appropriate and prompt management.
The most common sites for such injury are the bulbar urethra and the membranous urethra.

Posterior urethra is divided in to prostatic and membranous urethras. Commonest part to get
injured is membranous urethra because it is fixed.
Answer: ( B )

(34) A 65 years old man diagnosed to have prostatic CA, which is restricted to prostate. Best mode of
treatment
A) Radical proctectomy
B) Radiotherapy
C) Chemotherapy
D) Immunotherapy
E) B/L orchidectomy

Answer: ( A )

(35) 56 years old gentleman presented with right sided scrotal lump. It is trans illuminated and can
get above and can’t differentiate testis from that , ( SBA)
A) Varicocele
B) Hydrocele
C) Testicular tumour
D) Cyst of the cord
E) Haematocele

Examination of a scrotal swelling should be undertaken in both the upright and supine position. The
examiner should ask themselves a series of questions.
1. First, is it possible to get above the swelling to palpate a normal cord? For instance, if it is not
possible to achieve this, the swelling may represent an inguinal hernia that has entered the
scrotum.
2. Second, is the swelling primarily testicular or epididymal or is it enclosing both of those
structures?
3. Thirdly, does the swelling transilluminate? In almost all cases of scrotal swelling an
ultrasound is a useful adjunct, both in clarifying the nature of the swelling and in determining
whether the testis itself is diseased. ( Bailey and love 27E page 1503)
(remembering this type of method will help you in mcqs and in short cases.)

A) Varicocoele is a common condition and 90% are left sided. Development of a left-sided
varicocoele in later life may indicate the presence of a renal tumour. They are usually
asymptomatic and as such rarely need treatment. First-line treatment is embolisation in
symptomatic cases. Varicocoeles often recur, even after surgical treatment. The association
of varicocoeles with subfertility is controversial.
B) A hydrocoele is a collection of fluid within the tunica vaginalis. Primary hydrocoeles surround
the testis and transilluminate brigh.Ultrasound examination is valuable, especially when the
testis and epididymis are impalpabe. Hydrocoeles can be treated conservatively unless they
are large and symptomatic. Surgery is the mainstay of treatment. Testicular malignancy is an
uncommon cause of hydrocoele that can be excluded by ultrasound examination.
C) Testicular tumour is a solid testicular lump that cannot be felt separately from the testis may
be a malignant tumour. Lymphatic spread is to the para-aortic lymph nodes. Ultrasound is a
mandatory investigation in all cases of suspected testicular tumour. Tumour markers (AFP and
hCG) should be measured prior to orchidectomy.
D) A cyst of the cord is a smooth oval swelling that lies above the testis near the spermatic cord,
which is liable to be mistaken for an inguinal hernia. The swelling moves downwards and
becomes less mobile if the testis is pulled gently downwards.
E) The testis can be damaged either by blunt or by penetrating trauma. Injuries can range from
simple bruising, through significant intratesticular haematomas to rupture of the tunica
albuginea, with very significant collections of blood within the tunica vaginalis
(haematocoele).

Answer ( B ).
Trauma Surgery
2020 June
12) A 19 year old otherwise healthy rugby player was brought to the accident and
emergency department following a sports injury. He complained of a
central chest pain and examination he was dyspnoeic and tachypnoeic . His JVP was prominant.
On auscultation there was vesicular breathing with reduced heart sounds.
Mx in the A&E includes
A. FAST
B. Intercostal tube drainage
C. Needle thoracotomy
D. Predicardiectomy
E. Pericardiocentesis

Answers – T F F F T

Cardiac tamponade –
- Becks triad. Hypotension, raised JVP and distant muffled heart sounds.
- Can diagnose using FAST scan. Needle pericardiocentesis can be done in very unstable patients.
But if clotted can result in a dry tap. Best is to do a surgery.

13) regarding abdominal trauma


A. CECT abdomen is done in unstable patients.
B. CXR showing air under diaphragm in a patient with penetrating injury need immediate
laparotomy.
C. patient with liver laceration need laparotomy
D. Management depends on the mechanism of injury
E. Suspected small bowel perforation can be managed conservatively if the patient is stable

Answers – F T F T F
- Any abdominal injury can cause unexplained hypotension due to bleeding.
- All patients with suspected abdominal trauma should be checked by a FAST scan. It can detect
>100ml of fluid in the abdomen.
- The drawbacks are it is not good at detecting retroperitoneal injuries, may miss hollow viscus
injuries, affected by obesity and bowel gas and operator dependent.
- If the patient is hemodynamically unstable and abdominal injury is suspected 1st the patient
should be stabilized, a FAST scan done and urgent laparotomy should be done.
- In patients who are stable the best Ix to diagnosis is CECT abdomen. This is a must.
- Diagnostic peritoneal lavage is of minimal use as it is invasive and simpler methods such as
FAST scan are available.

E - Small bowel injuries need urgent repair. Hemorrhage control takes priority and these wounds can
be temporarily controlled with simple sutures
14) A 63 year old female presented with acute onset right knee joint pain.on examination temperature
is 38.8.knee joint is red, swollen and tender. what investigations will be useful in the management?
A. iv steroids
B. flucloxacillin
C.
D. Joint fluid aspiration for cytology
E. Serum rheumatoid factor level

Answers – F T _ T T
This is septic arthritis and should come under orthopedics.
Septic arthritis
- Presents with fever, pain, swelling and diminished movement of the affected limb.
- Commonest organism is Staph aureus. Streptococci and E coli can also cause.
- Commonly affects knee and hip joints (In neonates).
- Most common route of spread is hematogenous but may spread from a contiguous focus such
as a local wound or post-surgery.
- Immediate FBC, CRP and blood culture should be done.
- Before initiation of antibiotics USS guided aspiration of the joint fluid should be done and sent
for cell counts, gram stain and culture.
- After aspiration broad spectrum antibiotics should be given.
- IV given for 2 weeks and oral for 4 weeks. Usual Tx duration is 3- 6 weeks.
- Arthrotomy and drainage or arthroscopic drainage is a must.
- IV antibiotics should be given 1st and then switched to oral antibiotics.
- IV Flucloxacillin + Gentamycin or cephalosporin are suitable antibiotics.
- Immunocompromised states, joint disease such as rheumatoid arthritis, IV drug use are risk
factors.

Other inflammatory conditions can also cause a raised cell count and crystals may be seen in
infected joints as well as gout and pseudogout.

18) T/F regarding management of air way


A. fitted mask should cover bridge of the nose and mouth
B. elderly patients should not bag and mask ventilate
C. laryngeal airway is a supraglottic airway
D. the size of the oropharyngeal airway is from tragus to angle of mandible
E. The triple airway manoeuvres are chin lift, head tilt and jaw thrust

Answers – T T T F T
B - It is important to predict which patients may be difficult to ventilate. Several acronyms have been
formed to help predict who will be difficult to ventilate.
MOANS stands for "mask seal, obesity, age (elderly), no teeth, stiffness." BONES stands for "beard,
obese, no teeth, elderly, sleep apnea/snoring." These patients may be particularly difficult to ventilate
and may require the use of a supraglottic airway to improve chances of ventilation
38) 60 years old woman presented with right hip joint pain & difficulty in walking after the falling
from the bed. She diagnosed patient with breast carcinoma & now on adjuvant therapy. What is the
initial management of this patient?
A. USS of the right side hip joint
B. X ray of bilateral hip joint
C.
D. CT scan of right hip joint
E. MRI scan of hip

Answer – B

- When abnormal bone fails under normal load this is referred to as a pathological fracture
- The bone may be weakened by a primary bone tumour, secondary metastatic deposits,
haematological malignancy (myeloma, lymphoma, leukaemia), osteomyelitis and metabolic
bone disease (osteomalacia, Paget’s disease, osteoporosis).
- A pathological fracture should be suspected if the history is not consistent with the severity
of the injury.

2019 Nov
12) Chest injuries that respond well to ventilation
a) Cardiac tamponade
b) Flail chest
c) Open pneumothorax
d) Pulmonary contusion
e) Tension pneumothorax

Answers – F x F T F
Refer the common MCQ theory guide –chest wall injuries. You should know how to identify and how
to manage them.
a) Management of Cardiac Tamponade includes; (Short notes in Surgery by Dr.S.Sri Shankar –
Section 1 - Page 8 / B & L – 911)
 Needle pericardiocentesis
 Rapid volume resuscitation
 Definitive treatment - sternotomy / left thoracotomy

b) Management of Flail chest includes; (Short notes in Surgery by Dr.S.Sri Shankar – Section 1 –
Page 8 / B & L – 368)
 Mild cases - Analgesics- Intercostal block, Intra pleural local analgesia (if IC tube in situ), Oxygen ,
Physiotherapy
 Moderate cases - Oxygen, IC tube insertion
 Severe cases with respiratory compromise - Mechanical ventilation with positive pressure
c) Management of Open pneumothorax includes; (Short notes in Surgery by Dr.S.Sri Shankar –
Section 1 - Page 9 / B & L – 368)
 Close the wound with sterile occlusive plastic dressing
 Paste only on 3 sites- Flutter type valve
 Chest drain - not through the wound Larger tube(28FG or bigger)
 If lung does not reinflate, consider low pressure suction or 2nd drain
 Wound debridement and closure
 Physiotherapy and early mobilization

d) Management of Pulmonary Contusions include; (B & L – 371)


 Mild contusion - oxygen administration, pulmonary toilet and adequate analgesia
 Severe cases - mechanical ventilation is necessary
 Normovolaemia is critical for adequate tissue perfusion and fluid restriction is not advised

e) Management of Tension Pneumothorax includes; (Short notes in Surgery by Dr.S.Sri Shankar –


Section 1 - Page 8 / B & L – 367)
 Needle thoracotomy - insertion of large bore cannula / needle into the 2nd intercostals space in the
mid clavicular line
 Intercostal tube insertion through the fifth intercostal space in the anterior axillary line.

2019 March
10) In a triage,
A. ATLS guidelines are used to sort victims
B. Children are given priority
C. Individual should get best care
D. Done by Senior experienced person
E. Done in daily hospital admissions

Answers – F T F T F

Triage
- is the cornerstone of the management of mass casualties.
- aims to identify the patients who will benefit the most by being treated the earliest (greatest
good for greatest number )
- Triage should be undertaken by someone senior, who has the experience and authority to
make these critical decisions.
- triage needs to be undertaken in the field, before evacuation and at the hospital
- The assessment and restoration of airway, breathing and circulation are critical.
- Vital signs and a general physical examination should be combined with a brief history taken
by a paramedic or by a volunteer worker if one is available.
- Accurate documentation is important
- A rapid clinical assessment should be made taking into account the patient’s ability to walk,
their mental status and the presence or absence of ventilation or capillary perfusion
11) in ATLS adjuncts to primary survey are,
A. PR examination
B. BP measurement
C. CXR
D. FAST scan
E. Cervical X ray

Answers – F F T T T

Adjuncts to primary survey –


- ABG,
- ECG monitoring,
- FAST scan,
- SpO2,
- ETCO2,
- AP and lateral chest, neck and pelvic x-rays,
- Catheterization

31) 28year old man was admitted to the emergency unit following a fall from height with blunt
trauma to the right chest. PR-120, BP-70/50, Spo2-84%. Hyper resonance over right lung fields with
reduced breath sounds. Immediate management?
A. FAST scan
B. Wait till ICU available
C. chest tube
D. Needle thoracostomy
E. Urgent xray

Answer – D, this is tension pneumothorax. Therefore, relieve the pressure by needle thorcostomy.
Read previous explanation for the management.

40) A 30 year old man had sustained a flail chest after a blunt injury to chest following RTA. On
examination he is mildly dyspneic and maintaining normal SpO2 levels with O2 via nasal cannula. Chest
xray had found to have fractured ribs. No other injuries detected. What is the most appropriate
management of this patient?
A. Insert a IC drain
B. Give analgesia and chest physiotherapy
C. Plan a surgery to correct rib fracture
D. Apply chest corset and immobilize
E. Endotracheal tube insertion and ventilation

Answer – B, again very easy question, patient is maintaining saturation, so give analgesics and chest
physiotherapy.
2018 November
11. Spleenic Trauma,
a) Can occur with lower thoracic trauma.
b) Enlarged spleen has the same risk of rupture than the non enlarged one.
c) Intraarterial embohzation is a treatment option.
d) If stable, Can be managed conservatively.
e) Spelenectomy is done after giving immunization for capsulated organisms.

ANSWER – T F T T F
A – Spleen is covered with lower ribs and therefore injury to lower thorax can cause damage to spleen.

B – Enlarged spleen has a higher risk of rupture than a normal sized spleen.

C – Intra-arterial chemo embolization can be done by interventional radiologists

D – Stable injuries can be managed conservatively.

E – Spleenectomy done if the patient is unstable. So there is no time for the immunization. But before
the discharge patient should be vaccinated against capsulated organisms.

30. 50 year old male was knocked down by a van. He was conscious. There was bruising over right
hypochondrium. Blood pressure and Pulse rate are normal FAST scan showed fluid in the peritonium
with suspicion of LIver laceration. What is the best next step of the management?
a) Need ICU admission.
b) CECT abdomen.
c) Laproscopic exploration
d) Diagnostic Peritoneal Lavage.
e) Emergency laparotomy.

ANSWER - B
If the liver injury is suspected and the patient is stable CECT is done. If the patient is unstable surgery
should be performed.

39) . A 25 male motorcyclist was brought to the Accident Service Unit at a base hospital after a RTA
with a head injury: Impact on Left side of the head. After admission, Patient developed reduced level
of consciousness and GCS deteriorates from 13 to 7. Nearest Neurosurgica facility is located within
One hour range. What is the best management option you would consider to perform before
transferring the patient?
a) Intubate and ventilate.
b) Mannitol infusion.
c) Do a blurr hole in the left side.
d) Morphine infusion.
e) Analgesia for pain with a saline bolus.
ANSWER- A
First the airway should be stabilized. Since his GCS falls from 13 to 7 his airway is in danger and it
should be stabilized with intubation.

47. A 24 year old male patient was brought to the accidents Service unit after a road traffic accident.
After initial stabilization, patient was kept under observation. What is the earliest indicator to suspect
a hemorrhage?
a) Drop in diastolic blood pressure
b) Reduction in the urine output
c) Deterioration of the consciousness
d) Increase in heart rate
e) Worsening pallor

ANSWER - D

If a patient is going into shock the earliest indicator is increasing pulse rate.

2017 May
15) . Tension pneumothorax
a) Cause hyper-resonance of the opposite side
b) Causes distended neck veins
c) Causes mediastinal deviation to the opposite side
d) Causes obstructive shock
e) Is confirmed by a chest X-ray

ANSWER – F T T T F
A – Tension pneumothorax causes hyper resonance of the same side with absent breath sounds,
reduced expansion and reduced vocal fremitus of the same side. Trachea is shifted to opposite side.

B – Due to kinking of great vessels, the neck vessels can be distended.

C – In tension pneumothorax mediastinum is shifted to the opposite side.

D – Obstructive shock is resulted because the vessels in the thorax can get kinked.

E – Tension pneumothorax is a clinical diagnosis and should never wait for radiological confirmation

18. A 40 year old male is brought to the accident and emergency department following crush injury to
his lower abdomen from a collapsed wall. He is pale, restless, hypotensive and has blood at urethral
meatus. In this patient,
a) Application of a pelvic binder is a life saving measure
b) Two pink canulaes are inserted for fluid resuscitation
c) Uncross matched ‘O’ group blood is transfused when crystalloid resuscitation fails
d) Urethral catheterization is indicated
e) Urgent focused assessment with sonography in trauma(FAST) is required

ANSWER – T F T F T
A - Blood in the urethral meatus indicates a urethral injury. Most of the times it is associated with a
pelvic fracture. Application of the pelvic binder avoids damage to pelvic vessels which can result in life
threatening haemorrhage.

B – pink cannula is 20G and in trauma large bore cannula should be used ideally orange or grey.

C – If crystalloid resuscitation fails, because the patient is unstable, uncrossmatced O negative blood
can be given because it takes time to get crossmatched blood.

D – If a urethral injury is suspected urethral catheterization should not be done and suprapubic
catheterization should be done.

E – All patients with suspected abdominal trauma should be checked by a FAST scan. If the patient is
unstable first the patent should be stabilized and then FAST should be done.

27. A young male is brought to the accident and emergency department following an assault with a
blunt weapon to his head. Which of the following is the most important initial step in the management
of this patient?
a) Arrest scalp haemorrhage
b) Determination of the Glasgow coma score
c) Fluid resuscitation
d) Obtain a CT scan of the brain
e) Secure the airway

ANSWER - E
In any trauma patient we should follow the ATLS protocol and therefore always start with the airway.

2016 NOVEMBER
17) Indications for thoracotomy in a patient with blunt trauma to the chest
a. 200ml /hour for 4 hours
b. Persistent cardiac tamponade
c. Massive unilateral pulmonary contusion with hypoxia
d. Haemothorax of 1500ml of volume at IC tube insertion
e. Flail chest

ANSWERS – T T F T F
Indications for a thoracotomy,
 Initial draining of 1500ml
 Ongoing haemorrhage of >200ml/hr more than 3hours.
 In cardiac tamponade needle pericardiocentesis can be done to aspirate few ml and buy time
until thoracotomy is done to relieve tamponade and repair the heart.
 To control massive air leak
 To perform internal cardiac massage.
E - Flail chest is a clinical diagnosis. Treated with oxygen, analgesia and physiotherapy. If patient goes
to respiratory failure mechanical ventilation should be done.
C – Pulmonary contusion with hypoxia also occurs due to flail chest and management is same.

18. Which of the following are done during the primary survey?
a. ECG trace
b. Blood pressure and pulse measurement
c. Cervical spine stabilization
d. Examination of thoracolumbar spine
e. Ensuring adequate airway supply

ANSWER – T T T F T

A – Adjuncts of the primary survey – ABG, ECG, FAST scan, CXR, neck and pelvic xray, Catheterization.
Adjuncts of the primary survey also can be regarded as components of the primary survey.

B – This should be done under circulation.

C- In ‘A’ Airway management and C spine stabilization should be done. We assume that every trauma
patient has a spinal injury until proven otherwise.

D – In secondary survey we do log rolling and examine the spine.

E – ATLS protocol starts with the assessment of the airway.

44. A 26-year-old man admitted following a road traffic accident. He is dyspnoeic. Neck veins are
engorged. Trachea midline. Lungs normal. PR 136 bpm, BP 80/50 mmHg, RR 35/min. What is the
most likely condition?
a. Cardiac tamponade
b. Flail chest
c. Massive haemothorax
d. pulmonary contusion
e. tension pneumothorax

ANSWER – A

Features of cardiac tamponade,


Low BP ,Raised JVP ,Muffled heart sounds – Becks triad
It is not associated with mediastinal shift or reduced breath sounds.
ANAESTHESIA
Final MBBS MCQs (2016 November -2020 June)
{Annotations by Dulanja Perera}
2020 June
01.) Regarding complications following surgery
A. DIC can cause excessive bleeding from cannula site
B. Electrolyte disturbance result in post of confusion
C. Non bloody diarrhea excludes pseudomembranous colitis
D. Spinal anesthesia reduces the risk of constipation
E. Vomiting increases the risk of incisional hernia
Answers – T T F T T

A - Disseminated Intravascular Coagulation (DIC) is a consumptive coagulopathy where bleeding


occurs once the body has consumed its clotting factors.
It is clinically evident by excessive bleeding from cannula site.
Surgery is one of the many triggers for DIC. Trauma, pre-eclampsia, amniotic fluid embolism and viper
envenomation are some other triggers.

B - Among electrolyte imbalances that can cause post-op confusion, HYPONATREMIA is notorious. Do
you remember the MCQ in 2021 March paper regarding a patient with confusion following TURP? It
is a surgery which uses normal saline irrigation during the procedure and absorption of this fluid
through bladder mucosa can cause hyponatremia leading to post-op confusion especially if the
duration of the surgery is prolonged.

C - Pseudomembranous colitis caused by Clostridium difficile bacteria, mainly occurs following


treatment with antibiotics like cephalosporin’s and clindamycin generally presents with watery (Non-
bloody) diarrhea although in some cases feces can contain small quantities of blood. Therefore, the
presence of non-bloody diarrhea DOES NOT exclude the possibility of pseudomembranous colitis. This
condition is treated with either vancomycin or metronidazole.

D - This is self-explanatory. General anesthetics and muscle relaxants used in general anesthesia impair
gut motility causing constipation.

E - Another obvious statement. Anything that causes raised intra-abdominal pressure increases the
risk of incisional hernia.

02) Which of the following combinations are most appropriate for post-operative pain management
A. Epidural anesthesia- abdominal surgery
B. Lignocaine infusion - Laparoscopic surgery
C. Patient controlled analgesia - Day case surgery
D. Subcutaneous morphine- Carpel tunnel syndrome
E. Transverse abdominis plane block- Laparotomy
Answers - (T T T T T)

B. Lignocaine infusion is used as a pain management modality. Low dose lignocaine infusion (1.5-3
mg per kg per hour) blunts the inflammatory response by decreasing pain- causing cytokine
production.

19) Indications for CV line insertion in ICU,


A. inotrope support
B. Rapid infusion of crystalloids
C. Right heart pressure monitoring
D. Inotropes
E. TPN
Answer - (T T T T T)

44) 70 years old male has been given spinal anaesthesia for right side inguinal hernia repair. 2
minutes after the anaesthesia he complains of shortness of breaths & coarse voice. His upper limbs
are flaccid. What is the possible reason for that?
A. Anaphylaxis
B. Epidural analgesia
C. High spinal block
D. Hypoglycaemia
E. Local anaesthetic toxicity
Answer - C

45) 60 years old patient admitted to ICU with low blood pressure & tachycardia. She had a 3 days’
history of severe dysuria, high fever spikes. She was conscious but tachypneic & lungs were clear.
Her blood pressure was 80/40mmHg despite fluid resuscitation. What is the initial cardiovascular
drug you will give to maintain her blood pressure?
A. Adrenaline
B. Dobutamine
C. Dopamine
D. Noradrenaline
E. Vasopressin
Answer – D
- In this septic shock scenario, we have to employ a vasopressor in order to achieve a MAP of
65mmHg. First line pressor is Norepinephrine. Vasopressin is a second line option.
- Based on the pathophysiology of sepsis there is peripheral vasodilatation causing hypotension.
- That’s why a vasopressor like NE which has predominant alpha 1 agonist activity is preferred over
inotropes like dobutamine and dopamine which can complicate matters by causing cardiac
arrhythmias due to their preferred beta 1 agonist activity.
2019 Nov
2) 55 years old lady undergo mastectomy and axillary clearance, 8 hours following the procedure she
complains of severe pain at surgical site. What is /are acceptable for her pain management
a) Diclofenac Na suppository
b) Oral Gabapentin
c) Intramuscular Morphine
d) IV paracetamol
e) IV pethidine
Answers – (T F T T ?)

B. Gabapentin is indicated for neuropathic pain and it does not alleviate an acute pain like this.
E. Technically speaking pethidine injection can be given either intramuscularly or intravenously. The
only caution about IV pethidine is that it should be given as a slow injection in order to minimize the
risk of respiratory depression. In this regard E is a true response.
But quite often we see pethidine being administered intramuscularly. In my opinion, the examiner
might want to check if we know that IV is a route of administration for pethidine. However, since I
don't have 'parachiththa vijanana gnanaya' all I can say is I report you decide.

18) Rapid sequence induction


a) Done to prevent aspiration
b) Indicated in patients with delayed gastric emptying
c) Inhalation anesthetics are used for induction
d) Short acting muscle relaxants are used
e) Predominantly used for paediatric surgeries
Answers – (T T F T F)

C. 'Rapid' word in Rapid Sequence Induction(RSI) says it all. Since RSI is done for urgent indications, an
intravenous induction agent which acts quickly like etomidate or propofol is preferred over
inhalational agents.
D. Muscle relaxants with a quicker onset and offset of action in other words short acting agents like
suxamethonium and rocuronium are used.
E. RSI is safer in adults and it is not generally a popular mode of induction for children under 5 years.

41) 63-year-old patient diagnosed with pulmonary embolism on 5th day following total knee
replacement. He is hemodynamically stable. What is the best treatment option?
a) Dual antiplatelet
b) Placement of IVC filter
c) Therapeutic dose of anticoagulant
d) Thrombolysis.
e) Thrombolectomy
Answer – C
There is no wrong answer in the list. However, evidence shows that therapeutic anticoagulation is the
best treatment option in pulmonary embolism. Warfarin is started but until it takes effect in 3-5 days’
anticoagulation therapy is bridged with enoxaparin until target INR is achieved.
45) 60kg man with a 10cm laceration on his right forearm. What is the most suitable volume of local
anaesthesia for suturing the laceration
a) 18ml of 1% lignocaine
b) 18ml of 2% lignocaine
c) 6ml of 5 % bupivacaine
d) 9ml of 0.5 Bupivacaine
e) 9ml of 1% lignocaine

If you know the theory the rest is O/L maths. Since you're so good with such calculations I will break
down the calculation to the most simple form. At the end I will show you the short-method.
Theory- Maximum lignocaine dose for local anesthesia
Plain lignocaine – 3mg/kg
Lignocaine+Adrenaline – 7mg/kg
Since it is a forearm laceration we can use lignocaine with adrenaline. Therefore, the maximum dose
is calculated as 7mg/kg x 60kg = 420mg.
Commonly available lignocaine has a concentration of 2%(W/V). This means that each 100ml of this
solution contains 2g (2000mg) of lignocaine. Therefore, 1ml of the solution has 20mg of the anesthetic
agent(2000mg/100ml=20mg/ml)
Therefore, to obtain 420mg of the agent we need 21ml of the solution (420mg/20mg/ml = 21ml). But
this isn't in the answers. Mara wade apita une. Don't worry. We can handle this situation.
If we were to use plain 2% lignocaine the answer would be 9ml (3*60/20). This is also not in the
answers. Then the answer is 18ml of 1% lignocaine. If you can't figure out this take a piece of paper
and do the math.
Short method-
If you're using plain lignocaine volume of anesthetic solution required is calculated using the following
method.
Volume(ml) = Body Weight in kg ×3
20
Note – If you want to quickly check how many mg of a substance is in 1ml of solution when the
concentration is given as a W/V percentage simply multiply the percentage by 10.

46) 68-year-old diabetes patient was found unresponsive in ward with no breathing movements, he
was having a sinus bradycardia. What is the most important next step in management?
a) High flow O2
b) Measure the capillary blood sugar
c) Check carotid pulse
d) Obtain IV access
e) Intubate and ventilate
Answer – C

This is a question that reminds us of the first degree relatives of those who set these questions.
Whenever a known diabetic is found unresponsive there is no argument, we have to check CBS. As we
all have witnessed how much time it takes to get a CBS done in our ward set up. From the stem we
can gather that the patient is not breathing and has sinus bradycardia (The patient is probably
attached to a multipara monitor because nobody checks the pulse and say the patient has SINUS
bradycardia). Can't this patient have suffered from an MI? It is a possibility. So it is rational to check
for his carotid pulse which is the first step of BLS, because we need to initiate chest compressions. In
the mean time we can get a CBS done too and remedy that. According to chain of survival early
initiation of high quality CPR is crucial for better outcome.

2019 March
19) Which of the following surgeries are matched with the optimum anaesthetic procedure?
a) Evacuation of a thrombosed external pile - Local anaesthesia
b) Incision and drainage of paronychia - Brachial plexus block
c) Laparoscopic cholecystectomy - High spinal anaesthesia D
d) Lateral sphincterotomy - Spinal anaesthesia
e) Relocating a dislocated shoulder - IV sedation
Answers – (T F F T T)

B. A digital block is adequate for the drainage of paronychia


C. Lap chole is usually performed under general anesthesia. This is because creating a
pneumoperitoneum affects the movements of the diaphragm requiring intubation and ventilation.
However, there are centers that perform lap chole under spinal anesthesia. However, a high spinal
block may hamper intercostal muscles and even the diaphragm. So it is not safe to perform this surgery
with a high spinal block.

20) 50yrs old man with a hx of fever,vomiting,and abd pain for 2days duration present with pulse
110 BP 70/50 , tense tender abdomen and gas under the diaphragm in xray. low BP despite
adequate fluid resuscitation.which of the following T/F
a) corticosteroids have no place in initial management
b) low bp possibly due to reduced systemic vascular resistance
c) noradrenaline is the vasopressin of choice
d) septic shock is lickely
e) wbc 3500 /mm3 exclude sepsis
Answers – (T T T T F)

42) 60 year old man who has background of diabetes nephropathy, serum creatinine normal, is
planning undergo CECT abdomen. Which of the following would the best option to minimize
contrast nephropathy?
a) Normal saline
b) Post procedure haemodialysis
c) give post-procedural intravenous furosemide
d) Pre procedure IV NaHco3
e) Preoperative administration of NAC
Answer – A
Based on available evidence the best way to prevent contrast nephropathy is adequate hydration
with normal saline.

44) 50 year old male with ischaemic heart disease is undergo emergency laparotomy. His weight is
60 kg and pre surgical haemohlobin level is 9 g/dl. The expected amount of blood loss during first
hour is 1100ml. What is the best fluid to give?
a) 0.9% saline
b) 5% albumin
c) Hetastarch
d) Packed red cells
e) Hartmans
Answer – D

Normal blood volume of an average adult is 70ml/kg. So, the total blood volume of this patient is
around 4200ml. Expected blood loss in the first hour is 26% of his total blood volume which is
equivalent to a Class 2 hemorrhage. And to make matters worse his pre-op HB is on the low side.
Therefore, the best fluid to give him is packed red cells because no other fluid can maintain oxygen
carrying capacity.

45) A 70-year-old patient admitted to the ICU following laparotomy for intestinal obstruction. On
admission his arterial blood gas was; FiO2 - 40% pH : 7.25 (7.35 - 7.45) PO2 : 95 (95 - 100) PCO2 : 27
(35 - 45) HCO3- : 17 (22 - 26) Base excess : -15 (+1 - -2) What is the acid base abnormality found ?
a) Mixed acidosis
b) Primary metabolic acidosis
c) Primary metabolic acidosis with partial respiratory compensation
d) Primary respiratory acidosis with partial metabolic compensation
e) Uncompensated respiratory alkalosis
Answer - C

Pathophysiologically speaking, when there is an intestinal obstruction a metabolic acidosis will occur
due to loss of bicarbonate. According to the data given there's an acidotic pH value with reduced
bicarb level. Carbon dioxide level is low as a compensatory mechanism but it is only a partial
compensation because pH is still low. Therefore this is primary metabolic acidosis with partial
respiratory compensation.

2018 Nov
18. Following factors indicate Difficult Airway and intubation during general anesthesia,
a) Body mass index of 35kg/m
b) Mallampathy score - 3 (Three)
c) Burn contractures of the neck
d) Thyromental distance is more than 6cm.
e) Cervical spondylosis
Answers – T T T F T
Higher Mallampathy score and a lower thyromental distance indicate difficult intubation.
44. A 60-year-old lady underwent laparotomy due to perforated peptic ulcer. On the 5 the Post-
operative day, she developed a fever. Her respiratory effort became poor. What will be the Arterial
blood gas findings?
a) pH 7.44, pcm 35, p02 70, HC03 29
b) pH 7.2, pcm 40, p02 78, HC03 18
c) pH 7.2, pC02 55, pm 70, HC03 29
d) pH 7.5, pcm 21, p02 50, HC03 19
e) pH 7.44, pcm 40, p02 78, HC03 24

Answer – C
By the stem, it feels like the patient has developed a pneumonia. With poor respiratory effort she is
expected to be hypercarbic resulting in respiratory acidosis. Only B and C answers have a low pH value.
No need to even look at the other answers from this point. Oxygen levels in both answers are low. By
this alone we can't decide the answer. Metabolic compensation should increase bicarbonate level.
Only answer C meets these criteria. There we go! See how easy ABG analysis is. We got the answer
without spending a drop of ink.

45. 50 lady undergone an Elective Thyroidectomy. While you are monitoring in the ward during post-
operative period, you detect reduced urine output. Bladder is not palpable. What is the best next in
the management of this patient?
a) 5% dextrose Infusion
b) 0.9% NaCl Bolus
c) Fresh frozen plasma
d) Hartman's solution
e) IV Frusemide

Answer – B
This is pure basic theory. Normal saline is the go-to fluid for a fluid challenge.

2017 May
38. A 75 year old male present with severe abdominal pain and distention to the emergency
Surgical department. He has ischemic heart disease.An urgent CT abdomen shows evidence
of bowel ischemia. His arterial blood gas analysis is as follow
PH…………………………………-7.25(7.35-7.45)
PaC𝑂2…………………………..-75mmHg(80-100)
PaC𝑂2…………………………..-21mmHg(35-45)
Base excess………………-15(-2 to +2)
Which of the following best describe the acid base status
a) Metabolic acidosis with partial respiratory compensation
b) Metabolic acidosis with respiratory compensation
c) Metabolic acidosis without compensation
d) Respiratory acidosis with metabolic compensation
e) Respiratory acidosis without compensation.
Answer – A
Same question was asked in 2019. The only difference is this time base excess was given. It's no magic.
A more negative base excess means loss of bicarbonate. If you got it wrong refer to my annotation on
question 44 in 2019 paper.

48. 2 hours after laparoscopic cholecystectomy a patient to be drowsy. RR=10/min, pulse rate is
80/min, and BP is 130/80mmHg. The arterial blood gas reports as follows.
PH………………………………..-7.30(7.35-7.45)
Pa𝑂2……………………………...-94mmHg (80-100)
Pa𝐶𝑂2……………………………...-50mmHg (35-45)
The most appropriate immediate Ix is,
a) Application of CPAP
b) IV fluid bolus of 200ml colloid
c) IV morphine
d) IV naloxone
e) Intubation and ventilation
Answer – D
The diagnosis is in your hands the moment you see a reduced respiratory rate. Obviously a well-
meaning doctor may have prescribed a little too much of morphine to alleviate the poor patient's pain
resulting in an opioid overdose. That's the cause of his type 2 respiratory failure i.e. low carbon dioxide
with normal oxygen level. The antidote is naloxone. This doesn't cost you even 30 seconds despite the
fact that it is question number 48.

2016 Nov
15. Which of the following are true about anesthesia?
a. 5% bupivacaine is used to anaesthetize pharynx in upper GI endoscopy.
b. Adrenaline with lignocaine increases the duration of action the lignocaine used alone.
c. Prilocaine is used before injection giving injection
d. Lignocaine with adrenaline is used for anaesthesia in suturing a cut in tip of finger.
e. Local infiltration of 1% lignocaine is used to carpal tunnel decompression

Answers – FTTFT
A. Xylocaine spray is used to numb the throat before an endoscopy. Interesting fact is that Xylocaine
is a trade name and the actual local anesthetic in it is LIDOCAINE.

B. This is exactly the reason why we can go upto a maximum of 7mg/kg of lidocaine when it is
combined with adrenaline compared to the maximum dose of plain lidocaine. If you don't remember
the maximum dose of plain lidocaine find it out.Hint- It is definitely less than 7mg/kg.

C. Especially when giving injections to pediatric patients or performing phlebotomy on them topical
EMLA is used and this contains prilocaine. But this practice of using a topical anesthetic before an
injection, is not happening in our wards for obvious reasons.

D. This is something we have learnt by heart even before we saw a lignocaine vial for the first time.
Only Plain lignocaine for tissues with an end arterial supply unless you want to amputate that
fingertip or the earlobe later.
E. I'm sure you have seen this being done.
16. 58 years old male is planned to undergo an inguinal hernia repair. What factors would prevent
him undergoing a day surgery?
a. uncontrolled DM
b. Obesity BMI > 30 kg/m2
c. History of smoking.
d. Live alone at home.
e. Body mass index >30kg/m2
Answers – T??T?

22. 75 years old patient treated for hypertension and ischemic heart disease is schedule to undergo
TURP under spinal anaesthesia. Which drug should be discontinued at least one week before?
a. Aspirin
b. Atenolol
c. clopidogrel
d. warfarin
e. heparin

Answers – TFTFF
- Some surgeons opt to continue aspirin. But the general recommendation is to stop it a week
before surgery.
- Antihypertensives should be continued and even the morning dose needs to be given on the day
of the surgery.
- Clopidogrel is more potent than aspirin and it should be stopped at least one week before
surgery.
- Warfarin can be continued upto 5th pre-op day. It must be stopped but it doesn't require at least
a week of warfarin free interval. That is why we mark it as false because the question wants us to
pick the drugs that need to be stopped AT LEAST ONE WEEK prior to surgery.
- That highlights the importance of reading the question. Heparin is short acting unlike warfarin
and can be continued upto 6-12 hours prior to surgery. Sometimes heparin is used as a bridging
anti-coagulant after stopping warfarin.

34. A polytraumatic patient 25 year old male admitted to ETU. He is dyspnoeic, GCS 9/15, brought to
hospital. He is on 50% Oxygen. His ABG results were, pH 7.25, pCO2 90mmHg, pO2 – 60mmHg,
HCO3- 30, BE 3mmol/l. What is the most appropriate management?
a. CPAP
b. Endotracheal intubation and ventilation
c. IV 8.4% NaHCO3
d. IV mannitol infusion

Answer – B
Type 2 respiratory failure with a GCS of 9 means that most likely he has traumatic brain injury. So it
is wise to intubate and ventilate him.
36. 40-year-old man presented with features of acute abdomen. On examination hypotension,
tachycardia was present. 1l of normal saline administered. After 20 minutes what is the fluid
distribution?
a. 250 mL remain in the extravascular compartment
b. 250 mL remain in the intravascular compartment.
c. 500 mL remain in the intracellular compartment
d. 500 mL remain in the extravascular compartment
e. 500 mL remain in the intravascular compartment

Answer – B
This is first year, first semester, first week physiology. 25% of ECF is intravascular and the remainder
is extra-vascular. Normal saline is isotonic and therefore does not enter into the intracellular
compartment. All the normal saline infused will remain in ECF (intravascular + extravascular) with a
distribution ratio of 1:3 between the two compartments of ECF. The rest is grade 5 maths. Out of 1000
ml, 250 ml will remain intravascularly. This is the basis for using 1:3 replacement ratio when using
normal saline to resuscitate a patient with hemorrhagic shock. To replace a blood loss of 250 ml we
have to administer a 1 liter bolus of normal saline. That's why clinicians always say that YOU CAN'T
FORGET YOUR BASICS.

46. 40 years old previously healthy male was treated for cerebral abscess. Post treatment time was
uneventful except for persistently low Na⁺& low plasma osmolality. But his urinary Na⁺ is normal. No
peripheral oedema. BP is normal. Most likely diagnosis?
a. Acute kidney injury
b. Diabetes insipidus
c. Excessive diuretic therapy
d. Pituitary failure
e. SIADH

Answer – E
If it is AKI patient would have a raised BP and he would be oliguric and urinary sodium would be
deranged. In diabetes insipidus lack of ADH would result in polyuria with low urine osmolality. Central
diabetes insipidus due to pituitary failure would do the same. Since this patient has a normal urinary
Na+ level, diabetes insipidus is unlikely. Over-diuresis would increase urinary sodium too. As Sherlock
Holmes would say when you have eliminated the impossible, whatever remains, however improbable,
must be the truth.
Therefore, the answer is D. Just kidding. SIADH fits this case perfectly. Excessive ADH secretion will
increase water reabsorption from the distal convoluted tubule of the nephron causing a dilutional
hyponatremia. ADH does not alter sodium reabsorption. Therefore, urinary sodium excretion would
remain unchanged. Interestingly enough cerebral abscesses are a known cause of SIADH. So, even if
you didn't know anything else this fact would have led you to the correct answer.

47. A 53 year old woman presents with muscle pain, malaise, constipation and back ache. Laboratory
investigations revealed hypercalcemia. What is the appropriate management for her?
a. Administration of thiazide diuretics
b. Calcium resonium
c. IV Bisphosphonate
d. IV Normal saline
e. Neck exploration & parathyroidectomy
Answer – D

The patient has symptoms of hypercalcemia and investigations have confirmed it. Like a typical final
MBBS SBA there is no wrong answer here. Everything will eventually resolve her hypercalcemia. But
the most appropriate management for now is to hydrate her with normal saline.
We can't go for a parathyroidectomy straightaway because as we have crammed it is only one cause
of hypercalcemia. What if the patient has hypercalcemia secondary to paraneoplastic syndrome? So
lowering serum calcium first due to risk of cardiac arrhythmia and then investigating and treating the
underlying cause would be the most appropriate management.
Breast disorders
2020 June
09) 45-year-old lady present with 3 cm breast lump in the left upper quadrant,
FNAC found to be 5. there is no palpable axillary lymph nodes. Mammogram of the
bilateral breast normal. No clinical /imaging suggestive of metastasis. which of the following
is /are true regarding management?
A. Bone scan indicate before surgery
B. Breast conservation is preferred method
C. Lumpectomy is an option
D. Neoadjuvant therapy is indicated
E. Sentinel node biopsy is ideal method

Answers – F T F F T
- Here the lesion is clinically palpable, but radiologically not visualized. Examples for such a lesion
are fibroadenosis/ANDI, lobular carcinoma. Here as FNAC is diagnostic of malignancy lobular
carcinoma is the possibility.
- In breast CA, bone scan is indicated only if the patient is symptomatic/ ALP is high.
- Lobular carcinoma can be multifocal, yet until we confirm this histologically with a tru cut, we can
still do a breast conservation surgery.
- Lumpectomy is an option for benign lesions
- Indications for neoadjuvant in breast cancer are T4 tumour, 4cm sized tumour, 4 axillary lymph
nodes are positive.
- As axillary lymph nodes are not palpable, we can do a sentinel node biopsy during surgery.

33) A 43 years old woman presented to the breast clinic with retraction & green colour nipple
discharge for 2 months’ duration. Ultrasound breast scan shows custs & subareolar duct dilatation.
What is the possibility?
A. Duct ectasia
B. Breast carcinoma
C. Chronic breast abscess
D. Galactocoele
E. Paget’s disease of the breast

- Duct ectasia is the dilatation in one or more of the larger lactiferous ducts, which fill with a
stagnant brown or green secretion. This may discharge. These fluids then set up an irritant
reaction in surrounding tissue, leading to periductal mastitis or even abscess and fistula formation
- In some cases, a chronic indurated mass forms beneath the areola, which mimics a carcinoma.
Fibrosis eventually develops, which may cause slit like nipple retraction.
- Nipple discharge (of any colour), a subareolar mass, abscess, mammary duct fistula and/or nipple
retraction are the most common symptoms.

Answer – A, Duct ectasia


2019 Nov
9) Regarding breast carcinoma
a) Ductal in situ carcinoma is pre-invasive
b) Negative hormonal receptivity is poor prognostic factor
c) The markedly improved cure rate is due to improvement in surgical techniques
d) Commonest cancer in Sri Lanka
e) Prognosis is better in post-menopausal women

- Negative oestrogen, progesterone receptivity has poor prognosis as you lose the chance of
treating with tamoxifen. However, Herceptin receptor negativity is good.
- Improved cure rate is due to early identification with the diagnostic tests and screening
programmes in developed countries.
- Prognosis in breast cancer is assessed based on histological grade of the tumour, hormone
receptor status, measures of tumour proliferation such as Ki-67, growth factor analysis and
oncogene or oncogene product measurements. (Nottingham prognostic index)
Answers - T T F T F

33) A 31-year-old woman who is 7 weeks’ post-partum presents with painful, swollen left breast.
Examination reveals erythematous, tender and fluctuant area above the left nipple areolar complex.
What is the most appropriate management?
a) Core biopsy
b) Incision and drainage
c) IV antibiotics
d) USS guided needle aspiration
e) Wide local excision

- This is lactational breast abscess as evident by an acutely inflamed fluctuant area. But unlike other
localized infections, fluctuation is a late sign in a breast abscess.
- Repeated aspirations under antibiotic cover (if necessary using ultrasound for localization) be
performed. This often allows resolution without the need for an incision and will also allow the
patient to continue breast-feeding.
- I&D is less commonly needed as prompt commencement of antibiotics and repeated aspiration is
usually successful. Incision of a lactational abscess is necessary if there is marked skin thinning and
can usually be performed under local anaesthesia.

Answer – D, USS guided needle aspiration


2019 March
7) 30 year old woman who is breast feeding presented with painful area in right breast for 3 days. She
has mild fever. On examination the area is red, warm and tender. Which of the following are
appropriate in her initial management?
a. Antibiotics
b. Avoid feeding from affected side
c. Breast milk for culture & ABST
d. Incision and drainage
e. Ultra sound scan of the breast.

This is lactational mastitis.


- patient should be treated with an appropriate antibiotic, such as flucloxacillin or coamoxiclav.
- Feeding from the affected side may continue if the patient can manage. Support of the breast,
local heat and analgesia will help to relieve pain.
- Breastmilk culture isn’t needed as a lactational breast abscess is formed due to a blocked lobule,
and pus wont mix with the secreted milk.
- Lactational mastitis can be associated with a lactational breast abscess. Therefore, if an abscess
is formed repeated aspirations under antibiotic cover (if necessary using ultrasound for
localization) can be performed. I&D can be done if aspirations fail.
- US scan isn’t needed as it’s a clinical diagnosis. If breast abscess develops patient will need USS in
future.
Answers –T F F F F

8) Women with breast lump undergone wide local excision with axillary clearance. Axillary lymph node
sampling was negative. What is next step in management?
a) radiotherapy to breast
b) radiotherapy to axilla
c) radiotherapy to breast and axilla
d) chemotherapy
e) Mastectomy

Here as axillary lymph nodes are negative, and a breast conservation surgery is performed,
radiotherapy to the breast alone is enough.
Answer - A

28) 40 year old lady presented with unilateral recent onset blood stained nipple discharge. She has no
axillary lymph nodes or palpable breast lumps no cystic changes.
a) breast carcinoma
b) duct ectasia
c) ductal papilloma
d) fibrocystic change
e) mondor disease.
- Intraductal papilloma is a benign tumor found within breast ducts. The abnormal proliferation of
ductal epithelial cells causes growth.
- A solitary intraductal papilloma is usually found centrally posterior to the nipple affecting the
central duct. Multiple intraductal papillomas are located peripherally, found in any breast
quadrant affecting the peripheral ducts.
- Patients with symptoms often present with spontaneous bloody or clear nipple discharge. An
intraductal papilloma may be occasionally palpable.

Answer – C, ductal papilloma

29) Woman with a suspicious breast lump has undergone wide local excision with axillary clearance.
Axillary lymph node sampling was negative. What is the next management?
a) radiotherapy to breast
b) radiotherapy to axilla
c) radiotherapy to axilla and breast
d) systemic chemotherapy
e) mastectomy

Here as axillary lymph nodes are negative, and a breast conservation surgery is performed,
radiotherapy to the breast alone is enough.
Note: Even if axillary lymph nodes were positive, if a good axillary node clearance is done, radiotherapy
to axilla isn’t needed as it increases the risk of lymphedema.
Answer – A

2018 Nov
7. In the evaluation of a breast lesion in a female, a MRI is indicated over a conventional mammogram
in,
a) Multiple breast lumps in a single breast
b) In a patient who has undergone radiotherapy for previous breast lesion.
C) 50yr old with a breast lump.
d) Paget’s disease of the breast with no underlying lump.
e) 70-year-old lady with a breast lump.

Breast MRI is advantageous if


 Breast cancer risk is high and screening is needed at an earlier age (as mammogram is not
effective in dense breast tissues of the young female)
 Lobular carcinoma as micro calcifications are absent and missed in mammogram.
 If local recurrence is suspected after mastectomy/ breast conservative surgery because
mammo is difficult to interpret due to scar tissue

Answers – F T F T F
8. Pyogenic breast abscess,
a) Common among breast feeding women.
b) Cracked Nipple is a risk factor.
c) Incision and Drainage is a treatment option.
d) Staphylococcus aureus is the commonest causative agent.
e) Breast feeding should be stopped until full resolution.

Answers – T T T T F

35. Regarding malignant features in a mammogram, what is the best feature to suggest Malignant
lesion,
a) Architectural distortions
b) Ill defined edges.
c) Micro calcifications
d) Fatty hilum in lymph nodes
e) Size greater than 3cm.

Microcalcifications and speculations are the 2 most important features in mammogram suggestive of
malignancy. Of these, micro calcifications are more sensitive. But neither of them are specific.
Normal lymph nodes have a fatty hilum. In malignancy this is lost.

Answer – C, Micro calcifications

2017 May
7. Regarding breast conditions
a) Chronic granulomatous mastitis presents with recurrent breast abscesses
b) Gynecomastia is associated with chronic liver diseases
c) Paget disease mimics eczema of the nipple
d) Single duct blood stained nipple discharge occurs in duct papilloma
e) Streptococcus pyogenes is the commonest cause of lactation breast abscess

- Chronic granulomatous mastitis can clinically mimic bacterial abscess or breast cancer. Patients
may present with skin induration, tenderness, erythema, sinus tract formation, breast edema,
and/or a mass like lesion
- In CLCD, oestrogen metabolism is lost, therefore gynecomastia develops.
- Paget’s disease is the eczema like condition of nipple and areola. It can be a manifestation of an
underlying malignancy. It is seen in 2% of breast CA. Unlike eczema this is unilateral and non-itchy.
- A solitary intraductal papilloma is usually found centrally posterior to the nipple affecting the
central duct and causes blood stained nipple discharge.
- Staph aureus is the commonest organism causing lactational breast abscess.

Answer – T T T T F
8. Adverse effects of tamoxifen include
a) Deep vein thrombosis
b) Endometrial carcinoma
c) Hot flushes
d) Osteoporosis
e) Ovarian carcinoma
Answer – T T T F F

25. An 18 years old female present to the clinic with a firm 2 cm lump in the upper inner quadrant of
the left breast. The lump has well defined margins and is mobile.
What is the most appropriate next step in the management of this patient?
a) Excision biopsy
b) Fine needle aspiration
c) Mammography
d) Reassurance and discharge
e) Ultrasonography

A fibroadenoma is the most possible diagnosis as it is a young girl, and lump is mobile with clear
margins.
In view of triple assessment, fibroadenoma is confirmed with USS, and if patient wishes lumpectomy
can be performed i.e. an excision biopsy for pathological diagnosis. If not, can do FNAC to complete
triple assessment if we plan to manage conservatively.
Answer - E

2016 Nov
7. 48-year-old female presented with 2 weeks’ history of nipple discharge. Which of the following are
suggesting of malignancy of the breast?
a. Blood stained nipple discharge
b. Discharge from a single duct
c. Painful duct discharge
d. Mass under the nipple areolar region
e. Unilateral nipple discharge

Answer – T T F T T

8. Regarding gynecomastia,
a. Commonly unilateral.
b. is a feature of testicular tumour
c. Occurs due to glandular proliferation of male breast tissue.
d. Pubertal gynecomastia resolves spontaneously.
e. Tamoxifen is a mode of a treatment.
Gynecomastia can be seen in teratoma of testes.
Tamoxifen, an estrogen antagonist, is effective for recent-onset and tender gynecomastia.

Answer – T T T T T

30. 60-year-old lady presented to the clinic with an eczematous lesion of the right nipple areolar
complex. On examination, palpable breast lump (3x4cm) in right breast with ipsilateral palpable
mobile lymph nodes. What is the response that gives the most accurate diagnosis?
a. Eczema of right nipple areolar complex with lymph nodes in right axilla
b. Locally invasive right side breast CA
c. Paget’s disease of the right breast
d. Paget’s disease with right invasive breast CA
e. Paget’s disease T2N1Mx

Here the eczematous lesion is the Paget’s disease, a manifestation of an underlying malignancy. It is
seen in 2% of breast CA. Further the underlying lump is breast carcinoma with nodal spread.

Answer – D

31. A 26 years old lady presented with a lump in the right upper outer quadrant of the right breast for
2 weeks’ duration. It was 3cm in size. Mammogram and USS showed features suggestive of
malignancy. What is the next most important step?
a. Excisional biopsy.
b. Incisional biopsy.
c. FNAC.
d. Core biopsy.
e. MRI of the right breast.

A trucut is done to complete triple assessment.

Answer – D
Hernia
2020 June
06) Regarding hernia
A. Epigastric hernia commonly contain omentum
B. Ingunial hernia has increased risk for strangulation than femoral hernia
C. Hernia lateral and down to the pubic tubercle is femoral hernia
D. Ingunial hernia in children mesh repair reduce recurrence
E. Paraumbilical hernia is commoner in obese women
Answer - F F T F T
A - extra peritoneal fat
B - 50% of femoral hernia present as an emergency with very high risk of strangulation (Bailey and
love 27th edition page 1035)
D – usually we don’t do mesh repair in children. Herniotomy is enough.

30) 5y old girl presented with reducible umbilical hernia. The umbilical bulge was present from birth.
On examination there was 1.5 cm defect. She is on treatment for bronchial asthma for 2 years. The
reason that She was offered surgery as management
A. Hernia 1.5cm size would not automatically regress.
B. at the age of 5years hernia wouldnot have a spontaneous regression
C. Due to the possibility of incarceration, herniotomy should be performed
D. Its cosmetically in appropriate
E. with Bronchial asthma, will not cure
Answer - B
Umbilical hernia in children usually regress by 2 years. If not they are unlikely to resolve, they need to
undergo surgery (Bailey and love 27th edition page 1037)

2019 Nov
6) Regarding incisional hernia
a) Rectus closure by polyglactin reduce the risk
b) Higher rate of strangulations than femoral hernia
c) High incidence in obese
d). Laparoscopic repair is possible.
e) Prophylactic mesh reduces the risk
Answer – F F T T T
A - Polyglactin is an absorbable suture material. So it will increase the risk.
We need non absorbable or very slowly absorbable sutures of adequate guage.
B - Strangulation is less frequent and most likely to occur when the fibrous defect is small and the sac
is large. But Most incisional hernias are broad-necked and carry a low risk (page 1039)
But they can get obstructions due to adhesions.
C – predisposing factors include obesity, malnutrition, immune suppression, chronic cough, cancer
D - Open or laparoscopic surgery is possible
2019 March
17) 40yrs old presented with paraumbilical hernia.what are the indications for early surgery
a) longer duration of hernia
b) irreducibility
c) skin ulceration over the hernia
d) tenderness over the hernia
e) components being bowel as opposed to the omentum
Answers – T T T T T
Small asymptomatic ones may left alone.

37) 48 year old man coming with a irreducible painful hernia associated with abdominal pain.on
examination abdomen is tender. What is the immediate management?
b) Manual reduction under sedation
c) Explore hernial sac
d) Emergency laparotomy
e) ice packs and elevate the limb
Answer – D - this patient need emergency laparotomy to reduce the content of hernia and repair.

2018 Nov
16. 40 patient with an inguinal lump which is becoming more prominent on standing,
a) Femoral Hernia
b) Direct inguinal hernia
c) Inguinal lymph node
d) Saphena varix
e) Varicocele
Answer – T T F T T
Direct inguinal and femoral hernia, saphena varix and varicocele, undescended testes become
prominent on standing
This question lacks important information. In 2018 November J’pura paper its mentioned as A 40-year-
old female patient. Therefore, in that case varicocele in wrong.

37. 30-year-old man presented with a right inguinal swelling for 8 hours duration with a colicky lower
abdominal pain and vomiting. He was managed with Morphine and IV Fluids. His swelling disappeared
in 45 minutes. His abdominal pain and vomiting also subsided What is his condition at presentation?
a) Strangulated Hernia
b) Obstructed Hernia
c) Richter's Hernia
d) Incarcerated Hernia
e) Bowel perforation
Answer – B
Obstructions sometimes resolve with conservative management. The answers A and D are out, since
the swelling won’t reduce with conservative mx. Bowel perforation will not resolve with this Mx.
Richter’s hernia will not present with obstruction. It only contains a part of bowel wall.
2017 May
35. A 45-year-old female present with a reducible midline anterior abdominal wall lump 5 cm above
the umbilicus. She complains of dyspeptic symptoms with poor response to oral medication. She has
had a LRT done 7 years ago. What is the most likely diagnosis?
a) Divarication of the recti
b) Epigastric hernia
c) Incisional hernia
d) Lipoma
e) Paraumbilical hernia
Answer – B
Epigastric hernia is most likely, incisional hernia, lipoma, PUH is not possible. Since she also has
dyspeptic symptoms which doesn’t respond to medication explains she has sympotoms mimicking a
PUD. So epigastric hernia is most possible.

2015 Nov
9. A 40-year-old female presented with paraumbilical hernia of 2 cm in size. She is complaining of
intermittent abdominal pain. WOF is most likely content?
A. Ileum
B. Omentum
C. Peritoneal fluid
D. Preperitoneal fat
E. Transvers colon
Answer – B intermittent pain is mostly due to omental strangulation
This is 2015 November 37 (SBA)
Burns
2020 June
47) 40 years old lady rescue from burning house & bring to ICU. She is having stridor.
What is the most appropriate management option?
A. Ambo bag
B. Arterial blood gas
C. Endotracheal tube
D. Humidified oxygen
E. Oral airway
Answer – C
Patient was trapped in a burning house, she has a risk of developing laryngeal oedema and airway
obstruction. She already has stridor or hoarseness of voice therefore, intubation is needed.

2018 Nov
36. A 35-year-old lab worker presented with a burn injury over the left forearm due to bleaching
powder. What is the most appropriate next step in the management?
a) Wash well with water(lavage)
b) Add a weak alkali agent
c) Add a weak acidic agent
d) Add silver sulphur diazine
e) Apply local hydrocortisone cream
Answer – A

Chemical injuries
– there are two aspects: physical destruction of skin and systemic absorption
– Alkali injuries are usually more destructive than acids. And dangerous if contacted with eyes.
– initial Mx of any chemical injury is copious lavage with water.
– Then identify the chemical and assess the risks of absorption.

2017 May
12. Regarding burns
a) Boiling water causes deeper burns than hot oil
b) Fluid requirement for the first 12 hours is 50% of [4 x body weight(kg) x % burned surface area]
c) Hand burns do not require admission
d) The Lund and Browder chart is used to access the burned surface area
e) The presence of blisters indicates an epidermal burn
Answers – F F F T F
A – Scald (hot water burn) causes superficial burns while fat burns cause deep dermal burns (table
41.2 in Bailey and love 27th edition page 622)
C – Any burn which is likely to need surgery, any burn with significance to hands, face, feet or perineum
needs admission (Bailey and love 27th edition page 620)
D – useful in larger burns and it is accurate
E – superficial partial thickness burns has blisters
2016 Nov
33. A burn patient weighing 50 kg, sustained a partial thickness burn of 30% body surface area. What
is the amount of fluid that should be given within first 8 hours of injury?
a. Hartmann 2000 mL
b. Hartman 3000 mL
c. Hartmann 6000 mL
d. Hartmann 1000 mL
e. Hartmann 1500 mL
Answer – B
Total fluid requirement = 4 x 50 x 30/100
= 6L (6000 ml)
st
for the 1 8 hours = 3000 ml

2013 Nov – Colombo


8) Deep facial burn
a) Admission is mandatory
b) Fluid resuscitation essential
c) NG tube used in feeding
d) Airway injury is a complication
e) Managed open wound
Answers – T T T T T
Salivary gland diseases
2020 June
11) 52-year-old man is waiting to undergo submandibular gland resection. What are
the structures that can damage during surgery?
A. Ansa cervicalis
B. Hypoglossal nerve
C. Lingual nerve
D. Marginal mandibular branch of facial nerve
E. Vagus nerve

Answers – F T T T F

Submandibular gland resection (bailey and Love 27th edition, page 783)
Submandibular gland incision is indicated for
1. sialadenitis, when minimally invasive methods have failed
2. salivary tumours

Complications of submandibular gland excision


 hematoma
 wound infection
 marginal mandibular nerve injury
 lingual nerve injury
 hypoglossal nerve injury
 transection of the nerve to the mylohyoid muscle producing submental skin anesthesia

Three cranial nerves (CNs) are at risk during removal of the submandibular gland:
1. the marginal mandibular branch of the facial nerve;
2. the lingual nerve;
3. the hypoglossal nerve
An adequate incision coupled with meticulous hemostasis allows the surgeon to identify these
important structures during surgery.

2019 Nov
11) In a 50-year-old male presenting with a large parotid tumour,
a) Sialogram is indicated to exclude calculus
b) Core biopsy is mandatory prior to surgery
c) Ipsilateral facial nerve palsy is due to compression of the nerve
d) Pleomorphic adenoma will be the most likely diagnosis
e) USS is indicated

Answers – F F F T T
A - Sialolithiasis is less common in the parotid gland (20%) than in the submandibular gland (80%).
Parotid duct stones are usually radiolucent and rarely visible on plain radiography. They are frequently
located at the confluence of the collecting ducts; at the point the duct courses over the masseter
muscle or in the distal aspect of the parotid duct adjacent to the parotid papilla. The stones are easily
demonstrated on ultrasound. (B & L – 787)

B – Incisional and excisional biopsy are never done. FNAC is commonly used.
D - 80–90% of tumours of the parotid gland are benign, the most common being pleomorphic
adenoma
E - initial imaging modality of choice is ultrasound

2019 March
46) Lingual nerve is damaged during submandibular sialadenectomy. What is the most likely effect
on tongue?
A. Loss of general+taste sensation in ipsilateral anterior 2/3 of tongue
B. Loss of general sensation in ipsilateral tongue + ipsilateral taste in anterior 2/3 of tongue
C. Loss of ipsilateral general sensation in anterior 2/3 of tomgue
D. Loss of general sensation ipsilatearl tongue
E. Loss of taste sensation in ipsilateral anterior 2/3 of tongue

Answer – B
(there are some typing mistake in these answers)
Lingual nerve is a branch of trigeminal nerve; it also transmits fibers from corda tympani branch of
facial nerve. It receives taste sensation from anterior 2/3 and general sensation from anterior 2/3 of
tongue.
General sensation and taste to posterior 1/3 is from glossopharyngeal nerve.

2016 Nov
27. 21-year-old woman presented to the surgery clinic with pain and swelling over the right
submandibular region which is worse soon after meals. Most appropriate next step in the
management
a. Core biopsy
b. FNAC of nodule.
c. Submandibular sialadenectomy.
d. X-ray closed mouth occlusive view
e. CT-neck

Answer – D

Salivary gland Stones


 Most common site to occur is the submandibular gland.
 This is because the secretions are thick and drainage occurs against gravity.
 It presents with pain and swelling which is worse with meals and when thinking about meals as
saliva secretion increases.
 Most stones are radiopaque. X-ray closed mouth occlusive view is used for diagnosis.
Orthopantogram can also be used.
 For stones located in the periphery of the duct removal intraorally is possible. For ones located
inside need excision.

2015 Nov
16. Regarding salivary glands
A. Stones are common in submandibular gland
B. Marginal mandibular nerve can get damaged in submandibular sialadenectomy
C. Acute parotiditis commonly causes by staphylococcus aureus
D. Commonest tumour is warthin’s tumour
E. young women commonly get adenolymphoma of parotid

Answers – T T T F F
Warthian’s tumour = adenophymphoma

Neck masses
2020 June
35) 4-year-old child presented with lump in supraclavicular area. It was gradually increase in size. It
has brilliant trans illumination. What is the most likely diagnosis?
A. Cystic hygroma
B. Sternocleidomastoid tumour
C. Haemangioma
D. Pharyngeal cyst
E. Branchial cyst

Answers – A
Cystic Hygroma
 usually present in the neonate or in early infancy
 The cysts are filled with clear lymph and lined by a single layer of epithelium
 Swelling usually occurs in the neck and may involve the parotid, submandibular, tongue and floor
of mouth areas.
 The swelling may be bilateral and is soft and partially compressible, visibly increasing in size when
the child coughs or cries.
 The characteristic that distinguishes it from all other neck swellings is that it is brilliantly
translucent.
 The cheek, axilla, groin and mediastinum are other less frequent sites for a cystic hygroma
 Definitive treatment involving complete excision of the cyst at an early stage is best if possible
2019 Nov
35) A 50-year-old man, a heavy smoker, presents with a painless lump on the upper left neck of 2
months’ duration. He has no other symptoms. On examination the lump appears to be a firm, fixed
upper cervical node (level II). FNAC suggests malignancy. What is the most appropriate investigation
to arrive at the diagnosis?
a) Bronchoscopy
b) Excision biopsy
c) Fibre-optic examination of the larynx and pharynx
d) USS of the abdomen
e) Upper GI endoscopy

Answer – C
this patient has painless cervical lymph node which is firm and malignant looking.
Various conditions can present with painless cervical lymphadenopathy. Most of the time painless
lymphadenopathy is due to malignancy. The 1st step in assessing this condition is to go through the Hx
and identify the probable underlying condition.
 Lymphoma is a common condition giving rise to painless cervical lymphadenopathy.
 It may be associated with evening pyrexia, drenching night sweat, LOA and LOW.
 But keep in mind that most lymphomas only have lymphadenopathy. So after the history next
target should be to do a full examination to detect whether it is localized or generalized
lymphadenopathy.
 Keep in mind sometimes the lymphadenopathy may be there in para-aortic nodes, tracheal nodes
etc. These are not detected during Ex.
 If the history and examination does not give a clue to the possible diagnosis, in Ix the 1st step
should be to exclude this.
 So USS abdomen is done to detect hepatosplenomegaly and visceral lymphadenopathy. If present
it is most likely a lymphoma.
 Then the next step is to do an excision biopsy to confirm the diagnosis. If there is no
lymphadenopathy elsewhere the next step is to do a FNAC of the enlarged lymph node.
 Based on the cellular details further mx is planned.
 If squamous deposits are seen flexible nasopharyngoscopy to exclude a nasopharyngeal CA, UGIE
to detect esophageal CA, CECT chest and bronchoscopy to detect bronchial CA should be done.
 Thyroid CA may also present with cervical lymphadenopathy but it is not squamous.
In this patient, FNAC done, which means this is not generalized, therefore, next would be to do
nasopharyngoscopy.

2016 Nov
20. A 70yr old man presents enlarged hard lymph node in anterior triangle of neck. Biopsy revealed
deposit of squamous cell carcinoma. What are the investigations need to find out primary site?
a. nasal endoscopy
b. Upper GI endoscopy
c. CECT thorax
d. Bronchoscopy
e. Serum calcitonin
Answer – T T T T F
Again the same explanation.

2017 May
30.A 10-year-old male with poor oral hygiene and molar dental caries presents to the surgical
casualty ward with swelling of the neck and high fever 2 days duration. On examination he has
swelling of the floor of the mouth and displacement of the tongue posteriorly . what is the most
likely diagnosis?
a) Angio oedema
b) Ludwig angina
c) Sublingual haematoma
d) submandibular lymphadenitis
e) Submanddibular sialoadenitis

Answer – B

Ludwig’s angina
- characterised by a brawny swelling of the submandibular region combined with inflammatory
oedema of the mouth.
- These clinical features, as well as accompanying putrid halitosis define the condition.
- The infection is often caused by a virulent streptococcal infection associated with anaerobic
organisms.
- There may also be an underlying oral cavity cancer.
- The infection tracks deep to the mylohyoid muscle causing oedema and inflammation resulting
in an upward and backward displaced tongue giving rise to dysphagia and subsequently to
painful obstruction of the airway.
- Unless treated, cellulitis may extend beneath the deep fascial layers of the neck to involve the
larynx, causing glottic oedema and further airway compromise.
- Antibiotic therapy given ASAP using intravenous broad-spectrum antibiotics, with anaerobic
cover.
- If the swelling does not subside rapidly with such treatment, or in advanced cases where pus is
evident, a curved submental incision may be used to drain.
- The mylohyoid muscle may be incised to decompress the floor of the mouth and corrugated
drains placed in the wound, which is then lightly sutured.
- This operation may be conducted under local anaesthetic.
Skin and subcutaneous tissues
2020 June
31)70 years old women presents with a slowly growing ulcer on her nose, which is pearly in
appearance with raised margins & telangiectasia on the surface. What is the most likely diagnosis?
A. Melanoma
B. Bowens disease
C. Basal cell carcinoma
D. Squamous cell carcinoma
E. Tuberculosis ulcer

Answer – C
Basal cell carcinoma
- This is usually a slow-growing, locally-invasive, malignant tumour of pluripotential epithelial cells
- The strongest predisposing factor to BCC is UV radiation.
- It occurs in the elderly or the middle-aged after excessive sun exposure
- BCC can be divided into localised (nodular; nodulocystic; cystic; pigmented and naevoid) and
generalised (superficial: multifocal and superficial spreading; or infiltrative: morphoeic, ice pick
and cicatrizing).
- Nodular and nodulocystic variants account for 90% of BCC
- Treatment can be surgical or non-surgical.
- Tumour and surrounding surgical margins should always be assessed and marked under loupe
magnification
- Where margins are ill-defined, or tissue at a premium (nose, eyes) then either a two-stage
surgical approach with subsequent reconstruction after confirmation of clear margins, or Mohs’
micrographic surgery is advisable.

Sauqamous cell CA
- malignant tumour of keratinising cells of the epidermis or its appendages
- Associated with ultraviolet radiation, chronic inflammation and chemical carcinogens
- Bowen’s disease is SCC in situ. It usually presents as a slowly enlarging, erythematous scaly
plaque and may occur anywhere on the mucocutaneous surface of the body
- The appearance of SCC may vary from smooth nodular, verrucous, papillomatous to ulcerating
lesions.
- All ulcerate eventually, as they grow.
- The ulcers have a characteristic everted edge and are surrounded by inflamed, indurated skin

Melanoma is a cancer of melanocytes and can, therefore, arise in skin, mucosa, retina and the
leptomeninges. Cutaneous melanoma is caused by exposure to UVR
42)56 years old woman with 3cm lesion on the left cheek underwent wide local excision. What is the
most appropriate soft tissue transplant for cover her defect on the face?
A. A fasciocutaneous flap from ABO compatible donor
B. Full thickness flap from same patient from inner thigh
C. Full thickness flap from same patient from mastoid region
D. HLA matched partial thickness flap
E. Partial thickness flap from same patient’s inner thigh

Answer – C
- there are two types of skin grafts. They are full thickness grafts and partial thickness grafts.
- Partial thickness grafts
 heal by fibrosis and contracture.
 So they cannot be used to replace tissues in areas where scarring is not acceptable as
on the face.
 Advantage is donor site healing is better and quicker in partial thickness grafts.
- Full thickness grafts
 heal without scarring.
 Suitable in the face.
 Donor site healing is poor.
- Skin grafts don’t bring their own blood vessels. Skin grafts cannot be used to cover when
there are exposed bones, nerves and vessels. In these situations, skin flap covers are used.
Common sites are transverses abdominis and latissimus dorsi flaps.

2019 Nov
24) A 54-year-old farmer injured his leg while farming and 4 days presented with swelling of leg
around the injured area and there was a brownish discharge and crepitus on examination. What is
the most likely infectious agent?
a) Clostridium botulinum
b) Clostridium difficile
c) Clostridium perfringens
d) Clostridium tetany
e) Staphylococcus aureus

Answer – C

Gas gangrene
- Caused by Clostridium perfringens.
- Commonly seen in Diabetics, military wound injuries with soil, immunosuppression, Colonic
surgery etc.
- Presents as pain, swelling and crepitus in the affected limb with a thin brown sweet smelling
discharge.
- It is a life threatening condition and needs early treatment with high dose benzyl penicillin
and surgical debridement.
31) A 69-year-old women presented with a recent change of pigmented patch on her left cheek,
which has been there for past 15 years. On examination, it is thick and has an irregular border with
central nodularity. What is the diagnosis?
a) Acanthosis nigricans
b) Basal cell carcinoma
c) Melanoma
d) Squamous cell carcinoma
e) Strawberry naevus
Answer – C
- Melanoma occurs mainly in females. Risk factors are female gender, fair skin, FHx, sunburn.
- Direct spread causes satellite lesions, lymphatic spread cause Intransit lesions and via blood
it goes to bones, brain, liver and lung.
- Basal cell carcinoma – almost always in white people. In sun exposed areas, 90% on face
above a line from ear lobe to corner of mouth.
- Squamous cell carcinoma - In males mainly. In chronic inflamed areas like scars.
- Strawberry naevi - Present since birth
- Acanthosis - Hx of insulin intolerance may present mostly.

2018 Nov
33. 65-year-old lady presented with a history of hypo pigmented skin nodules on her nose which was
necrotizing at the center & contact bleeding for 2 months. On examination, she had a skin lesion
which was everted & necrotic at center. It had pale margins. What is the appropriate next step in the
management?
a) Full thickness Wedge biopsy of the skin lesion.
b) Surgical resection of the lesion and put a flap.
c) Neoadjuvant Chemoradiotherapy
d) Cone Biopsy
e) Cryotherapy

Answer – A
This is suggestive of a squamous cell Ca and The histological diagnosis is necessary before the
definitive management

2017 May
44. An 80 year old man present with a basal cell CA on the back of the chest. It is 2 cm in diameter.
What is the most appropriate Mx?
a) Cryotherapy
b) Radiofrequency ablation
c) Radiotherapy
d) Topical chemotherapy
e) Wide local excision

Answer - E
Unless excision of a BCC is complete, there is a 67% recurrence rate if margins are grossly involved
50. A 45 years old DM patient presents with high fever and a painful swelling over the upper back.
Ex, shows large erythematous, warm, tender swelling with purulent discharge from multiple points
around the center.
What is the most likely diagnosis?
a) Abscess
b) Carbuncle
c) Furuncle
d) Infected sebaceous cyst
e) Necrotizing fasciltis

Answer – B

Boils
- Usually caused by Staphylococcus.
- It’s an infection of hair follicles. Presents as a painful red swelling which later becomes filled
with pus and starts to discharge spontaneously and resolve.
- Occur in the scalp, axilla, back of the neck and other hair containing areas.
- Does not need antibiotics.
- A collection of boils is known as a carbuncle. It has multiple openings. Can discharge. May
need I & D with antibiotics.

2016 Nov
45. 50 years old farmer presented with a skin lesion on his right foot. It was an ulceration with
smoothly rolled up margins & pearly colour. What is the most likely diagnosis?
a. Basal cell CA
b. Cock’s peculiar tumour
c. Melanoma
d. Seborrheic dermatitis
e. Squamous cell CA

Answer – A
Rolled up edges are pathognomonic for basal cell carcinoma.

2015 May
10. Regarding skin graft
A. Contractures are a feature
B. Donor area is dressed every other day
C. Full thickness graft does not include dermis
D. Serosal discharge is a contraindication
E. 55% body surface are can be covered by split skin graft

Answers – T F F F T
A - Skin grafts shrink with time. There is early shrinkage which is caused by fibroblasts contracting -
that is called contraction - and late shrinking caused by the granulation tissue being replaced by
collagen. That's called contracture
B – Donor area dressing is kept usually for about 1 week or so
D - All granulation tissue has a serous discharge because unlike skin it isn’t waterproof.
Also, just like the normal skin next to it, if you swab and culture you will always grow something. All
our skins will.
So even a culture positive is not a contraindication unless its Strep pyogenes - because that fellow
will keep secreting stuff like hyaluronidase and streptokinase and keep lysing the attachment the
graft is trying to make with the granulation tissue.
E - A split skin graft can be "meshed" by making cuts on it. One reason is to increase surface area.
The other is to let that serous fluid to come out without separating graft from granulation tissue.
With maximum meshing, a piece of split skin can cover an area even 8 times its size, although usually
its maximum of 3 times the size.

You might also like